Nature and Purpose of Business Class 11 Important Extra Questions Business Studies Chapter 1

Here we are providing Business Studies Class 11 Important Extra Questions and Answers Chapter 1 Nature and Purpose of Business. Business Studies Class 11 Important Questions with Answers are the best resource for students which helps in class 11 board exams.

Class 11 Business Studies Chapter 1 Important Extra Questions Nature and Purpose of Business

Nature and Purpose of Business Important Extra Questions Short Answer Type

Question 1.
Differentiate between Economic and Non-economic activities.
Answer:
Distinction Between Economic And Non-Economic Activities:

Basis Economic Activities Non-Economic Activities
1. Meaning These activities are undertaken by people to earn income to meet their material needs. These activities are of social and religious nature and do not generate any economic gain.
2. Purpose/Motive These activities are undertaken with economic motives, for the generation of income and wealth to earn a living. These activities are undertaken by social or psychological motives. The basic purpose is to serve other segments of society.
3. Need Satisfaction These satisfy the economic needs of people engaged such as food, clothing, shelter, etc. These satisfy the social and psychological needs of people engaged.
4. Types of Examples People engaged in economic activities include traders. Manufacturer. Teacher. Doctor. Electrician. Worker, etc. People engaged in household activities, charitable work, social work, the welfare of the poor. etc. are said to be engaged in non¬economic activities.
5. Logic These activities are guided by rational considerations of cost and profits. Sentiments and emotions guide these activities.

Question 2.
Explain briefly the objectives of commerce.
Answer:
Functions/Objectives/Purpose of Commerce: Following are the important objectives of commerce:
1. Earning profit: The prime motive of commercial activities is to earn a profit. It is the profit motive that encourages and motivates us to start a business and undertake commercial risks. No business can survive without making adequate profits to meet its cost of remunerating the factors of production involved in it.

2. Commitment to social values: The prime motive behind commercial activities is profit motive i.e. earning something. It does not mean that the business should indulge in anti-social activities because such activities are not only illegal but also bring a bad name to the business. It must honor social values and safeguard the interest of consumers, workers, and the community. The social objective helps to improve the reputation and public image of a business.

3. Fair return and safety of capital: It is the prime duty of commercial enterprises that they must pay a fair return on the capital invested by the proprietors of the business. The capital should not be lost in the process of commercial activities. Shareholders must receive dividends at competitive rates.

4. Responsibility towards workers – Workers being essential, active, and sensitive factors of production must be rewarded suitably. The business objective is to ensure the welfare of employees by providing good working conditions, fair wages, and facilities such as housing, medical, etc. Such welfare activities help to improve the physical and mental health of workers.

Question 3.
Explain the difference between Industry, Commerce, and Trade.
Answer:
Difference between Industry, Commerce, and Trade:

Points of Difference Industry Commerce Trade
1. Meaning It relates to the production of goods and services. It relates to the distribution of goods and services. It relates to the actual purchase and sale of goods.
2. Capital It requires a huge amount ’ of fixed and working capital for production It requires limited fixed capital but huge working capital. It requires small capital as per turnover.
3. Scope It covers genetic, manufacturing, and constructive industries. It covers trade and auxiliaries to trade. Does it include? ‘Mental and external trade
4. Basis It is the basis of modern business. Its basis is industries and professions. it is based upon commerce.
5. Utility Goods are produced by transforming forms, creating form utility. Goods are produced by a change of place or by storing. Creating place utility. Goods are produced by transferring their possession, thus creating possession utility.
6. Place of work Industries are established at a particular place. It may be a work-shop or factory. Here, goods are transferred from one place to other places The market is the place where trading activities are performed.

Question 4.
Define profession and mention the basic feature of a profession.
Answer:
Profession: A profession is a specialized occupation that involves the rendering of personal services by the use of professional knowledge. Examples of professionals are doctors, lawyers, accountants, engineers, etc. Each professional has a membership of the professional body (such as the Medical Council of India or Bar Council of India) which enforces the code of conduct among the members.

The basic features of a profession are as follows:

  1. A profession requires specialized knowledge and training about the concerned field. They must be evolved through scientific methods of observation experiment and experience.
  2. The membership of the professional body is compulsory in the case of a profession. For example, a chartered accountant must be a member of the Institute of Chartered Accountants of India. The body regulates and develops professional activities and enjoys legal powers as a statutory body.
  3. There is an established code of conduct enforced by the professional body. It contains norms of behavior for the members. Members who violate the code can be de-recognized and disqualified from the profession.
  4. A professional cannot advertise himself if it is prohibited by the professional body.
  5. Professionals charge fees for the services rendered to their clients.

Question 5.
Explain the term ‘Employment’ and give its distinctive features.
Answer:
Employment or Service: A person is said to be employed or in service when he undertakes to render personal services under a contract of employment. In return for his services, the employee gets wage or salary, allowances, bonus, and other benefits. The employer may be government, a government undertaking, or a private firm. Even a professionally qualified person such as a lawyer can join as an employee with some organization and work in return for the specified salary and other benefits and perquisites.

The distinctive features of employment or service are as follows:

  1. There must be a relationship between employer and employee.
  2. It involves performing the duties assigned by the employer under a written or oral contract.
  3. It does not need any capital investment.
  4. The employee gets wages/salaries and related benefits as a reward for the services.
  5. An employee can’t transfer his job to another.
  6. An employee has to follow the rules and norms prescribed by the employer.
  7. There are no standard qualifications to get employment. It all depends upon the nature and requirements of the specific job.

Question 6.
Give a short note on Secondary Industries.
Answer:
Secondary Industries: Secondary industries are concerned with the processing materials which have ready been produced at the primary stage. The output of the primary industry is used as the raw-material for secondary industries.

Secondary industries may be of two types:

  1. Manufacturing industries, and
  2. construction industries.

Manufacturing Industries: These industries are concerned with the processing or transformation of raw-material and semi-finished products into finished products. The products of extractive industries generally become the raw materials of manufacturing industries. Factory’ production is the outcome of the manufacturing industry. Manufacturing industries are of the following types.

(a) Analytical: In the analytical manufacturing industry, basic raw-materials are broken into several useful materials. Petroleum refining is an example of an analytical industry. The crude oil is extracted from beneath the earth and is processed and separated into petrol, kerosene, gasoline, lubricating oil, etc.

(b) Synthetic: Two or more materials are mixed together in the manufacturing operations to form some new products. Products like soap, Cement, paints, fertilizers, and cosmetics are produced by synthetic industries.

(c) Processing: In this industry, raw materials are processed through various stages to make the final products. Textiles, sugar, and steel are examples of this category.

(d) Assembly Line: In the assembly industry, the finished products can be produced only after various components have been made and then brought together for final assembly. Production of automobiles, watches, televisions, bicycles, railway wagons, etc. is some typical examples of this industry.

Construction Industries: These industries are concerned with the construction of buildings roads, dams, and bridges. These industries use the products of manufacturing industries such as iron and steel, cement, lime, mortar, etc., and also the products of extractive industries such as stone, marble, etc. The remarkable feature of these industries is that their products are not sold in the sense of being taken to the markets. They are constructed and fabricated at fixed sites.

Question 7.
In today’s competitive world, the service industry plays a major role. Explain the tertiary or service industry.
Answer:
Tertiary or Service Industry’ – In this sector, various types of services may be classified into two broad categories commercial and personal. They serve as the backbone of the modern industrial system.

Territory activities include transport, banking, insurance, warehousing, and advertising which provide the infrastructure for industry and trade. In other words, these are service activities that support business enterprises. Since these are all commercial activities, firms engaged in banking, transportation, insurance, etc. are known as commercial enterprises. Moreover, the performance of these services involves distinct operations and processes. That is why they are also called ‘Service industries’.

Tertiary units bridge the gap between the producers of goods and services and their consumers. They help in removing various hindrances that arise during the production and distribution of goods and services.

The contribution of these industries is stated below:

  1. Transport facilities overcome the barriers of distance and create place utility. Transport widens the market and helps to equalize prices at different places.
  2. Banking provides credit facilities to industrial and trading firms besides providing banking services. Bank also facilitates business activity by providing, safe and quick means for remittance of money.
  3. The insurance covers various kinds of business risks.
  4. Warehousing provides storage facility to the producers and traders. It removes the hindrance of time and thereby creates time utility.
  5. Advertising provides information to consumers.

In short, the above activities help industry and trade through movement, financing, risk, coverage, storage, and promotion of goods.

Question 8.
What are the various sectors of industrial organizations in India? Mention briefly.
Answer:
Various industrial organizations may be grouped into three sectors as follows:
1. Primary Sector: Primary industries include all those activities which are connected with the extraction, production, and processing of natural resources. Farming, mining, oil drilling, lumbering, fishing, etc. are examples of the primary sector- activities. Genetic and extractive industries come under the primary sector.

2. Secondary Sector: Secondary industries are concerned with the processing materials which have already been produced at the primary state. The output of the primary sector is used as the raw material in the secondary sector production. For example, growing cotton is the primary industry but the manufacture of textiles from cotton is a secondary industry. Steelmaking, ship-building, pottery, oil refinery, carpentry, house-building, interior decoration, plastic manufacturing, etc. are examples of secondary production. The secondary sector consists of manufacturing and construction industries.

3. Tertiary Sector: This sector industries include those services which facilitate a smooth flow of goods and services for satisfying consumer needs.

These services may be classified into two broad categories:
(a) commercial, and
(b) personal.

Commercial services include trading, transport, warehousing, insurance, banking, packaging, advertising, communications, etc. Personal services refer to teaching, nursing, police, medical, detective, entertainment, etc. Thus, tertiary industries provide support services to primary and secondary industries and facilitate trade.

Question 9.
Define the terms small business, tiny units, and ancillary units.
Answer:
Definition of Small business: A small business may be defined in terms of size. Once a firm goes beyond a certain size, it is no longer classified as small. The size limit for a small firm may be laid down in terms of any one or more of the following criteria, namely,
(a) the amount of capital invested,
(b) the number of persons employed, and
(c) the value of annual turnover.

The small scale enterprises have been classified into three segments: small units, ancillary units (those manufacturing components and spare parts), and tiny units (very small units including handloom and cottage industries.)

The present limit of investment in plant and machinery’ for these segments is as follows:

  1. Tiny Units having an investment in plant and machinery up to Rs. 25 lakhs.
  2. Small Scale Industrial (SSI) units having an investment in plant and machinery up to Rs. 100 lakh (Rs. One Crore).
  3. Ancillary Industrial Units having an investment in plant and machinery up to Rs. 100 lakh.

It has been the policy of the Central Government to give encouragement to entrepreneurs in the small sector. For the growth of small enterprises, the government provides various incentives and concessions in the form of capital subsidy, technical know-how, market, and infrastructural facilities.

Several institutions like State Finance Corporations, Small Scale Industries Corporations, Small Scale Industry’ Development Organisation, Small Industries Service Institutes, Directorate of Industries, District Industries Centres, and Small Industry Development Bank of India have been set up to help the small entrepreneurs in their pursuit.

Despite the growth of multinationals and other big firms, small business has survived and there is considerable scope for setting up small business firms in India. Liberalization and new technology have opened up new areas of small business complementarity to big business.

Nature and Purpose of Business Important Extra Questions Long Answer Type

Question 1.
Explain in detail the social objectives of the business in today’s context.
Answer:
Social Objectives: Business does not exist in a vacuum. It is an integral part of society and it can achieve its economic objectives only by having deep roots in the society. According to Henry Ford, “The purpose of business is not only earning profit but also discharging responsibilities towards the society,” A business must be guided by social objectives since it is a part of the society and gets men, materials, and machines from the society only.

The decision taken by the business has a great influence on the socio-economic conditions in the country. Business is not merely an economic entity, it is a social institution as well. Therefore, it is in the interest of business to pursue certain objectives that are expected by people.

The social objectives of the business are as follows:
1. Better Quality Goods at Fair Prices – The business must provide better quality products as desired by the customers. Quality means purity as in the case of food items and medicines or safety as in the case of electrical goods or durability as in the case of TV and refrigerator. Consumers prefer the products which are of satisfactory quality and are available at reasonable prices.

In fact, consumers have become increasingly conscious of quality and want value for money. Therefore, an important objective of a business is to produce and supply goods of proper quality to satisfy the expectations of consumers. The prices charged for the goods should also be reasonable.

2. Fair Trade Practices: Anti-social practices include hoarding, black marketing, and adulteration. Making false claims in advertisements to mislead and exploit people is an example of unfair trade practice. Such practices are not only illegal but bring a bad name to the business community. Therefore, businessmen must avoid such means of making money. The business should follow fair business practices all the time.

3. Generation of Employment: Every business should grow and expand its operations to create new frontiers of employment for society. The business has tremendous scope for the generation of employment opportunities. Business must provide employment without any discrimination on account of caste, creed, religion, or sex. Business is expected to give special consideration to handicapped and weaker sections of society in the matter of employment. Business firms that pursue this objective can improve their public image.

4. Employment Welfare – Employees are a valuable asset and they make significant contributions towards the success of the business. It is an important responsibility’ of the business to promote the welfare of the employees. Businesses must recognize the dignity of labor and treat employees as partners rather than as mere working hands. Businesses must provide good working conditions, housing, transport, and medical facilities besides fair wages to their employees.

Question 2.
Can profit be the sole objective of a business? Explain the reasons in favor and against the profit objective of the business.
Answer:
Can profit be the sole objective of a business? – Despite the indispensable role in business, profit cannot be the all and all of the business. Profit maximization objective is undesirable and social accountability is also the responsibility of business. According to Urwick, “Earning of profits cannot be the objective of a business any more than eating is the objective of living.”

A business unit is an economic entity in which various factors of production are used. Capital is one of the factors of production and the reward for investing capital is given in the form of profit. Therefore, a business should not be run only to maximize the reward of one factor of production, i.e., the capital. Besides’earning profits, it should also aim at the satisfaction of customers, the welfare of workers, community service, etc.

The argument in favor of profit as the sole objective: Earning profits is essential for a business due to the following reasons:

  1. It is a sign of healthy business as profit is the chief motivating factor in business.
  2. It would provide sufficient return to the investors of capital. Profit is considered to be an index of success in business.
  3. It would provide funds for reinvestment in the business.
  4. A profit-making concern enjoys goodwill in society.
  5. The assets of the business would be used for maximizing profits. Misuse of assets would thus be avoided.

Arguments against profit maximization: Profit maximization should not be the sole motive of any business. The arguments against profit maximization are as under:

  1. Profit maximization ignores the interests of labor, customers, and society.
  2. Unfair means such as hoarding, black marketing, or adulteration may be followed to maximize profits.
  3. The long-term interest of the business may-be ignored to maximize profits in the short-run.
  4. In the present-day environment, a business can’t be effective with the sole objective of profit maximization. It must also set objectives in areas like customer satisfaction, social responsibility, environmental protection, research, and development, etc.

The profit-making and social service objectives of the business are not contradictory to each other, they go hand in hand. According to Henery Ford, “Mere money chasing is not business. The businessman who keeps his customers satisfied by service will definitely earn good profits.

To conclude earning profits through service to society is the real objective of the business.

Question 3.
Explain in detail the various types of risks.
Answer:
Types of Business Risks – Business is subject to a wide variety of risks. The different types of business risks may be classified in the following ways:
1. Pure and Speculative Risks: ‘Pure risks’ are those risks that relate to chances of loss and there is no possibility of profits. For instance, when a fire breaks out there is a chance of loss only, no gain. Theft, accident, strike, lockout, damage in transit are some examples of pure risks. A businessman may avoid, insure or simply assume the pure risks.

‘Speculative risk’ are those risks which make the possibility of both losses or gain depending on the future. For example, the development of a new product may result in a large number of profits or big losses. Changes in demand, price fluctuation, changes in fashion and taste, etc. are examples of speculative risks. Speculative risks can be reduced, avoided, or shifted to others.

The distinction between pure risk and speculative risks may be presented as under:

  1. First, pure risk is always inherent in business whereas speculative risks are deliberately assumed by a businessman.
  2. Secondly, pure risks may or may not result in a loss but they never bring extra gain to the entrepreneur. On the other hand, the speculative risk may cause loss or gain. Thirdly, the pure risk is generally insurable but the speculative risk can not always be insured. Lastly, speculative risk enables a businessman to earn profits while pure risk fails to do so.

The pure risk may be categorized as under:
(a) Personal risks: These risks relate to the individual capacity loss of earning. Old age, sickness, disability, unemployment, premature death, etc. lead to loss of income or assets.

(b) Property risks: Property risks are those risks that relate to the loss of property. Direct physical loss or damage to property, loss of income from the property, non-availability of property for use, additional expenses incurred on the property to make it usable, etc. are examples of property risks.

(c) Liability risks: These risks involve the possibility of loss due to the damages or compensation payable to third parties on account of intentional or unintentional torts or injury to the rights of others.

2. Insurable and non-insurable risks: Insurable risks are those risks that may be avoided by insuring them. Goods in stock or in transit can be insured against fire, theft, etc. The essential features of insurable risks are as follows:
(a) The risk must arise out of the ordinary course of business. It must be accidental without the fault of the insured.
(b) There must be an element of uncertainty as to the occurrence of risk or the time of its occurrence.
(c) The risk must be common to the units insured.
(d) The loss or incidence of risk must be foreseeable and capable of being estimated or measured, with a fair degree of accuracy.
(e) The loss must be large enough to cause hardships.

Non-insurable risks cannot be insured against because their occurrence cannot be forecasted and. determine. In the words of Hall, “Those uncertainties which cannot be forecasted and which are caused due to lack of business entrepreneurship, lack of mental presence, cannot be insured and, therefore, they are non-insurable risks.” Changes in demand and supply, price fluctuations, changes in fashion, etc. are examples of non-insurable risks.

3. Internal and External Risks: Internal risks are those risks that occur during the normal course of business running. Fire, breakdown of machinery, negligence or dishonesty of employees, a strike by the workers of the firm are examples of internal risks. External risks involve those losses which result from forces beyond the control of the business. Changes in market conditions, technological changes, political changes, natural calamities, social disturbances, etc. are examples of external risks. Management has comparatively little control over external risks.

4. Fundamental and Particular Risks: Fundamental or general risks are group risks that affect the general group or large segments of the public. These risks are impersonal in origin and consequence. Floods, earthquakes, cyclones, famine, storms, wars, inflation, unemployment, etc. are examples of fundamental risks. These risks are caused by factors that are beyond the control of the individuals who suffer the loss. Society is expected to shoulder such risks because they are not the fault of any particular individual.

Particular risks relate to individuals responsible for their occurrence. Bank robbery, burning of a factory, murder of a manager, etc. are all particular risks. Such risks are the responsibility of the particular individuals who cause or suffer them.

5. Static and Dynamic Risks: Static risks are those risk which has no bearing on the economy. Such risks lead to the destruction of an asset or changes in its possession. Human factors, dishonesty of employees, natural calamities, etc. are examples of static risks. Dynamic risks affect the economy, e.g., price level fluctuations, changes in income and output, technological changes, changes in consumer tastes, etc. Dynamic risks are the result of adjustments to the misallocation of resources. Therefore, they are a source of gain to society in the long run. Dynamic risks are less predictable as they do not regularly occur.

6. Property and Personal Risks: Property risks relate to the risk of property due to natural or man-made causes. For example, floods may destroy crops. On the other hand, personal risks relate to the risk of the personal life of workers working in the business. Such risks may occur due to accidents, occupational diseases, etc. For instance, a worker may lose his right hand due to an accident while working in the factory.

Question 4.
How the business risks can be prevented? Explain the preventive risks.
Answer:
Preventing Risks: Various groups interested in business Le. individual, firms, or government plays an important role to prevent the risks arising in the business. The management of an individual firm can take steps for loss prevention and control. Efficient planning and effective control help to reduce risk.

The main techniques of reducing business risks are as follows:
(a) Prediction and Marketing Survey: Improper planning causes many risks in business. Scientific forecasting of future economic conditions makes the management aware of likely opportunities and threats in the future business environment. As a result, management can formulate appropriate plans in advance to meet the challenges of the future.

Marketing surveys help in providing information about market conditions helpful to a businessman can make the necessary change in products, prices, distribution channels, and sales promotion techniques. Efficient market planning help in reducing the risk of over-production, wrong products, defective distribution, etc. An intensive selling campaign may be used to maintain regular demand and to build up brand loyalty among consumers,

(b) Research and development: Losses due to technology change, maybe overcome through scientific research and development. It can develop new and remunerative products before the present products become obsolete! Research and development are also helpful in standardization and control of quality so that consumers can get safe and reliable products.

(c) Credit screening and control: Careful screening of customers and prompt collection of outstanding debts are useful in reducing the possibility of loss through bad debts. Similarly, proper inventory control can reduce the risks of loss.

(d) Safety programs: Risks may be avoided with proper safety programs. Cold storage or refrigeration is helpful in the preservation of perishable products. Special packing may be used to reduce spoilage or leakage of goods in transit or storage. Similarly, steps can be taken to reduce damage by rats, pests, vermin, etc. Medical care facilities help reduce the loss of life on account of accidents in the factory. Adequate lighting, covering of damaged floors, keeping aisles free of obstructions, etc. help reduce accidents.

(e) Training and development of employees: Proper training to successor employees may be helpful to reduce the risk in the care of death, resignation, or incapacity of a key executive. Similarly, training workers helps reduce the incidence of industrial accidents and spoilage.

(f) Business combination: The risk of competition can be reduced through collective action by the competing firms which may agree to restrict output, allocate markets or charge uniform prices. Business combinations can avoid excess supply, a fall in prices, and combative advertising.

(g) Government regulation: Government regulatory mechanism may reduce business risks. The government may impose import duty to protect domestic industry from foreign competition. It may stabilize prices, freight rates, taxes, etc. to make the environment of business less risky.

Question 5.
Mention the activities auxiliaries or Territory to trade.
Answer:
Auxiliaries To Business/Trade: (Tertiary Activities or Aids to Trade, Business, and Industry): Activities that assist or support the trade are known as auxiliaries to business or trade. They are an integral part of commerce. These include transport, warehousing, insurance, financing and banking, and other allied services which are known as aids to trade.

These services are discussed below:
1. Transportation: Transport helps in removing the hindrance of place in the exchanges of goods and services. It results in the equitable distribution of goods in far-flung areas. Transport has linked all parts of the world with the help of efficient means of transport.

2. Banking: Banks provide a device through which’ payments of goods bought and sold are facilitated. In other words, banks facilitate the purchase and sale of goods on credit. Banks also perform the useful economic function of collecting the savings Of the people and business houses and making them available to those who may profitably use them. Thus, banks may be regarded as traders in money and credit.

3. Insurance: With the help of insurance, a businessman can protect himself from various risks. An insurance company performs a useful service of compensating for the loss arising from the damage caused to the insured goods through fire, pilferage, theft, flood, and the hazards of sea transportation. Insurance is based on the “pooling of risks” principle.

4. Warehousing: There is generally a time lag between the production and consumption of goods. This problem can be solved by storing the goods in warehouses. Storage creates time utility and removes the hindrance of time in the trade. Warehousing these days has become an important element of the business.

5. Advertising: Advertising and publicity inform the consumers about the availabilities of products & services. In the absence of advertising, goods would not have been so due to a widely scattered market. It is through advertising that the customers come to know about the new products and their utility. Because of the physical spatial distance between the producers and the consumers, advertising is necessary to bridge the information gap.

5. Packaging: Packaging is traditionally done to protect the goods from damage in transit and to facilitate the easy transfer of goods to customers. Packaging helps in the conveyance and handling of goods safe and free from spoilage. Trade and Transport of goods have become easier and safer due to improvement in methods of packaging.

Development Class 11 Important Extra Questions Political Science Chapter 10

Here we are providing Class 11 Political Science Important Extra Questions and Answers Chapter 10 Development. Political Science Class 11 Important Questions with Answers are the best resource for students which helps in class 11 board exams.

Class 11 Political Science Chapter 10 Important Extra Questions Development

Development Important Extra Questions Very Short Answer Type

Question 1.
Explain the concept of development.
Answer:
The word ‘Development’ carries many connotations. It is a very wide concept with different aspects. The very purpose of the development is to build a modem society, to change the face of not only every section of the society but of every man by taking the fruits of development to the last man. It seeks to make maximum utilization of all the resources and to make their just distribution. It is opposed to the concentration of resources in few hands or the resources remain unexploited. Therefore it is a very positive concept, however, it has been understood in a negative sense only.

Question 2.
Mention some main objectives of the development.
Answer:
Although the concept of development is a wide term with broader objectives. Its main objectives can be specified as under

  1. Identification of unexploited resources.
  2. Maximum utilization of available resources.
  3. To build a modern society by transforming the traditional and backward society.
  4. To ensure that fruits of development reach the last man in the queue.
  5. To make the change and growth human-oriented.

Question 3.
What are the targets of Development?
Answer:
Followings are the areas of targets of Development:

  1. Poverty
  2. Unemployment
  3. Unexploited Resources
  4. Industrialization
  5. Modernisation of agriculture
  6. Illiteracy
  7. Low per capita income and low per capita consumptions.

Question 4.
What are the main challenges of Development?
Answer:
Followings are the challenges of development in developing countries of Asia and Africa

  1. The low standard of living
  2. Illiteracy
  3. Poor health
  4. Malnutrition
  5. Lack of basic amenities
  6. Feudal agrarian system
  7. Legacy of colonial economies of under-developed countries.

Question 5.
What are the main features of the development model of India?
Answer:
India has adopted a socialistic pattern of society which is based on the principle of From each according to his ability and to each according to his ability. We have a mixed economy in which some economic activities are to be undertaken in the public sector and others are taken in the private sector. We have adopted planned economic development with a five-year plan system. Some targets are fixed for realization in every plan. Economic pursuits are undertaken in different fields with the collaborations.

Question 6.
What are the social costs of development?
Answer:
The process of development has its social cost which can be mentioned as under

  1. Displacement of the people from their homes
  2. Change in culture
  3. Illiteracy
  4. Adoption of a new culture
  5. Loss of livelihood
  6. A search for a new identity
  7. a Large number of rural and urban poor.

Question 7.
Explain in brief tire environmental costs of development.
Answer:
The unplanned development has definitely rather, unfortunately, has its environmental cost which can be understood as under

  1. Environmental degradation.
  2. Displacement of the people from one area to another area.
  3. Natural disasters like Tsunami and earthquakes etc.
  4. Global warming.
  5. Greenhouse effects.
  6. Water pollution.
  7. Air pollution.
  8. Sound pollution.
  9. Ecological disorder.
  10. Energy crisis.

Question 8.
What is sustainable development?
Answer:
The present model of developments seems to be oriented to the present needs of mankind, neglecting the needs of the future generation. Sustainable development is an approach to development that seeks to use the presently available resources in such a way that:

  1. There should not be misuse and wastage of present resources.
  2. All the resources should be exploited fully.
  3. There should be conservations of energy.
  4. Policies should be future-oriented.

Question 9.
What do you understand by the term environmentalism?
Answer:
Environmentalism stands for environmental movement which works to protect natural resources and the eco-system. It emphasizes that human beings should learn to live in harmony with the requirements of the eco-system and should not manipulate the natural environment to serve the personal petty interests which harm nature and the cause of future generations.

Question 10.
Discuss the impact of environmental movements.
Answer:
The different environmental movements started in different parts of the world have been able to evoke public consciousness and awareness and also successfully pressuring to modify the governmental policies in the light of environmental goals and sustainable development.

The roots of environmentalism can be traced back to the nineteenth century. Today the environmental movement has become a worldwide phenomenon with thousands of Non-governmental groups and even some green political parties are pursuing the cause of protection of the environment. In India Chipko movement started by Sunder, Lai Bahuguna has been playing an effective role.

Question 11.
How growth and development are related?
Answer:
Growth related to the advancement in agricultural and industrial areas and progress in infrastructure and production in agriculture and industrial areas thereby affecting all other aspects of human life but it does not necessarily ensure the just and equal distribution of the fruits of such growth. Development seeks to ensure this distribution of progress and change of material development and development in infrastructure when economic growth and redistribution of the benefits do not go together, the benefits are cornered by few development seekers to break this chain for improving the standard of living of underprivileged.

Question 12.
What are social indicators of development?
Answer:
Followings are the main social indicators of development:-

  1. Literacy and education level.
  2. Life expectancy.
  3. Maternal mortality rates.
  4. Housing.

Question 13.
What do you understand by the basic need approach?
Answer:
The human development report of the United Nations Development Programme (UNDP) ranks countries on the basis of their performance in social indicators like literacy, life expectancy, and maternal mortality rates which are called as Human Development Index. According to these parameters, the development should be a process that allows more and more people to make meaningful choices. For this fulfillment of basic needs like education, health, and shelter. This approach is called as basic needs approach.

Question 14.
What do you mean by right claims?
Answer:
Since the fruits of development have not reached the people for whom the development meant. It is desirable that people should be given the right to claim their dues from the state and society. It is necessary to check the process of concentration of resources in the hands of few privileged sections of society. It becomes more essential in the democratic framework of the government where people have the right to be consulted in the decision-making process, which is related to them.

Question 15.
How the democratic participation is useful for development?
Answer:
Democratic participation ensures the maximum involvement of maximum people in the decision-making process in the democratic framework of the government. Democratic participation is useful to the development. Both democracy and development are concerned with the realization of the common good and the welfare of maximum people. The people’s participation in the decision-making process makes the development people’s oriented. A decentralized approach can bring, people’s participation and development together.

Development Important Extra Questions Short Answer Type

Question 1.
Explain the meaning of the term ‘Development.
Answer:
The meaning of the concept of development is different for different people. One thing is definitely common that development means the diffusion of growth to all sections of the society to raise the standard of life of the people. The development conveys the ideas of improvement, progress well-being, and inspiration for a better life. Through the process of development, society seeks to achieve the vision of the future for the future generation.

The development has also been Understood and used in a narrow sense y understanding it related with limited goals such as increasing the .ate of economic growth or modernizing the society. In fact, the development is not concerned with just achieving the present targets or completing projects like dams and factories i.e. growth of infrastructure but it is more concerned with realizing the broader vision of society. We can say that development is a value ridden concept.

Question 2.
Explain various issues that are involved in the process of development.
Answer:
Since the concept of development is very wide, a number of issues are involved in it which are as under

  1. The rights of the people should be taken into account during the course of development.
  2. To make development more pervasive, there should be the democratization of the social, economic, and political environment.
  3. The benefits of development should reach all sections of society.
  4. There should be qualitative improvement in the lives of the people.

Question 3.
Explain socialist, communist, and capitalistic model of development.
Answer:
Every state has its own model of development. Every state is concerned with the welfare of its people and raising the standard of living of its people. Although the number of models has been evolved there have been three main models of development in prevalence. These models are as under

  1. Socialistic model
  2. Communistic model
  3. Capitalist model

1. Socialistic model:- Socialist pattern of development prevailed in developing countries of Asia and Africa to make distribution of resources as per the reeds of the people and to exploit the potentialities of the people to the fullest.

2. Communist model:- Communist model of development initially started in the countries of Eastern Europe and then spread to some other countries. It was based on the principles of Marxism.

3. Capitalist model:- This model of development become popular in the USA. and European countries which are based on the principle of economic pursuits on competition basis and profit basis with the minimum role of the state.

Question 4.
What are the main targets of development?
Answer:
The process of development seeks to achieve positive changes and improvement in the following areas

  1. Poverty
  2. Unemployment
  3. Illiteracy
  4. Unexploited resources
  5. Regional imbalances
  6. Social backwardness.
  7. Fatalism
  8. Impact of colonialism.

Question 5.
Enumerate certain features of development undertaken in different parts of the world.
Answer:
The process of development has undergone many changes over the years in different countries as it is understood in its meaning. Initially, it was linked with the economic growth and social development with the purpose of modernization of societies. The main features and activities undertaken were as under:

  1. Industrialization
  2. Commercialization
  3. Urbanization
  4. Agrarian reforms
  5. Use of science and technology
  6. Educational development
  7. Secularisation
  8. Democratization.

In the process of development, the state was given a leading role to play in the above areas.

Question 6.
Explain the mode of development in India.
Answer:
After independence, India got the colonial legacy at the social, economic, and political levels. India adopted a mixed economy for its development. Planning Commission was set to start planned economic development. The first five-year plan was started in 1951-56. So far we have successfully completed the ten five years plan which has helped in transforming the Indian society and economy to an appreciable extent. Today India is on its way to being a modernized society. The process of liberalization, globalization, and privatization has fastened the pace of development. In India, the process of development is going in Libera secular democratic framework. All efforts are being made to take the benefits of development to all sections of the society through their participation at all levels.

Question 7.
What is the impact of development in India?
Answer:
India, through its planned socio-economic development, has achieved tremendous growth in different areas i.e. agriculture, roads, housing, industry, medicine, health, raising employment opportunities, education, and developing infrastructure. Through the doses of modernization, India has been able to transform Indian society in the last sixty years. In fact through the development face of India is changed from an agrarian, traditional and backward, and undeveloped society to a modem, secular, democratic, and developing society.

But still, we have not been able to achieve the goals of development in the real sense and as per the true meaning of development. We are still in grip of inequalities, disparities, and imbalance. Still, we have 26% of people who are yet to reach the level of subsistence needs. Still, we have situations of excess and deprivation. The ultimate objective of development is rational utilization of the resource and to take the fruits of development to the last man. We feel that we are far away from these two goals of development.

Question 8.
What are the main hindrances in achieving the goals of development in developing countries like India?
Answer:
Almost every society is in the process of development and is trying to get the goals of development. But there are certain structural hindrances in the path of development, particularly in developing countries. Some of the hindrances are as under:-

  1. Lack of definite and relevant model of development.
  2. Lack of political will.
  3. Lack of people’s awareness.
  4. Traditional social structure.
  5. Traditional occupations.
  6. Lack of people-oriented approach of bureaucracy.

Question 9.
What do you know about the social cost of development?
Answer:
The process of development involves a lot of desirable and undesirable activities with negative and positive consequences which can be called social costs. These social costs can be understood as under

  1. Displacement of persons due to developmental projects like bridges and dams and other such projects.
  2. Migration of people in search of better opportunities and better quality of life in which they lose their old identities and due course of time acquire new values and new identities.
  3. Pressure on urban civic amenities due to the process of urbanization in the wake of development.
  4. Loss of traditional skills acquired in a long time.
  5. Loss of culture.
  6. Regional and cultural confrontations.
  7. Social agitations.
  8. Unemployment and uncertainties.

Question 10.
Explain the environmental costs of development.
Answer:
The environmental cost of development can be measured in the following ways.

  1. Pollution.
  2. Ecological crisis.
  3. Environmental degradation by the phenomenon like depletion of ozone, ozone hole, global warming, and Green House effects.
  4. Natural disasters like earthquakes and Tsunami.
  5. Loss of human life.
  6. Loss of forest due to deforestation cause havoc.
  7. Loss of subsistence needs of the poor.
  8. Wastage of medicinal plants.
  9. Loss of traditional sources of energy, therefore, leading to the energy crisis.
  10. Dirty drinking water.

Question 11.
What is the meaning and objectives of Environmentalism?
Answer:
The thinking and supporting, the thinking of protecting the environment and environmental life is known as environmentalism. It is therefore as an urge and passion to save the environment and human life from the hazards which are being played in the name of development.

Environmentalists maintain that human beings should learn to live in harmony with the rhythms of the eco-system and should not indulge in such selfish activities that are destroying the beauty and utility of the eco-system and environment. It is a fact that if this trend is not checked/stopped, we will get barren earth, poisoned rivers, and polluted air for future generations. Due to this fear, the environmental movement has become a worldwide movement with the support of government and non-governmental organization who have been successful in modifying
the government’s industrial and developmental policies in the right of goals of protection of the environment.

Question 12.
Explain the relationship between growth and development.
Answer:
Although the two concepts i.e. growth and development are very frequently used for each other and simultaneously but there is a perceptible difference between the two, although both are related to the welfare of man and society. Growth denotes a change in infrastructure and progress in different areas. The growth has not been able to achieve the goals of social justice. It is largely material development. On the other hand, the concept of development is broader than growth and can be understood as a process that seeks to improve the quality of life of the people at large. Growth is not an indicator of social justice. It is the development that is an instrument and measure of social justice and quality of life of the people.

Question 13.
How the development can be measured?
Answer:
Measuring economic growth is not adequate as the indicator of social and economic progress and social justice. The need of measuring the development was felt. Several attempts have been made in this direction. One such attempt is the Human Development Re| which is annually brought out by the United Nation Development Programme (UNDP) which ranks the different countries of the two, on the basis of their performance in social indicators like education, lii expectancy, and maternal mortality rates. This measure is called the Human Development Index. This proves that the development revolves around the basic needs of man. Thus development is a process which allows more and more people to make meaningful life with the minimum adequate availability of basic needs like food, education, shelter, and health.

Question 14.
What do you mean by the Top-down approach of development? What changes does it need?
Answer:
There has been the social and environmental cost of development at different levels, due to which the development could not bring the desired results. The benefits of development have not been evenly distributed among the people. It has been due to the ‘Top-down’ approach, which has been the strategy of development. The very important feature of the Top-down approach has been the selection of priorities and preferences and methods of implementation of the decisions and project^ were all generally decided by the top bureaucrats, politicians, executives, and technocrats with the actual participation of those for whom these decisions are taken. This was the scenario of both the democratic and non-democratic countries. Thus the need for alternate model development is felt.

Question 15.
What do you understand by the Concept of Right Claims?
Answer:
In most of the models of development the benefits of change, progress and development have not been reached to the people for which they were meant and are cornered by the privileged sections of the society. It has been due to the Top-down approach in the developmental models. Now the time has come and which duly accepted and recognized that the aspirations and needs of the affected people should be taken into account. People should be allowed to put up their views, rights, and claims before the decision-makers.

In a democratic framework, people have the right to be consulted in the exercise of decision making and in the process of development. The people have their natural; rights on the available resources for developmental activities. Therefore on the fruits of development, local people have a natural claim which should be duly given to make development meaningful.

Question 16.
What is the significance of Democratic Participation in the process of development?
Answer:
Democracy is a system of government that provides an opportunity to the people to participate in the decision-making process and the developmental activities which are a pre-requisite of the success of democracy. Democracy also aims to provide a better quality of life by establishing equality, justice, fairness, brotherhood, the dignity of man, and ensuring people’s participation.

Democracy and development, both are concerned with realizing the common good. The most important relationship between democracy and development is that development is supposed to be undertaken by people’s participation at the micro-level and macro-level.

Development Important Extra Questions Long Answer Type

Question 1.
Define development, its objective and suggest the best modes of development.
Answer:
Lucian Pye, a great politician scientist explained the idea of development in his famous book ‘Aspects of Development’ “as a process of rational utilization of the available resources to establish modern society. In the contemporary world, however, the word of development has been used for progress, change, modernization, and growth. In fact” Lucian Pye explained the concept of development in terms of socialization, modernization, democratization, secularisation, and people’s participation.

As far as the objectives are concerned, they are not just material progress in different fields like production, goods, amenities, dams, projects, and the well-being of a few. In fact, its objectives are wide. The ultimate objective of the development is as under:-

  1. To make the best use of the resources by identifying them properly.
  2. To ensure the fair distribution of fruits of development to all sections of society, particularly up to the underprivileged class.
  3. To raise the standard of living of the people.
  4. To achieve the common good of the people.

There have been different models of development in different parts of the world with democratic and dictatorial governmental structures In all these models of development, there has been the Top-down approach in decision making which prohibited the local initiative and participation in policymaking and decision-making process. To achieve the desired goals of development, there is a need to adopt such a system or model of development which ensure the people’s participation up to a desirable extent and recognizes their rights and claims.

Peace Class 11 Important Extra Questions Political Science Chapter 9

Here we are providing Class 11 Political Science Important Extra Questions and Answers Chapter 9 Peace. Political Science Class 11 Important Questions with Answers are the best resource for students which helps in class 11 board exams.

Class 11 Political Science Chapter 9 Important Extra Questions Peace

Peace Important Extra Questions Very Short Answer Type

Question 1.
What is Peace?
Answer:
Peace is one of the most desired situations not only by the saints and philosophers but also by every ordinary person. Even those who get indulged in violence do the violence in search of peace. The concept of peace has been conceived and understood differently by the people of different ages. Generally, peace has been defined as the absence of war. But this definition of peace is incomplete and misleading. Peace is something mental and psychological. Peace is freedom from unnecessary tensions and worries at the individual level as well as collective level. It is also a fact that peace is a personal and relative matter. At the individual level, peace depends, how he/she takes the things!

Question 2.
What do you mean by structural violence?
Answer:
When the conflict, exploitation and tensions are at the institutional level in perpetuation, it is called as the structural violence. In, such structural violent situation, peace will the immediate causality. For example, the traditional Indian caste system which was based on hierarchic system disclosed a group of people as untouchable and put them at the lowest ladder of the society. It naturally bred hatred, tension and conflict. Similar is the situation of colonialism, where one country dominated, exploited and subjected enter country leading to violence and revolt.

Question 3.
How inequality harms the cause of peace?
Answer:
Inequality in any form is certainly injurious to the cause of health. Any social order based on inequalities will not have peaceful and harmonious relationship^imong the members of that society because’ these will be tension, competition, dominance and hatred.

Question 4.
Give the names of the system of structured violence.
Answer:
There can be any system of structural violence. Following are the examples from history in which structural violence prevailed.

  1. Traditional Indian social order in which a group of people was declared untouchable.
  2. Indian family system, where women were declared inferior and incapable.
  3. Colonialism which institutionalised group exploitation of other countries.
  4. Racism, where a group of other caste and race was exploited.
  5. Communalism, where people of another religion were exploited.

Question 5.
How structural violence caused harm to peace?
Answer:
Different forms of structural violence like colonialism, racism, communalism, hierarchic social order and low status of women prolonged the tension and make a particular type of psyche which institutionalised not only inequalities and exploitation but tension and conflict also. Psychological and tangible harm suffered by the victims of violence created grievances which were transmitted from generation to generation. In this way, such structural violence harmed peace for a long time to come.

Question 6.
How the violence can be eliminated?
Answer:
It seems to be difficult if the not impossible proposition that violence can be eliminated from society totally. It needs healthy psyche, mind frame built-in a congenial and healthy environment. Several age-old spiritual principles like compassions, meditations and preachings have been playing their role in their own way to make healthy psyche and thinking of man to establish peace.

Question 7.
Can violence be helpful in the establishment of peace?
Answer:
Although, it is generally thought that violence is used to establish peace. Particularly at the state level, it is used for establishing peace. It is because of this the army and police are maintained and is trained in the use of weapons for violence and wars, But it is all superficial view. The everlasting peace can only be achieved by the positive makeup of mind frame which can be possible with the positive environment built on the basis of harmonies and cooperative relations among the people of the society.

Question 8.
What are the reasons for the growing violence in our society?
Answer:
There are a number of reasons for the present growing state of violence in the present-day society, but we can give some of them, which are as under

  1. Intolerance
  2. Increasing selfishness
  3. Excessive view of casteism, communalism and regionalism
  4. Increasing culture of consumerism and naturalism
  5. Criminalisation of politics
  6. Corruption in society
  7. Lust for power and money
  8. Structural violence.

Question 9.
What exactly is peace? Why it is fragile in today.
Answer:
Peace is in fact a state of mind free from worry, tension, hatred and fear. It is related to the framework of mind and mindset. Peace is a very positive situation in life. In today’s world, it is found to be very fragile i.e. it is in a disturbing state. The environment of societies has become violent and uncertain. There is a situation of mistrust and distrust. Most of the people are in mental tension in such situation peace has become very much unachievable.

Question 10.
How does the human mind help in establishing peace?
Answer:
Human mind shapes the behaviour of man and behaviour of man creates peace or tension therefore mind plays a significant role in establishing peace. The Constitution of the United Nations Educational, Scientific and Cultural Organisation rightly observes “Since war begins in the minds of men, it is in the minds of men that the defences of peace must be constructed.” For this several sages and saints preached the spiritual principles of tolerance and compassion and practices of meditation were followed to make a mindset so as to establish and promote peace.

Question 11.
Do you agree that non-violence means weakness?
Answer:
The main reason for increasing violence in society is a misunderstanding about non-violence. Many people equate non-violence with weakness, which is totally fallacious. In fact, weak people resort to war and violence. It is the quality of strong persons to be tolerant and non-violent. Gandhiji gave a broad meaning of non-violence. For him non¬violence meant not just refraining from causing physical harm but also includes the thought of causing physical or mental harm to anybody. It is the task of only the strong person. A weak person cannot have this quality.

Question 12.
How Gandhiji experimented with non-violence?
Answer:
Many people think it is the violence which can bring the just ends, but it is again fallacious. The ultimate result will come only thing non-violence which is proved by Gandhi’s experiments in South Africa and India. Gandhiji made India independent only through non-violence. Violence can bring results for short devotion but .ultimate just end is brought only through the non-violence.

Question 13.
How the state is related to peace?
Answer:
In today’s world, there are sovereign nation-states which are inspired with narrow nationalism. Each modem nation-state is organised on the basis of nationality Every nation-state behaves like a human being. As the behaviour of men is shaped by his personal interest in a similar way the behaviour of a nation is shaped on the basis of his national interest. Since the national interests of the states, a leads to differences leading to clashes and conflicts which ultimately poses threat to peace at the individual level as well as collective level. To protect its interest every state uses the force. Each state has police and army as the instrument of coercion.

Question 14.
What do you mean by Pacifism?
Answer:
Pacifism denotes thinking which opposes war and violence for settling differences and disputes. It prefers to settle the disputes through discussion, negotiation and dialogue and not through violence or war. Pacifism is a principle which is based on pragmatism. The supporters of Pacifism believe that use of force and resort to war is morally wrong. They prefer peaceful means like the negotiation for settling the disputes than the war which, they say will never just end.

Question 15.
What are the main challenges to peace in the modern world?
Answer:
Followings are the main challenges for peace

  1. Intolerance
  2. Materialism
  3. Increasing consumerism.
  4. Terrorism
  5. The criminalisation of society and polity.

Peace Important Extra Questions Short Answer Type

Question 1.
Define the concept of peace.
Answer:
There has a number of persons as champions of peace. Many saints, philosophers and statesman have also championed the cause of peace. Although there have also been many philosophers, warriors and army Generals who have glorified war and did not value peace because they believed that only conflict could facilitate the growth of civilisation. Several thinkers have condemned peace and commended violence and conflict as essential vehicles of individual heroism and social vitality.

Peace is the situation of no war, no tension and no conflict of any kind. Peace is a state of mind. The essential elements of peace are justice, tolerance, truth and compassion. Peace is something eternal and pious ultimate happiness of mankind lies in the peace. There have been ardent supporters of peace such as M.K. Gandhi, Gautam Buddha etc. who denounced war and violence.

Question 2.
Name the factors which have been threatening international peace.
Answer:
Peace at an individual level, national level and international level has been facing threats from different sides. Nation-states have been engaged in conflicts, tensions and wars like the ordinary human beings over petty issues. These issues have been posing threats to international peace.

The post-war period of the world has been marked by intense rivalry’ between two superpowers i.e. Capitalist USA and Communist USSR. There are have been a number of wars and tensions in different parts of the world which threatened international peace. Many wars on the planet have caused the destruction of human life and property.

Question 3.
What do you mean by structural violence? Explain with example.
Answer:
Violence occurs in society at various levels in different forms. It occurs at the individual level, collective level, in an unorganised way and also organised way. An organised form of violence is called structural violence. Traditional caste system treated certain groups of people of a particular caste as untouchables, who were subjected to collective humiliation and exploitation and deprivation. Only after independence, Art. 17 of Indian Constitution abolished it but, still, we have scars of this evil till today.

Similarly, women have been treated badly in a male-dominated society. They were considered inferior and incapable and unworthy of doing any great thing. They have also b.een subjected to hard atrocities.

Another example of structural violence in colonialism which prevailed for a long time in different parts of the world. Racism and communalism were also systems of structural violence.

Question 4.
Discuss racism as structural violence.
Answer:
Racism prevailed as the system Of oppression of an entire racial group or community in South Africa and Negro slavery in the United States of America and the slaughter of Jews in Germany. Apartheid (a policy of racial discrimination) was followed in South Africa by the white-controlled government which treated the black people of the country as second class citizens. Such structural violence continues from one generation to another generation. Continuous exploitation creates grievances that persist over generations and are provoked on slightest incident or remark.

Question 5.
How the violence can be eliminated?
Answer:
It is said that war and tensions lie in the mind of man. Violence is the expression and manifestation of tensions of the human mind. Therefore if the violence is to be eliminated i.e. removed that there should be peace in mind.

The Constitution of United Nation Educational Scientific and Cultural Organisation rightly observes as “Since wars begin in the minds of men, it is in the minds of men that the defences of peace must be constructed.” At the same time, it is also the fact that mindsets are shaped by the socio, economic and political environment of the society. Therefore to eliminate tension and violence, the healthy and positive environment has to be built up. Violence is not the product of individual psyche but is the result of group and collective psyche of the society which also needs reform for peace.

Question 6.
Can violence ever promote peace?
Answer:
Many people think that by mean of violence we can get just ends and also we can establish peace. Many people consider violence for the establishment and promotion of peace. The state is created only on this thesis. Hobbes, Locke and Rousseau have created state to get rid of violence at a natural stage and to establish peace in which life and liberty of the people could be protected. It is argued that tyrants and oppressors can be checked and removed only by force and by using violence. In different parts of the world imperialism and colonialism have been removed by violent Liberation movements. Different terrorist organisations and militants work on this thesis that just cause of self-rule can be achieved only by violent movements.

Question 7.
Discuss the role of the state in establishing and maintaining peace.
Answer:
It is often said that the division of the world into a large number of independent sovereign states is a big obstacle as the hindrance in realising the peace. The state is sovereign and independent identity. It has its own interest and it pursues it. The behaviour and actions of the state revolve around the national interest of the state while for the pursuit of large peace, it is necessary that one should think of larger humanity.

States make a distinction between its citizens and the regions. For the protection of the interests of its citizens, they may harm the interests of the citizens of other states which will naturally lead to clash and conflict in which the cause of peace would suffer. In this way state itself is responsible for the harming the cause of peace while initially the state cause into existence for the building of peace. The state has become an instrument of coercion and force. Peace needs a congenial atmosphere, balanced behaviour of the people for which there should be a peaceful environment.

Question 8.
Explain the approaches to the pursuit of peace.
Answer:
There are mainly three approaches to the realisation of peace. The first approach attaches maximum importance to the states, respect their sovereignty and treats competition among them as a fact of life. It aims at the management of the competition among various people.

The second approach grants the deep-rooted nature of state rivalry. It stresses the positive presence and possible inter-dependence. It underscores the growing social and economic co-operation.

The third approach considers the system to be a passing phase of human history. It envisages the emergence of supranational order and sees the fastening of the community as the surest guarantee of peace.

Question 9.
Explain the philosophy of Pacifism.
Answer:
Pacifism is thinking which is opposed to war-violence as a means of settlement of disputes. It advocates the settlement of disputes by negotiations and other such peaceful methods. Pacifism is a philosophy which denounces war because it considers any form of war as immoral. Pragmatic pacifism does not adhere to such an absolute principle but holds that there are bitter ways of solving the disputes. They say that war and violence have an only a negative impact and no any positive aspect. It is quite fallacious that wars settle the disputes and bring desired ends. In general, other pacifists follow the principle of non-violence. It is therefore Pacifism follows peace and non-violence and make every effort to establish and maintain peace in the world.

Question 10.
Explain various challenges to peace in the present-day world.
Answer:
Today’s world is increasingly becoming more and more violent in which peace is the ultimate target. Peace is diminishing at the individual level, collective level, at the national level and international level. Problems like pollution, terrorism have made human behaviour vulnerable and unbalanced which results in frequent violent events. Followings are main factors which have provoked violence and wars:

  1. Increasing population
  2. Increasing materialism
  3. Increasing consumerism
  4. Lust for power
  5. Lust for money
  6. Increasing means of communication
  7. Prolonging of differences on a certain national and international issue.
  8. Religious fundamentalism.

Question 11.
Discuss the role of the UN in establishing peace.
Answer:
UNO came into existence on 24th October 1945 after the end of the Second World War which played havoc with mankind, particularly When an atom bomb was used in Japan. Various wars of regional levels and First World War and the Second World War changed the international environment with the violence and tensions.

UNO in its charter made the foremost objective to establish the International Peace and Security Council took this sole responsibility to establish and maintain international peace. UNO through its several specialised agencies tried to transform international security through mutual co-operation. It is through the mutual co-operation, that UN has made effort to built understanding among the countries of the world so as to make the congenial atmosphere for the promotion of international peace. It is to the credit of UN that it has been able to check the out-break of Third World War and the contending parties to the disputes have agreed to come to the negotiation with the various types of efforts and decisions, the tension in international relations is reduced.

Question 12.
How terrorism has disturbed the peace?
Answer:
Terrorism is a menace which is disturbing the human psychology in a big way. The terrorism porter is emerging in the behaviour of man as well as in the behaviour of states. The rise of terrorism is partly a response to the self-serving conduct of the aggressive states. In today’s world, it is posing a maximum threat to peace. Religious fanatism and fundamentalism are the big instruments of terrorism which are found in many parts of the world. The human community has suffered a lot due to terrorist attack in different parts of the world.

Question 13.
Can war be justified to achieve peace?
Answer:
At different quarters it is thought that to achieve the just ends, the war is a desirable weapon or a method. In the history of the world, a large number of wars have been fought on the basis of the thesis. But history bears the proof that no war has ever been produced the desired result except the destruction and tensions. Iraq and Iran fought for eleven years together for their viewpoints but no state could get anything except loss of face and destruction. Iraq invaded Kuwait and occupied its territory claiming to be it’s own but ultimately Iraq had to withdraw from Kuwait in the wake of the Gulf war in 1990 which was masterminded by the USA. Peace can only be achieved only by love, compassion, non-violence and tolerance.

Question 14.
Discuss the relevance of Gandhian principles for peace.
Answer:
M.K. Gandhi was one of the great champions of the peace who advocated the need for establishing peace for the welfare of the whole of mankind. In preaching the peace, he was internationalist because he thought of whole mankind. He experimented with his concept of truth non-violence while giving the generalisation.

Gandhian philosophy of satyagraha and non-violence is more relevant and needed than ever before because today the whole world is in the grip of materialism and is threatened by the evils like terrorism and fanaticism. Gandhi said that truth is the ultimate reality which must be pursued by the man at every cost. It will only bring ultimate peace and inner happiness. Tolerance is a must for peace and non-violence because intolerance will lead to tensions and wars. Gandhi said that to be non¬violent does not mean weakness. In fact, it is the strengths of a man if he is tolerant.

Question 15.
How and why the war had been glorified by different persons?
Answer:
Whereas the peace has been cherished by different philosophers, saints and thinkers, it is the war which has also been glorified at different levels by different thinkers, philosophers and the rulers. George Bernard Shah glorified war and had said that war is necessary for the sustenance of patriotism and keeping the army men and arms active.

The 19th-century German philosopher Friedrich Nietzsche was one of those who glorified war. Nietzsche did not value peace because he believed that only conflict brings dynamism and growth of civilisation. Similarly, many other thinkers have also glorified and condemned peace. They were of the view that peace brings inaction and war brings action. Nationalist thinkers and warriors also command war:

Question 16.
What have been the main reasons for war in the world?
Answer:
If we look and examine the history of various disputes and the wars which took place in different parts of the world we find the following factors were largely responsible for war and tensions.

  1. Racism
  2. Extreme Nationalism.
  3. Greed of rulers
  4. Colonialism
  5. Imperialism
  6. Religious fanatism
  7. Religious fundamentalism
  8. False pride
  9. Non-availability of the sources
  10. Internationalism.

Peace Important Extra Questions Long Answer Type

Question 1.
Define the concept of peace, its forms and methods by which peace can be achieved.
Answer:
Peace has ever been an important concept as many other concepts like democracy and secularism. Peace has been the ultimate object of every system. Peace is not just the end of war and tension it is something more. A just and lasting peace can be attained only by articulating and removing the hidden grievances and causes of conflict through a process of dialogue and understanding. The peace can also be understood as the absence of violent conflict of all kinds including war riot, massacre and assassination. Peace in the above definitions is partly true.

Peace is related not to just the physical world it is also related to spiritual and psychological world. It is the outcome of mental saturation and mental satisfaction. Peace is concerned more with the inner behaviour of man. Peace is related to the balanced relationship of the socio-economic environment of the society. If there are any kind of imbalances in society, peace will be elusive. Violence is certainly the negation of peace which has been in existence in the society in different forms.

In Indian society, it is found in caste hierarchy which was based on caste subordinations leading to tensions and violence among different castes.

Women as a social group had been considered inferior and were subjected to humiliation leading to tensions in social relations.

At political level imperialism, colonialism exploited the political communities leading to violent revolts. In such cases, peace has been the victim.

Social Responsibilities of Business and Business Ethics Class 11 Important Extra Questions Business Studies Chapter 6

Here we are providing Business Studies Class 11 Important Extra Questions and Answers Chapter 6 Social Responsibilities of Business and Business Ethics. Business Studies Class 11 Important Questions with Answers are the best resource for students which helps in class 11 board exams.

Class 11 Business Studies Chapter 6 Important Extra Questions Social Responsibilities of Business and Business Ethics

Social Responsibilities of Business and Business Ethics Important Extra Questions Short Answer Type

Question 1.
Explain in brief the concept of social responsibility of business.
Answer:
Concept of Social Responsibility:
Meaning and Rationale: Business in today’s world is recognized and accepted as a social and economic activity of society. The business is carried on within the societal norms in order to satisfy the needs of the society. All the factors of production i.e. Men, machines, materials, money, and equipment are supplied by the society to the business as such business owes its existence to the society.

The society originates, sustains, and manages to conduct its affairs in the. the interest of society. In other words, social responsibility is the obligation of the business towards different groups of society; in addition to its profit earning.

  • “Social responsibility is the personal obligation of everyone, as he acts for his owner’s interests to assure that right and legitimate interests of all others do not impinge.”-Knootz O’Donnel
  • “In the real sense, the assumption of social responsibilities implies recognition and understanding of the aspiration of society and determination to contribute to its achievement”-George A. Steiner

The business has its obligation to pay a reasonable return to its owners, to pay interest at competitive rates to investors, to pay reasonable remuneration to employees, supply goods to customers at reasonable rates, and to conduct the affairs of the business in accordance with social commitment and values.

Question 2.
“Customers are the foundation of the business.” Explain the statement in the light of the social consciousness of consumers.
Answer:
Peter F. Drucker, the management Guru, rightly states that the customers are the foundation stone of any business. The ultimate objective of any business to gain maximum profit can only be achieved by providing the right goods and services to consumers and try to provide maximum satisfaction to consumers. In the opinion of Butler, “The customers hold a topmost place in the organization hierarchy.

It is, therefore, necessary that businesses should avoid adulteration, substandard product, defective measurement or weights, deceptive and advertisements, and omission from services and courtesy. Business premises is for customers not the customers for the business.

The business has the following obligation towards customers:
1. Need-based production: Business should produce only those goods which satisfy the expectation and aspirations of the customers.

2. Supplying goods at a reasonable price: Every business aims at producing goods at the minimum cost. It should supply goods to customers at a reasonable price. The price of the commodity must be reasonable and competitive.

3. Appropriate distribution: The producers and manufacturers should make their goods and services at the appropriate places and times, so that customers may not face more difficulties in acquiring them. The increase in the number of middlemen will increase the price. The channels of distribution must be sufficient to maintain the smooth supply of goods to customers. The importance of customers in business must be recognized and accepted by every firm.

Question 3.
Discuss the responsibilities of business towards employees or workers.
Answer:
Workers are the major force in any organization. They do not only work as a factor of production but also helps in working the other factors. It is rightly said that the owner of a business simply invests capital in the business, but employees/workers invested their entire life for the benefit of the organization. Therefore, it is the prime duty of the business to fulfill its humane, social, and business obligations.

The responsibility of business towards employees or workers may be discussed as under:
1. Fair wages: it is the social, moral humane, and business responsibility of every firm to pay a reasonable amount of wages to employees in order to live a respectable life in society.

2. Security of the job: Workers must feel part and parcel of the organization and treat themselves as a permanent assist in the business. He should contribute his best without fear of being removed and retrenched.

3. Creating a congenial atmosphere of work- Working conditions must be congenial to work. The place of work should be clean, properly ventilated, and free from dirt and suffocation. There should be proper arrangement of safety measures to avoid accidents.

4. Scientific selection and training of workers: The selection and appointment of workers should be based on testing. Employees should be selected strictly as per the requirements of the works. In other words, the right workers should be appointed for the right jobs.

Late Prime Minister Pandit Jawahar Lai Nehru rightly conclude that
“In the present economic and industrial structure, the relationship between workers and customers should not be that of owner and workers but workers should be treated as partner and colleague and in this vim; there can be the peaceful solution of all the economic problems.”

Question 4.
Explain the concept of ‘Human Rights’. Also mention cases for Human Rights.
Answer:
Concept of Human Rights: Human rights provide equality to individuals in their interests. Human rights basically meant to provide a basis for justifying one’s action and provide protection and assistance. Human rights lay stress on the concept of humanity. All big business organizations should follow and promote human rights.

Cases for Human Rights- Human rights have received high priority in our society’. In order to get these rights, many movements have also appeared.

The following statements may be mentioned in favor of human rights in general, in society’ and in particular in business.
1. Protection against Human Injustice- Businessmen generally do not protect government patterns and establish their own social and economic parameters. Human rights come to light when people think that injustice being perpetuated.

2. Provides Benchmarks for Law Land Policies: After independence, certain basic rights became natural which do not act under any law or policy. These rights take precedence over the particular laws and standards created by society.

3. Respecting the Human Values: Some human rights are taken as basic rights. These rights are the overriding significance of human rights over others, human rights, and legal rights and provide entitlements beyond legal jurisdiction should be respected by others.

There is a great difference between human and legal rights. One may have a legal right to do something inhumane but for doing any work of humanity there is no need to have legal rights. Legal rights are derived from the constitution and policies while human rights derived independently. Human rights are based on human norms. Entitlement of human rights can derive from a system of human standards independently of any particular legal system. These rights prohibit todo something inhumane.

United Nation declares the following as Human Rights:

  • The right to work, free choice of employment, good working conditions, right of protection against unemployment.
  • Right of just or favorable remuneration.
  • Right to form and join trade unions.
  • Reasonable limitation of working hours and periodic holidays with pay.

Question 5.
Outline the major Environmental Pollution Control Activities,
Answer:
Major Environmental Pollution Control Activities:

  1. National conservation strategy in 1992, policy statement for environment and development, Policy statement for abatement of pollution 1992, National Forest Policy 1988, and in 1986. Environment (Protection) Act was initiated for pollution control.
  2. Standards related to air, water, and noise levels were formulated by a multi-disciplinary group keeping in view the international standards, technologies, and impact on health and the environment.
  3. Action plans and identification of 17 categories of major polluting industries.
  4. Identification of 24 major polluted areas for pollution control.
  5. Factories were asked to use coal wherein % of smoke will not be more than 34%.
  6. Action plans for 141 polluted rivers started.
  7. In order to reduce the pollution from automobiles, cleaner fuels, low sulfur diesel, and compressed natural gas (CNG) should be used at the manufacturing stage.
  8. Starting of clean technologies for big industries.
  9. FortheclustersofSSI units ‘Common Effluent Plants’ was set up.
  10. The echo mark scheme started to increase the production/consumption of Environment-friendly products.
  11. A zoning atlas was prepared to get environmental informational district level.
  12. Environmental epidemiological studies were initiated in seven critically polluted areas to study the impact of the environment on health.
  13. Financial assistance to initiate pollution control environments and to shift industries in the outer places.
  14. Environment Pollution (Prevention & control) authority was established.
  15. Prohibition of smoking in public places and use of polythene bags.

Question 6.
What are the various factors influencing business ethics?
Answer:
Factors Influencing Business Ethics: The main determinants of business ethics are as follows:
1. Social values- Social forces and pressures exercise considerable influence on business ethics. Often, different groups in society compel businessmen to discontinue unethical practices. Morality, behavior, beliefs emerge from social values, social forces exercise influence on business to observe ethics in business.

2. Legislation: Laws are generally passed to keep a check on unethical practices. They are their use of social pressures. When society considers a practice unethical, it may exercise its influence to get that practice declared illegal. For example, the Monopolies and Restrictive Trade Practices Act has made monopolistic trade practices illegal in India. Prevention of Food Adulteration Act 1976, Drugs and Cosmetics Act 1946, Prevention of Black-marketing and Maintenance of Supply of Essential Commodities Act 1980 has to enact to keep a check on malpractices of business.

3. Government rules and regulations: Government regulations provide guidelines for acceptable practices. For example, the government has made it compulsory for tobacco companies to give the statutory warning “smoking is injurious to health” in the advertisements for cigarettes.

4. Industry norms: In some industries and trades, specific codes of conduct have been laid down. In addition, many organizations have laid down guidelines for regulating the behavior of their employees. Most industries have an ethical climate that governs the code of conduct of the employees. An individual working in the enterprises to observe the code of conduct of the enterprise, and norms established in the industry.

5. Personal Yahies-The personal beliefs of the individuals working in an organization also influence business ethics. However, sometimes there is a conflict between personal moral values and company goals. Generally, employees look to their superiors and tend to adopt their values and actions. The behavior of competitors and associates also influences business ethics. An honest businessman must-keep their personal interest subordinate to die interest of society.

6. Professionafisation: Professional managers normally tend to have higher ethical standards than family managers. Therefore, the growing professionalization of management has exercised a healthy influence on ethics in business. These days professionalism of management has been generating more ethics in the business.

Question 7.
Write short notes on CSR?
Answer:
It is the responsibility of every form of business enterprise to act in a socially desirable manner. But the concept of CSR is corporate social responsibility used particularly with reference to a company. It may be defined as achieving commercial success in ways that honor ethical values and respect people, communities, and the natural environment. It means addressing the legal, ethical, commercial, and other expectations that society has from corporate who should take decisions and actions that fairly balance the claims of stake-holders. CSR is viewed as a comprehensive set of policies, practices, and programs that are integrated into business operations, supply claims, and decision-making processes throughout the company.

Question 8.
List the environmental problems that cause damage to the natural environment?
Answer:
The United Nations has identified eight problems that cause damage to the natural environment.

These are:

  1. Ozone depletion
  2. Global warning
  3. Solid and hazardous wastes
  4. Water pollution
  5. Freshwater quality and quantity
  6. Deforestation
  7. Land degradation
  8. Danger to biological diversity.

Question 9.
Define Corporate Governance?
Answer:
It is originated intheUnitedKingdomforthepurposeofimproved accountable to directors to shareholders, with emphasis on more transparent auditing and increased responsibilities of independent directors and division of roles of chairman and managing directors for safeguarding the interest of the shareholders.

Question 10.
What are the main ground rules of ethics?
Answer:
The following are some rules which all human beings should follow in life.

  1. Be trustworthy
  2. Have respect for other
  3. Own responsibility
  4. Be fair in dealing
  5. Be caring towards well being of others.
  6. Prove to be a good citizen

Question 11.
Write the effects of pollution?
Answer:
Pollution: The injection of harmful substances into the environment is called pollution. It changes the physical, chemical, and biological characteristics of air, land, and water. Pollution harms human life and the life of other species. It also degrades living conditions while wasting or depleting raw material resources. It also damages our historical monuments. It causes risks to the environment, human health and damage to natural and man-made resources.

Question 12.
Give some examples of Business ethics.
Answer:

  1. Giving enough dividends to the shareholder on their investment.
  2. Making goods available of good quality and quantity at a reasonable price.
  3. Making timely payments to the suppliers.
  4. Having healthy competition with the competitors.
  5. Observing the government laws and helping the government by paying taxes in time.
  6. Making employment to the society.
  7. Saving the environment from getting polluted.

Question 13.
“The concept of social responsibility is ultimately in the interest of business community itself.” Do you agree? Explain.
Answer:
“The concept of social responsibility is ultimately in the interest of the business community itself. I agree with this statement because business is an organ of society and it operates in a socio-economic environment. It can justify its existence by fulfilling its obligations to society. No doubt business is an economic institution and it cannot survive without economic performance. But economic results depend upon the goodwill and support of the society’. Business gains support only when it assumes its social responsibilities.

Secularism Class 11 Important Extra Questions Political Science Chapter 8

Here we are providing Class 11 Political Science Important Extra Questions and Answers Chapter 8 Secularism. Political Science Class 11 Important Questions with Answers are the best resource for students which helps in class 11 board exams.

Class 11 Political Science Chapter 8 Important Extra Questions Secularism

Secularism Important Extra Questions Very Short Answer Type

Question 1.
What do you mean by Secularism?
Answer:
Secularism is a high doctrine about the relations of state and religion. There is a Western, American, and Indian view of secularism in the world. The very essential and common feature of secularism is the separation of state and religion. People must be left free in matters of religion to profess and practice any religion of their choice. The state should not have its own religion. It should neither patronize any religion nor it should discriminate against any religion. Western secularism does not allow minority rights, however, the Indian view of religion allows minority rights and positive interference.

Question 2.
How the secularism promotes justice and equality?
Answer:
Secularism is not a simple separation of politics and religion, it is something humane, democratic, and natural concept based on broad philosophy. Its purpose is to remove the inequalities, domination and discrimination, and exploitation in inter and inter-religious and social groups. Secularism is the most powerful philosophy which opposes all forms of undue domination. It seeks to promote justice and equality by humanizing and educating and enlightening society.

Question 3.
What are intra-religious dominations?
Answer:
All the societies are full of not only inter-religious domination and exploitation, but it also takes another conspicuous form which is intra- religious domination in which we find the domination, exploitation, and discrimination within the same religious groups. Secularism is opposed to all such forms of exploitation everywhere. It seeks to promote freedom within religion.

Question 4.
Write some basic features of a secular state.
Answer:
Followings are certain basic features of a secular state:-

  1. The state should not be a theocratic state.
  2. It should not have its own religion.
  3. It should not be governed by any religious man.
  4. There should be a complete separation between state and religion.
  5. The state should not patronize any religion.
  6. The state should not discriminate against any religion.
  7. The state should promote equality and justice within and without the social and religious groups.

Question 5.
What should be the ultimate aim of Secularism?
Answer:
The separation of state and religion is not the only aim of the secular state. In fact it the means to the end (ultimate goal) which is to make a domination-free, exploitation-free, and discrimination-free society based on justice, freedom, and equality. A secular state must be committed to principles and goals which are at least partly derived from non¬religious sources. These ends should include, peace, religious freedom, freedom from religious oppression, discrimination, and exclusions in inter and inter-religious groups.

Question 6.
What type of Secularism was practiced by Kemal Ataturk in Turkey?
Answer:
Kemal Ataturk came to power in Turkey after First World War. He had a different view of secularism which he had practiced there and which was not based on the principled distance between the state and religion. Instead, it supported active involvement and intervention of religion. He was determined to put an end to the institution of Khalifa in public life. Through this type of secularism, he wanted to modernize the society of Turkey.

Question 7.
Write the main features of the Western View of Secularism.
Answer:
It is said that secularism has western and American origin followings are main features of the western view of Secularism:-

  1. Complete separation of state and religion.
  2. No state interference in the matters of religion.
  3. No interference of religion in state affairs.
  4. Independent and exclusive jurisdictions of both i.e. state and religion.
  5. No religious classification of any Public policy.
  6. The state cannot aid any religious institutions.

Question 8.
What is the Indian View of Secularism?
Answer:
Followings are the main features of Indian secularism out of these features, some are similar to that of western view but some are their own.

  1. Separation of state and religion to certain limits.
  2. Inter-religious equality.
  3. Removing the inter and intra-religious dominations and exploitation.
  4. Promotion of minority rights.
  5. Giving aid to religious educational institutions on the basis of equality.
  6. Positive interference in the management of religious affairs.

Question 9.
What are the common features of Western View and Indian View of Secularism?
Answer:
Reading of the Indian Constitution shows that Indian Secularism is fundamentally different from western secularism. Yet it has many things in common.

  1. Separation of state and religion.
  2. No state religion.
  3. No patronage to any religion and no discrimination against any religion.
  4. Promotion of equality and justice.

Question 10.
Write in brief the views of Pt. Jawahar Lai Nehru on Secularism.
Answer:
Nehru had some different and peculiar and personal views on Secularism. His thesis of secularism was equal protection to all religions but not favoring any religion at die expense of others and forbids the state from having its own state religion. He gave the foundation of the Indian view of secularism. For him, secularism also does not mean being irreligious or hostile to any religion. In this way, he was altogether different from Ataturk of Turkey. Nehru also did not support the complete separation between state and religion.

Question 11.
Discuss the impact of Western Liberalism on Indian secularism.
Answer:
The Western Liberal thinking changed the concept of equality prevailing in Indian ancient society. It brought to focus the neglected and marginalized nations of equality. It also helped in promoting inter¬community equality to replace the notion of hierarchy. All this change of notion of equality brought about a change in Indian secularism, which was the result of an interaction between what already existed in a society that had religious diversity and the ideas that came from the west. It resulted in an equal focus on intra-religious and inter-religious domination.

Question 12.
On what grounds the Indian Constitution is criticized?
Answer:
Indian secularism is criticized on the following grounds:

  1. As Anti-religious.
  2. Western import (Idea is taken from the west).
  3. Minorities (It is based on the appeasement of minorities).
  4. Interventionist (Excessive intervention in religious affairs).
  5. Vote bank politics.
  6. Unpracticable.

Question 13.
Do you agree that Indian? Secularism is an interventionist?
Answer:
Indian secularism is criticized by the critics arguing that Indian secularism is interventionist because it is coercive and it interferes excessively with the religious freedom of communities. In fact, Indian secularism allows interference in religious affairs, particularly in the management of religious activities of the communities. But this interference is not negative, it is positive. Indian secularism rejects non-interference in religion but it does not mean that it is excessively interventionist. Indian secularism follows the concept of principled distance.

Question 14.
How far it is correct to say that Indian secularism is anti-religious?
Answer:
It is said that secularism in India is anti-religious. This is fallacious. Indian secularism may be against institutionalized religious domination but it cannot be referred to as the anti-religions. It is also argued that Indian secularism threatens religious identity but the fact is that Indian secularism promotes religious freedom and equality. Therefore it does not threaten religious identity, but on the contrary, it protects the religious identity. If it threatens any form of religious identity, it is dogmatism fundamentalism and the narrowness of any religion.

Question 15.
What is a Minority?
Answer:
Indian secularism is charged with Minoritism which means that Indian secularism advocates minority rights. It is a fact that it is only Indian secularism that advocates minority rights. Minority means to suggest that in Indian secularism gives special privilege to the minorities which is a sort of appeasement of the minorities. This charge also does not hold any ground.

Secularism Important Extra Questions Short Answer Type

Question 1.
Explain the meaning of Secularism.
Answer:
Secularism is a philosophy that is concerned with the relationship between state and religion. Secularists think that religion is a personal matter of a person, the state has nothing to do with religion. Hence it should neither patronize any particular religion nor it should discriminate against any religion. A man should be left free to profess and practice any religion of his or her choice. Secularism does not mean to be irreligious or anti religiousness.

In fact, Secularism means respect to all religions and ways of life because ultimately religion’s a way of life. Secularism is not a narrow philosophy, it is a broader view of life and ways of leading life which have been interpreted differently in different areas in different periods. This is a philosophy based on human and democratic values that seek to achieve equality, justice, and freedom and try to remove discrimination, exploitation, and domination within and outside the social and communal groups.

Question 2.
What do you mean by inter-religious domination and intra-religious domination?
Answer:
Although religion is a positive and pious feeling and viewpoint it has been in grip of a number of negative feelings of hatred, suspiciousness, negative competition, and urges for dominance within religions and without religions i.e. inter-religious groups and intra-religious. There are hatred and communal riots between Hindus and Muslims. They fight for dominance over each other. Similarly, there is a sense of inferiority and superiority within religions.in Hindus, women and SCs have been dominated by male and upper castes. Similar are the examples in Muslims, Christians, and Sikhs.

Question 3.
Explain the essential elements of a secular state.
Answer:
A state-organized on the basis of the doctrine of secularism is called a secular state. Generally, the state with the separation of religion is called the secular state. But the mere separation of state and religion is not a sufficient requirement for a secular state because there are many states which are non-theocratic and continue to have a close alliance with a particular religion.

To be truly secular, a state must not only refuse to be theocratic but also have no formal, legal alliance with any religion. The separation of religion and state is not a sufficient condition but a very much a necessary condition of a secular state. A secular state must be committed to principles and goals which are at least partly derived from non-religious sources. These goals should include peace, religious freedom, and freedom from exploitation, discrimination, and exclusions. It also seeks to remove inter and intra-religious dominance and exploitation. A secular state allows its citizen full freedom in religious and social matters without the interference of the state.

Question 4.
Discuss India as a secular state.
Answer:
India is a secular state as conceived by the architect of modem India, Pt. Jawahar Lai Nehru. India does not have any state religion but supports all the religions. It has the followings features of a secular state:-

  1. There is no state religion in India.
  2. It neither patronizes any religion nor it becomes a hindrance in the way of any religion.
  3. Citizens are free to profess any religion in which they have failed.
  4. Indian secular inter-religious exploitation.
  5. Indian secular state religions affairs.

Question 5.
Discuss the Western model of Secularism.
Answer:
It is said that secularism is of Western and American in origin. Followings are the main features of the Western model of secularism:

  1. Complete separation of religion and state.
  2. No interference in the matters of each other. All the states and religions have their own jurisdiction.
  3. Neither state will interfere in the matters of religion nor religion will interfere in the affairs of the state.
  4. No policy of a state can have an exclusive rationale.
  5. No religious classification can be the basis of any Public policy.
  6. The state cannot aid any religious institution.
  7. It cannot give financial aid to educational institutions run by religious communities.
  8. Citizens are free to profess and practice any religion of their choice.
  9. It emphasizes individual liberty.

Question 6.
What are the merits of secularism?
Answer:
Human history has witnessed a lot of religious fundamentalism and fanaticism which had caused harassment and exploitation of man by man. But the development of science, technology, education, and press and with the development of democratic culture, people have become rational and broadminded. All these developments have brought the idea of secularism. It has the following merits.

  1. It is based on a broader outlook.
  2. It is more democratic.
  3. It is more scientific and based on rationalism.
  4. It is more humane.
  5. It is a more natural philosophy.
  6. It is a more acceptable and popular philosophy.

Question 7.
Who was Kemal Ataturk? What was his view of Secularism?
Answer:
Kemal Ataturk was a Turkish philosopher who propounded his own theory of secularism which was practiced in Turkey. It was altogether different from the western and Indian views of secularism. His secularism was not about the separation of state and religion, instead, it involved active intervention in and suppression of religion.

He came to power in Turkey after First World War. He was determined to put an end to the institution of Khalifa in the public life of Turkey. He wanted to break Turkey all traditional thinking to make it modem one. He set out as an aggressive Wanner to modernize and secularise Turkey. He encouraged western manner thinking and clothing for men and women to change the traditional and orthodox view of life and religion.

Question 8.
Explain the views of Pt. Jawahar Lai Nehru on secularism.
Answer:
Pt. Jawahar Lai Nehru was a great philosopher, humanitarian, and a democrat by heart. He gave his doctrine of secularism on the basis of these referred values. His doctrine of secularism is the part of the Indian Consitution. The main features of Nehru’s doctrine of secularism are as under:-

  1. Separation of state and religion.
  2. No state religion.
  3. Equal protection by the state to all religions.
  4. Neither patronage to any religion nor discrimination against any religion.
  5. All people are free to profess and practice any religion.
  6. Positive interference of the state in religious affairs, particularly the management of religious activities and social reforms.
  7. Complete opposition to communalism.
  8. The purpose was to establish equality, freedom, and justice.

Question 9.
Describe the characteristics of Indian secularism.
Answer:
It is said that Indian secularism is an imitation of western secularism. It is far from the reality that Indian secularism is fundamentally different from western secularism. Indian secularism does not only emphasize the separation of religion and politics but the idea of inter-religious equality is crucial to Indian views of secularism. India has already a culture of inter-religious tolerance. The liberal values of the west brought ideas of inter communitarian equality to replace the notion of hierarchy which prevailed in India for a long time.

Indian secularism is equally opposed to the oppression of weaker sections within a society. Indian secularism allows positive interference in religious matters. It also allows the minority rights. Indian secularism gives citizens to profess and practice any religion of their choice. It is a personal matter of the citizen. Secularism does not mean to be irreligious or to be anti-religious or to oppose the religion. It supports all the religion. ,

Question 10.
On what grounds the Indian secularism is criticized? Are they justified?
Answer:
Indian secularism is criticized in different quarters on the following grounds:-

  1. It is said to be anti-religious.
  2. It is considered as an imitation of western secularism.
  3. It is said to be based on minorities.
  4. It is called interventionist because it allows interference in religious matters.
  5. It is based on vote bank politics.
  6. It is criticized for being impracticable.

A careful study of Indian secularism concludes that all the above charges are far from reality and are not justified for being based on a poor understanding of Indian secularism.

Question 11.
Explain the constitutional position of Indian secularism.
Answer:
India is a secular state. The nature of Indian secularism is based on the doctrine of Pt. Jawahar Lai Nehru about secularism. Following is the constitutional position of Indian features

  • In the Preamble of the Indian Constitution, the word secular is given which declares India as a secular state.
  • Art. 25 of the Indian Constitution states:
    a. Every one will be free to profess and propagate any religion or not to have any religion.
    b. There, will not be a state religion.
  • Art. 26. Every religious community has to right to have educational institutions for the propagation of their religion.
  • Art. 27. Amount inverted on the religious matters cannot be taxed.
  • Art. 28. No religious education can be given in the educational institutions which are run by the state.
  • The state can interfere to manage religious affairs.

Question 12.
What is the ultimate objective of Secularism?
Answer:
Secularism which prevailed in different forms in different parts of the world has broader objectives. It seeks to realize a secular society that is free from all forms of domination in inter and inter-religious communities. It seeks to promote freedom within religions and equality between, as well as within religions. All these goals of secularism can be realized only in a state which is secular, democratic, and free from intolerance, fundamentalism, and fanaticism. There should be a relation among the communities based on mutual understanding and a sense of togetherness. There should not be any religious discrimination.

Question 13.
Differentiate between the Indian view of secularism and the western view of secularism.
Answer:
In the following areas, both views differ from each other. However, they have one similarity that there should not be a state religion in a secular state.

Western view Indian view
1. It supports the complete separation between state and religion. 1. It does not support a complete separation between state and religion.
2. State cannot interfere in each other’s matters as both have their own separate jurisdiction. 2. State can interfere in the affairs of the religion but this interference will not be negative, it will be positive.
3. Here state cannot aid any religious institutions run by the religious communities. 3. Here state can aid the religious institution on the basis of equality.
4. It does not seek to promote minority rights. 4. In India, Constitution has provided for the promotion of minority rights.

Question 14.
What makes Indian secularism distinctive?
Answer:
It is a fact that Indian secularism is based on high philosophy. Its following features make it distinctive

  1. It arose in the context of deep religious diversity.
  2. It is influenced by western modern ideas and nationalism.
  3. India had a culture of inter-religious tolerance for a very long time.
  4. Impact of western modernity and values neglected and marginalized values of equality which helped in replacing the notion of hierarchy.
  5. It is a mixture of Indian religious diversity and the ideas of the west which resulted in the removal of intra-religious and inter-religious dominations.

Question 15.
How far do you agree with the criticism of Indian secularism that it is based on Minoriticism or is pseudo-secularism?
Answer:
A very serious charge against Indian secularism is that it is pseudo-secularism. It is based on minorities i.e. the appeasement of minorities keeping in view the vote banks of minorities. India seems to be divided between genuine secularists and pseudo-secularists. It is true that Indian secularism advocates and supports minority rights but not for any special reason.

India is a plural society where people of different dialects, languages, and religions live together. In this situation some are in majority’ and some are in minority. In this situation, there may be fear of inter religions and intra-religious domination and exploitation. India is a democratic country therefore for the purpose of infusing confidence and a sense of belongingness, minority rights are given. But they are not given any special or privileged position. Therefore the charge of pseudo-secularism is not correct. Indian secularism aims to build a society based on harmonious relations between the majority groups and minorities.

Question 16.
Do you agree that the Indian view of Secularism is interventionist?
Answer:
Although separation of state and religion is the main characteristic of Indian secularism it does not emphasize the complete separation rather it advocates support to all religions. It also allows interference in religious affairs and the management of religious activities. But this interference is positive and not negative. India secularism allows interference for reforms and to establish equality and justice and remove the inter and inter-religious group’s domination and exploitation. Therefore if it is interventions, it is for good reasons.

Secularism Important Extra Questions Long Answer Type

Question 1.
Explain the main features of Western View and Indian View of Secularism critically.
Answer:
It is said that Indian secularism is originated from the west, therefore it has certain similarities with the western view of secularism but it is also very much different from the western view in a number of ways.

Western view of secularism:
Followings are certain features of the western view of secularism:

  1. Complete separation of state and religion.
  2. The state and religion will not interfere in each other’s affairs. Both have their respective jurisdiction.
  3. No religious classification can be the basis of any public policy.
  4. The state cannot aid any religious institutions.
  5. Its conception of equality and liberty is an individualist manner.
  6. There is no scope for minority rights.

The Indian view of secularism:
Followings are the main features of the Indian view of secularism:-

  1. The separation between religion and states but support to all the religions.
  2. Indian secularism allows positive interference in the matter of religious affairs and management of regions’ activities.
  3. A state can aid religious institutions.
  4. A state can give financial assistance to educational institutions run by the states.
  5. Its conception of equality and liberty is community-based.
  6. It allows minority rights.

Indian secularism is criticized on the following bases:-

  1. Anti-religious
  2. Western origin
  3. Minorities
  4. Interventionist
  5. Based on vote bank politics
  6. Impracticable.

Emerging Modes of Business Class 11 Important Extra Questions Business Studies Chapter 5

Here we are providing Business Studies Class 11 Important Extra Questions and Answers Chapter 5 Emerging Modes of Business. Business Studies Class 11 Important Questions with Answers are the best resource for students which helps in class 11 board exams.

Class 11 Business Studies Chapter 5 Important Extra Questions Emerging Modes of Business

Emerging Modes of Business Important Extra Questions Short Answer Type

Question 1.
Define E-commerce and mention its opportunities in the modern business world.
Answer:
Meaning of Electronic Commerce (E-commerce): Internet commerce or E-commerce (electronic-commerce) is on-line electronic technology connected via the internet to assist and enhance a variety of business processes, functions, and systems. Using the Internet for commerce means no physical presence of parties is required.

Electronic commerce is electronic communication among enterprises, including customers, suppliers, business partners, govt, organizations, and financial institutions. It encompasses a broad range of activities like ordering, invoicing, transportation, delivery, and payment can be done electronically.

Definitions of E-commerce: “It is an electronic communication among enterprises, including customers, suppliers, business partners, government organizations and financial institutions. ”- Dr. Janardhan

Thus, we can understand “e-commerce is a general concept covering any business transaction executed electronically between parties such as companies to companies (business-to-business) companies and consumers (business-to-consumer), consumers and consumers, business and the public sector and between consumers and the public sector.

Therefore, we can say that electronic commerce is a generic name for a range of technologies that allows the transfer of business information using electronic means. This would, therefore, include technologies such as Electronic Data Interchange, Universal Product Certification or Article Numbering, geographical positioning System, etc. which allows the exchange of information reducing human intervention to a minimum.

Opportunities of E-commerce: The opportunities of electronic commerce are not just available to large corporations and government departments but many small to medium businesses are also discovering new cost-effective opportunities by using internet-related technology to help their business operations locally or internationally. It makes it possible to work round the clock and around the world. It offers business concerns more cost-efficient and time-efficient.

Moreover, e-commerce offers great opportunities for developing countries. It can help them to enter the prosperous global market place and hence serve to reduce the gap between rich and poor countries.

In today’s business world corporate applications of electronic commerce have focused on computer-to-computer interactions. The output of one computer program becomes the input to another. The system that is used to generate a printed purchase order and mail it to the supplier now sends the purchase order as an electronic data interchange message to a computer at the supplier’s facility. The order is automatically, electronically received, and input by the supplier’s own order processing software and does not have to be keyed in. Payments are similarly automatic with electronic funds transfers (EFT) replacing cheques. It makes no sense to spend money on paper and people when two computers can structure the transaction for a few amount.

E-commerce offers the following opportunities to suppliers and commensurate benefits to consumers:

  • Global choice.
  • Improved competitiveness or quality of services.
  • Personalized products and services.
  • Rapid response to needs.
  • New products and services.

Question 2.
Give the meaning of E-business.
Answer:
Meaning of E-business: E-business (Electronic business) is the conduct of business on the internet. It involves not only buying and selling but also servicing customers and collaborating with business partners. E-business is the future of all business. It encompasses the use of technologies, processes, and management practices that enhance organizational competitiveness through the strategic use of electronic information.

It goes beyond e-commerce by integrating e-commerce tightly with business operations to improve performance, create value and enable new relationships between businessmen and customers. It involves strengthening relationships with customers and suppliers. It is beneficial to check the competition and useful for developing new product ideas and sources. It is more concerned with re-designing business processes.

Question 3.
What are the various uses on benefits of e-commerce?
Answer:
Uses of E-commerce:
1. Customers search: E-commerce helps in finding customers for the product or service by sending them informative e-mails for the product or service. These e-mails help customers to understand the product and decide as to whether it should be purchased or not. It makes use of a search engine to find an appropriate seller’s website.

2. Pre-sales inquiries: Customers can contact the company’s sales executives through the company web-site to have information about the product price, specification, etc. It may select the desired seller from the advertisement or a friend’s recommendations.

3. Information publishing and dissemination Company web¬site helps in providing the latest information about the price, discount, quality of the product, etc. Web-site can be accessed anywhere in the world.

4. Sales of product: Online sales of products can be made on the website. Customers can choose the product from the catalog of photographs and order online. Payment can be made through cheque, draft, or credit card.

5. Advertising product: E-commerce helps to advertise the product through e-mails, website, telemarketing, etc. Customers can be given the latest information on the e-mail or website.

6. Sales promotions: Sales can be increased by online trading on the internet also helps to provide better customer service by receiving their complaints on the e-mail.

7. Purchasing goods: Suppliers can be found on the internet for a particular product or service. Internet search engines like Google, Yahoo help in locating suppliers from all parts of the world.

8. Funds transfer: Internet banking has the facility of funds transfer from one bank to another. The customers are provided secured identity to carry out such transactions on the internet. Payment may be made through credit card or cheque online.

9. After-sale service: Customers can be provided efficient after-sales service by contacting them via the e-mail address. Customers can put the complaints through companies e-mails.

Question 4.
Explain the process of E-business or online transactions.
Answer:
Process of E-commerce Transactions:
1. The cardholder browses items the merchant has for sale: Traditionally, the buyer walks into a bookstore and browse items on the shelves. If he finds something that he likes, he puts it into his shopping cart.

With electronic shopping, the buyer uses his Internet browser to browse items. Arvind Cloth Mills, Surat has for sales on its web pages. If he finds a cloth that interests him, he can add it to his virtual shopping cart. Arvind Cloth Mills Website provides a mechanism for keeping track of the order state, thereby remembering which books the buyer decides to keep in his cart.

2. The cardholder decided which of the items to purchase: With electronic shopping, as well as with traditional shopping, the buyer can remove any or all of the clothing he has placed in his shopping cart. (This functionality is merchant-supplied and is not a part of SET.) He pays only for the ones that he has decided to purchase.

3. The cardholder is presented with an invoice containing the list of items and their price: Traditionally, the buyer takes his clothes to the cash register, and the clerk rings up his purchase and presents him with a total price.

With electronic shopping, the buyer makes his decision to pay via Arvind Cloth Mill’s Website by clicking on a “Pay Now” button or something similar. He is then presented with a sales order electronically, from the cloth mill’s server, or on his computer via his electronic shopping software.

4. The cardholder selects a means of payment: When standing at the checkout counter at the cloth mill, the buyer decides which of the credit cards in his wallet, he wishes to use for his purchase.

Similarly, the buyer may have multiple cards he can choose from to complete his SET transaction.

5. The cardholder gives the merchant order confirmation along with the means of payment: Traditionally, this is as easy as the buyer handing the clerk a credit card.

With SET, the order and payment instructions are digitally signed by the buyer and are then sent to the Arvind Cloth Mill, Surat.

6. The merchant requests authorization for the purchase: With both electronic and traditional shopping, the cloth mill sends an authorization request to its acquire via its payment gateway. The acquire sends this information for processing to the buyer’s issue.

The issue returns with an authorization response. The response includes an indication of whether the authorization request has been approved. In turn, the acquirer responds to Arvind Cloth Mill, Surat with the outcome (acceptance or rejection) of the processing.

7. The merchant delivers goods to the cardholder: The cloth mill’s clerk, in the case of traditional acceptance, give the buyer his clothes and sends him on his way.

Question 5.
What are the various security problems that arise in E-commerce transactions?
Answer:
Security problems related to E-commerce: E-commerce has been facing certain problems that endanger the security and safety of business conditions.

The important problems are discussed below:
1. Hacking: It means unauthorized entry into a website. The hacker can destroy the data on the website which would cause a huge loss to the owner of the website. This would interrupt the business transactions carried out through the website. In several countries, the governments have set cybercrime cells to take action against the crimes committed by the hackers. In recent years, there has been an alarming increase in the cases of hacking.

2. Viruses: There are several viruses that can clean up all the information stored in the computer. The viruses can enter a system through e-mail or disc drive floppies or affected websites. Until the: virus is cleaned out, all operations come to a standstill. In such a situation, the business firm will suffer a loss of revenue. Viruses cause huge loss. of revenue and time to clear the disc drive floppies or websites.

3. Brand-hijacking: The Internet allows the creation of powerful brands in a very short period of time. But brand identity created through radio, T.V., and the press is very costly and time-consuming. The internet brand may overshadow the existing brands causing huge loss to their owners. For instance, Amazon became the leading bookseller brand within a period of about two years and it offered tough challenges to the established names such as Barnes and Noble. As a safeguard, each company should establish its brands in cyberspace without further delay.

4. Impersonation: Hackers may pretend to be customers themselves. They thus make use of stolen credit cards of real customers.

5. Fraudulent trading: A business enterprise operating a website may indulge in fraudulent practices. It may create a fake website, take away money from customers and not supply the product/service to the customer. It may not protect confidential information about customers. Parties to a transaction cannot deny their participation later on.

6. Improper registration of domain names: A dishonest person may register a domain name linked to the established brand name or trademark of another firm. For example, he may register vodafone.com. in his name to sell the domain name to the rightful owner at a price.

Question 6.
What is the meaning of the term ‘outsourcing’?
Answer:
Outsourcing (Introduction): In today’s eta, businesses are more focused on cost control measures as it is not easy to increase prices if a market share has to be retained or improved. Competitive advantage can be realized only when businesses become more efficient in their operations and reduce the cost of inefficiencies. In such a case specialization plays an important role.

During the 1990s, a new trend could be seen wherein the repetitive nature of work was being transferred to outside agencies so that efficiency in day-to-day operations can be achieved and businesses remain focussed on their core competencies. This phenomenon of hiring outside firms for services is known as outsourcing of services. It is a trend that is radically reshipping business. It relates to long term contracting out the non-core activities to third parties. With a view to benefitting from their experience, expertise, efficiency, and investment.

Meaning of Outsourcing Services: Outsourcing is the latest trend in today’s business world, going outside your own organization to obtain specialized services of various kinds. It is called Business Process Outsourcing (BPO), which essentially means getting tasks accomplished through an outside agency. Every outsourcing relationship involves processes and procedures. These are often standardized but it is important that there is some flexibility to ensure that they can evolve with the relationship.

Question 7.
Mention the various characteristics or features of outsourcing.
Answer:
1. Getting work done through outsiders: Outsourcing is the process of getting work done from specialist individuals outside the business. For example, an enterprise hires other agencies for repairing its machines rather than repair them by its staff.

2. Quality job: Through using outsourcing sources people get good quality. For example, a specialized faculty of a particular subject can teach the same subject better than general faculty.

3. Improvement in the number of products: Outsourcing helps to improve the number of products. A specialized courier agency or a transport company serves business better than its own drivers of trucks/ vehicles.

4. Expansion of business: The outsourcing process links growing businesses to a range of state-of-the-art services and, resources using outsourcing resources any business can expand as per its need. For example, using an advertising agency as a source of outsourcing a company/business can inform the features and quality of the product to people in an easy way.

5. Better standard of living- Outsourcing provided the best services to the people. In the modern world, every individual wants to avail of the maximum opportunity of his precious time. That’s why, he wants to use specialists for his different needs, which provides a better standard of living.

Question 8.
Mention in brief the various types of outsourcing.
Answer:
Type of outsourcing: Any non-core business process can be outsourced. Outsourcing of some of the popular services are given below:
1. Financial Services: Big companies require to obtain several financial services from outside or external sources. Every business firm requires some sort of financial service, e.g. payroll accounting services, merchant banking, underwriting, etc.

Manufacturing and commercial firms except for very large size find it more convenient and economical to depend upon outside financial agencies for the various financial services. For example, Reliance Industries may outsource financial services from HSBC Bank for issuing ADRs/GDRs in foreign capital markets.

2. Advertising: All big firms outsource the assignment of advertising their products and services to specialized handling. Business firms generally depend upon advertising agencies for designing, developing, and disseminating advertisements for their products and services. Some big advertisers such as Coca Cola, Pepsi, Hindustan Lever, and others have agreements with advertising agencies. Under the agreement, the advertising agency undertakes to provide all services concerning advertising in return for a fee. Outsourcing also helps to make use of the services of experts of ad-agency.

3. Courier Service: Big business firms have to send letters, parcels, etc. in large numbers. They make use of sendees offered by courier firms such as DTH, Overnight Express, and others. These courier firms collect all articles to be deposited from the offices of client firms and dispatch them to their destinations. Courier service is faster and more efficient, cheaper, quite reliable, and personalized service to clients.

4. Customer Support Service (CSS): ‘Customer is like a precious ornament which should not be stolen by anyone else.’ Several services are offered to customers to support the purchase and use of products. Companies are realizing that customer service is an excellent way to achieves competitive advantage in a highly competitive market.

Customers need home delivery service, repair, and maintenance of consumer durables, information, and counseling about alternate brands that best meet their needs. Business firms can concentrate on the sales function by outsourcing customer support services. For example, GE Capital and some other companies have set up call centers in India to provide support services to their customers in different countries.

Question 9.
Give a brief view of various advantages or need of outsourcing in today’s competitive world.
Answer:
Advantages/need of outsourcing: In today’s business world, outsourcing has become a buzz word to achieve specialization in services due to lack of time and unlimited requirements. Everyone wants more satisfaction in less time so that he uses specialists who create requirements of outsourcing of resources.

The principal needs of outsourcing are classified as follows:
1. Concentration on Core Competency: BPO provides an opportunity to the business to concentrate on core areas of its strength and competence. It can content rule on advertising, financial, courier, and customer services.

2. Enhancement of Business: As we know that every businessman wants more and more profit and personal development. For that, he should try to become distinguished from others by providing standard and specialized quality of a product. In order to achieve this objective, he should use outsourcing resources to make his dreams true.

3. Specialisation: Through outsourcing specialization in business operations can be achieved. This reduces cost and improves the quality of business operations.

4. Obtaining Better Quality of Product Services: The standard of living, of people, is improving day by day. Every individual wants to use the standard quality of goods and services. For this purpose they use outsourcing resources, so that standard quality of goods services could be produced.

5. Big Savings: Outsourcing agencies are competent enough in their services and perform the job at a lesser cost. It results in savings.

6. Avoidance of Labour Problems: Outsourcing requires a lesser number of persons to be employed by the business, so labor problems are also reduced.

7. Lowering the chances of Risk: Outsourcing to a service provider who is a specialist in the field adds to the success of the business firm and lowers the chances of risks.

Emerging Modes of Business Important Extra Questions Long Answer Type

Question 1.
What are the various requirements for the successful implementation of E-commerce?
Answer:
Requirements for successful implementation of E-commerce: Resources Required for E-commerce: E-commerce has tremendous potential in the Indian economy. But it requires the following resources for its successful implementation:
1. Computer Hardware: The first requirement for the implementation of e-commerce is the availability of adequate computer hardware. The computer system with the business must have adequate speed, memory, and nodes to handle the expected volume of business transactions. It should provide the necessary internet service provider and application service provides.

2. Qualified Personnel: It is essential for the business firm to have technically qualified personnel to make use of computer networks and the internet for business transactions. In particular, the staff must be trained to handle trade inquiries, obtain and process orders from customers and ensure prompt deliveries of goods.

3. Well-designed Website: Customers will gain access to the company’s website through their computers linked to any network. To carry out e-commerce, the business enterprise must develop a comprehensive website. This will facilitate communication with customers, other business enterprises, and even government departments. The information should contain business history, nature, and types of products offered along with pictures, graphs, and other technical details. The website may also make available price lists or catalogs of various products.

4. Telecommunication Facilities: E-commerce can be carried out by a business enterprise if it has access to an effective telecommunication system. E-commerce will succeed only if adequate telephone lines and internet facilities are available and there is no interruption in these services.

For the spread of e-commerce among lower-income groups and to the rural areas, it is essential that telecommunication and internet services are adequately available at a reasonable cost, otherwise, e-commerce will remain confined to rich people. The cost of hardware and the price of using the internet must be within the reach of low and middle-income groups.

5. Reliable Payment System: For the success of e-commerce, the business enterprise needs to develop an efficient system of receiving payments for the goods sold. Electronic transfer of money from the customers should be facilitated. Similarly, there should be a facility for electronic transfer of any refund to the customers. For this, the business enterprise needs to make arrangements with commercial banks and credit card agencies. An inbuilt system of refunds, in case of the excess amount, is received should be created.

Question 2.
Write a note on the security of business transactions through E-commerce.
Answer:
Security of business transactions in E-commerce: E-commerce can succeed when both buyers and sellers trust the system of exchange. This trust can be generated by ensuring the security of e-commerce transactions.

There are five essential elements for ensuring secure transactions over the Internet.

  1. Authenticity
  2. Integrity
  3. Confidentiality
  4. Non-reputability of origin
  5. Non-reputability of receipt
  • Authority: Authority means enabling each party in a transaction to ascertain with certainty the identity of the other party.
  • Integrity: Integrity implies honesty in dealings.
  • Confidentiality: Confidentiality means the privacy of the transactions.
  • Non-reputability of origin: Non-reputability of origin refers to ensuring that the parties to a transaction cannot subsequently deny their participation.
  • Non-reputability of receipt: Non-reputability of receipt implies that the seller cannot subsequently deny the payment received.

These mechanisms depend upon certification by a trusted third party (such as a government body). Therefore, global electronic commerce requires the establishment of a global certification system.

In order to ensure the security and safety of e-transactions, the following points need to be observed and followed:

  • Authorization and Authentication of the documents precisely having no possibility of fraud.
  • Confidentiality of the message, that the contents of the message may not be scanned by an unauthorized party.
  • The integrity of the message and originality of the contents should be observed.

Secure transactions require that all parties use up-to-date software and hardware. Since consumers are the weakest link, organizations offering e-commerce must take special care to alert them about possible security problems. For example, www.buyandhold.com issued a security alert because some older versions of Netscape Communicator and Microsoft Internet Explorer included a root CA certificate which was set to expire at the end of 1999. The alert encouraged visitors to upgrade their browsers.

It is necessary to make customers feel comfortable about ordering online.

Question 3.
What are the various opportunities or benefits of E-commerce in the modern business world?
Answer:
Opportunities and benefits of E-commerce: E-commerce makes it possible to work round the clock and around the world. It enables companies to be more efficient and flexible in their internal operations, to work more closely with their suppliers, and to be more responsive to the needs and expectations of their customers.

It allows companies to select the best suppliers regardless of their geographical location and to sell to a global market. It also offers business concerns more cost-efficient and time-efficient means of working with customers, suppliers, and development partners.

E-commerce offers several opportunities to suppliers and commensurate benefits to customers. These are given below:
1. Global reach/Global choice or access: The Internet is true without boundaries. The boundaries of electronic commerce are not defined by geography or by national borders, but rather by the coverage of computer networks. Since the most important networks are global presence and to conduct business worldwide.

Access to the global marketplace helps to improve the operations of business firms. It would not be an exaggeration to say that in the absence of the internet, Globalisation would be restricted in scope and speed.

The corresponding customer benefit is a global choice: a customer can select from all potential suppliers of a required product or service, regardless of their geographical location.

Supplier opportunity Customer benefit
the global presence or reach improved competitiveness mass customization and “customization” shorten or eradicate supply chains substantial cost savings novel business opportunities having the global choice quality of service personalized products and services rapid response to needs substantial price reduction new products and services

2. Improved competitiveness/Quality of Service: Electronic commerce enables suppliers to improve competitiveness by becoming “closer to the customer”. As a simple example, many companies are employing electronic commerce technology to offer improved levels of pre-and post-sales support, with increased levels of product information, guidance on product use, and rapid response to customer inquiries.

The corresponding customer benefit is improved quality of service. Internet is responsible for the popularity of the phrase “networked individuals and firms are more efficient than networked individuals and firms.”

3. Mass Customisation/Personalised Products and Services: With electronic interaction, suppliers are able to gather detailed information on the needs of each individual customer and automatically tailor products and services to those individual needs. This results in customized products comparable to those offered by specialized suppliers, but at mass-market prices.

One simple example is an online magazine that is tailored for the individual reader on each access to emphasize articles likely to be of interest and exclude articles that have already been read. Imagine a restaurant that does not have any requirements of physical space, but an online menu can be presented.

Customers visit your websites, decides the menu, place the order, food is delivered and the payment collected on an electronic clearing system.

4. Shorten or Eradicate Supply Chains/Rapid Response to Needs: E-business is truly a business as enabled and enhanced by electronics and offers the advantages of accessing anything anywhere, anytime. Electronic commerce often allows traditional supply chains to be shortened dramatically. There are many established examples where goods are shipped directly from the manufacturer to the end consumer by-passing the traditional staging posts of wholesaler’s warehouse, retailer’s warehouse, and retail outlet.

(Typically, the contribution of electronic commerce is not in making such direct distribution feasible since it could also be achieved using paper catalogs and telephone or postal ordering: but rather in making it practical in terms of both cost and time delays.)

The extreme example arises in the case of products and services that can be delivered electronically when the supply chain can be eradicated entirely. This has massive implications for the entertainment industries (film, video, music, magazines, newspapers), for the information and “detainment” industries (including all forms of publishing), and for companies concerned with the development and distribution of computer software.

The corresponding customer benefit is the ability to rapidly obtain the precise product that is required, without being limited to those currently in stock at local suppliers.

5. Substantial Savings/Substantial Price Reductions: One of the major contributions of electronic commerce is a reduction in transaction costs. While the cost of a business transaction that entails human interaction might be measured in dollars, the cost of conduction a similar transaction electronically might be a few cents or less. Hence any business process involving “routine” interactions between people offers the potential for substantial cost savings, which can, in turn, be transacted into substantial price reductions for customers.

Question 4.
What is the outsourcing of services? Mention in detail the various services which may be outsourced by a business unit.
Answer:
Concept of outsourcing of services: The term ‘outsource’ means getting something from external sources rather than creating it within the organization. For instance, a firm may outsource the ‘advertising activity’ to an advertising agency instead of setting up its own advertising department. Similarly, other services may also be outsourced for better quality service and also to achieve economy.

This practice is called Business Process Outsourcing (BPO). The basic feature of BPO is that’ companies hire out.on contracting those services or tasks which are performed on regular basis’. For example, a business enterprise may outsource the employment of personnel, training of personnel, and payroll accounting to a specialized service provider

Traditionally, industrial organizations have faced the problem of ‘Make or Buy’ certain components or parts. They can compare the costs of both in-house production and outsourcing the components while taking such decisions.

Types of Services: There are several types of services that are offered by business firms to their customers or clients.

The significant services include the following:
(a) Financial services such as underwriting, issue management.
(by Advertising service.
(c) Customer support services.
(d) Courier service.
(e) Communication services such as STD, ISD, local phone, etc.
(f) Hotel services.
(g) Airlines services.
(h) Banking services.
(l) Health services.

Outsourcing of Financial Services: Big companies require to obtain several financial services from outside or external sources (i.e, outsourcing) because they don’t possess the infrastructure and capability to perform such services.

These include the following:
1. Underwing to Securities Most of the companies get their new issues of shares underwritten with the underwriters. Underwriters give a guarantee to the company that entire new issues will be sold to the public. In case, some securities are not subscribed to by the public, the underwriters undertake to purchase them. Underwriters get a small commission per share for their services.

2. Merchant Banking: Merchant bankers sponsor capital issues of companies. They help the company in drafting the prospectus, and application forms, compliance with legal formalities, the appointment of the registrar to deal with the share applications and transfers, a listing of securities, the arrangement of underwriters, selection of brokers, publicity, etc.

3. Registration of shares: Specialized firm known as registrar to issues deal with share application, registration of share, transfers, etc.

4. Depositories: They hold shares of shareholders of different companies in dematerialized (intangible or paperless) form.

Advertising Services: Business firms depend upon advertising agencies for designing, developing, and disseminating advertisements for their products and services. All big firms need to advertise their goods and services which require specialized handling. They can outsource this assignment to advertising (ad) agencies. Ad agencies help their clients in preparing the advertisement budget, advertisement program, and advertisement copy.

They prepare ad films, documentaries, and other promotional material on behalf of their clients. They also help the clients in choosing the ad media such as TV, radio, newspaper, hoarding, etc.

By entrusting the advertisement assignment to an ad-agency, the business firm is saved of the botheration and cost of employing a copywriter, film-maker, cameraman, and others. This turns out to be economical for the firm in the long-run. Outsourcing also helps to make use of the services of experts of the ad-agency.

Courier Service: Courier service involved delivery of letters, documents, and small parcels from one place to another through private operators known as couriers or courier companies.

The reasons for the popularity of courier services are as under:

  1. Courier service is faster and more efficient as compared to the postal service.
  2. Courier service is cheaper as compared to the postal services.
  3. Courier service is quite reliable.
  4. It provides personalized services to the clients.
  5. There is no long queue at the courier companies as is the case with the post offices.
  6. Courier companies provide the facility collecting the letters from the premises of their regular clients for onward delivery.

Types of Courier Services: Courier forms such as DTH, Overnite Express, and others collect all articles and dispatches to their destinations.

  1. Messenger Service: Under this, letters and documents could be delivered at the local and regional levels generally within a period of 24 hours.
  2. Inter-State Service: This courier service is intended to deliver letters and documents from one state to another within a period of 24 to 48 hours.
  3. Overseas Desk-to-Desk Courier Service: This service offers desk-to-desk document deliveries using air-courier services. Several international couriers with their own fleet of aircraft are operating in this field such as DHL Couriers, Transworld Couriers, Blazeflash Couriers, Express Air, United Parcel Service (UPS), etc.

Customer Support Services: ‘Customer is like a precious ornament which should not be stolen by anyone else.’ The above saying clearly states the importance of customer satisfaction and delight. It is the dissatisfied customer who does not repeat orders with the same seller. He may be dissatisfied because of some problem with the product or because of lack of care in attending to his complaints or inadequate after-sale service.

The term ‘after-sales service’ means responding to a customer with a problem. The problem may relate to ‘product complaints’ during the warranty period and even after that, spare parts, etc. This requires an organized system of efficient customer support services. Customers need home delivery service, repair, and maintenance of consumer durables, information and counseling about brands, and best meet of their needs.

In the present-day competitive environment, every business must take care of the grievances/complaints of the customers and do the needful to please the customers.

There are two options as regards provision of customer support services, namely,

  1. creation of customer support services department within the organization, or
  2. entering into an arrangement with an outside agency to provide after-sales services on behalf of the organization.

Traditionally, business organizations have organized their own customer support services departments. Such a department is manned by trained personnel to attend to the complaints of their customers.

The quality of after-sales services by this department determines the image of the organization. Every organization can’t afford to have a separate customer support service department because of a shortage of funds and inadequate infrastructure.

In such a case, it is advisable to outsource various customer support services from a reputed agency. Such an agency would take operational responsibility for providing customer support services on behalf of the organization against a stipulated amount of payment every year. If the agency or service provider fails to provide efficient and satisfactory services, the organization can cancel the contract with it and engage some other agency.

Generally, the customers of products like consumer durables (Fridge, TV, Air Conditioner, etc.) are widely spread throughout th€ country. They might need repair services during the ‘warranty period’ or even after that. If such services are not easily available, the image of the manufacturer and his brand will go down in the market. In the past, some TV manufacturers in India failed because they could not provide satisfactory after-sales services to their customers.

In such a situation, it is better to outsource such services to some reputed service provider or agency. The manufacturer will be saved from the botheration of engaging staff and creating the infrastructure for providing after-sales services to the customers.

Business Services Class 11 Important Extra Questions Business Studies Chapter 4

Here we are providing Business Studies Class 11 Important Extra Questions and Answers Chapter 4 Business Services. Business Studies Class 11 Important Questions with Answers are the best resource for students which helps in class 11 board exams.

Class 11 Business Studies Chapter 4 Important Extra Questions Business Services

Business Services Important Extra Questions Short Answer Type

Question 1.
Explain the need and benefits of services.
Answer:
The need for Service Sector: Modern trade is the symbol of modem civilization. It has crossed every barrier and reached even the distant and remote parts of the world. It has touched every man. The economic welfare of mankind has been made possible by trade. The physical distribution of goods requires series of interrelated activities, which help in the flow of goods from the producer to the final consumer.

Serv ice sector ensures the smooth supply of goods and services. The service sector consists of a series of interrelated activities concerned with producers to consumers. Service facilities help ensure the supply of the right quantity of the right products to the right place at file right time.

Benefits of Services:
1. Quick and economical services to customers: Efficient service to customers is the only way to have an edge over the competitors. Customer satisfaction can be gained by providing quick, economical, and efficient services to consumers.

2. Minimisation of cost: The distribution costs are part of the price of goods. The costs of transportation, insurance, warehousing, and financing increase the price of the commodities. The efficient and cheap supply of these services minimizes the cost, which is beneficial to consumers.

3. Additional sales volume: The service sector through its efficient transportation, communication, warehousing, and advertising facilities helps increase demand and supply of sales. These services provide knowledge about the commodity and also help in their distribution. Services improve the faith and loyalty of customers.

4. Stabilisation of prices: Transportation helps in the transfer of goods from areas of abundant supply to the areas of scarcity, so the price remains almost the same everywhere. Warehousing adjusts the supply according to demand to avoid violent fluctuations in prices.

5. Removal of time and place barrier- There is a time gap between the production and consumption of the commodities. The warehousing services keep the commodity in their safe custody for the period between production or purchases and sale. This service protects goods from damage and destruction. Modem transport facilities have enabled the movement of commodities from one place to other places.

Question 2.
What are the various types of financial services? Explain in brief.
Answer:
Types of Financial Services: In addition to the traditional financial services of Banking and Insurance the following new financial services have emerged.
1. Merchant Banking: Services of intermediary regarding the issue, management, underwriting corporate restructuring are referred to as merchant banking. They provide growth of the capital market and help in developing an investment climate in the country.

2. Loan Syndication: The approach of borrowers to several banks willing to syndicate a loan, specify ing the amount, and the tenure of the loan is termed as loan syndication.

3. Venture Capital: Subscribing the equity shares of borrowers in return for part of ownership.

4. Factoring: The practice of selling accounts receivable to other companies or agencies for raising funds is termed factoring.

5. Leasing: The legal agreement by which the user of an asset may make payment of lease rent to the owner of the asset in return of use. At the end of the agreement, the lessee takes possession of the assets.

6. Mutual Funds: A company that pools funds from individuals to invest in shares or debentures and in short-term securities.

Question 3.
What are the various types of life insurance policies undertaken by the policyholders?
Answer:
Types of Life Insurance Policies: The life insurance corporation has got different policies suiting to the needs of different persons. These policies are as Under:
Business Studies Class 11 Important Questions Chapter 4 Business Services 1

1. Whole life policy: This policy reins throughout the whole life of the assured. The sum assured becomes payable to the beneficiary only after the death of the insured. The amount of premium is comparatively lesser in this policy. The period of the policy is indefinite. Its payment is made to the dependents of the insured only.

2. Endowment policy: This policy is for a fixed amount and specified period. If the policyholder survives for the period of the policy he is paid the specified amount of the policy with a bonus. An endowment policy provides both protection as well as savings for old age.

3. Joint life policy: This life policy is taken by two or more persons jointly. These persons may be partners of the firm, employees of an organization, members of the family, etc. According to the terms of the policy, the premium is paid jointly. On the death of anyone member of the group, the entire amount of the policy or the amount as per agreement is paid.

4. Annuity life policy. In this policy, the amount of the policy is paid before the maturity of the policy after one year or the year or the expiry of the fixed period. The entire amount of the policy is paid on the death of the policyholder or after the expiry of the period of the policy, whichever is earlier.

Question 4.
Differentiate Life Insurance and Fire Insurance.
Answer:
Difference between Life Insurance and Fire Insurance:

Points of difference Life Insurance Fire Insurance
I. Element of safety/investment It has got both the

the element of safety and investment.

It has got an element of safety only.
2. Surrender value The assured can surrender the policy before its maturity. The insured cannot surrender the policy.
3. Contract of indemnity It is not a contract of indemnity. It is a contract of indemnity.
4. Number of years The life insurance policy is taken for many years. Fire insurance policy is taken for one year. It has to be renewed after one year if it has to be continued.
5. Insurable interest It must be present at the time of the contract It must be present both at the time taking policy and at the time of loss.

Question 5.
Explain the term Double Insurance and Re-Insurance and differentiate the two.
Answer:
Double Insurance: Any person is free to take more than one insurance policy for the same property or goods. But he cannot recover more than the amount of loss actually suffered because a contract of insurance is a contract of indemnity.

Reinsurance: When an insurer risks that are beyond his control, he may get the whole or a part of his risk reinsured with other insurers. This is known as reinsurance.

Points of difference Double Insurance Reinsurance
1. Meaning Ensuring the same risk with two or more companies is Double Insurance. It is a contract of sub-insurance between the insurer and the reinsurer.
2. FilingClaim A claim can be filed with all insurers but restricted to an actual loss in case of fire and marine policies. The insured will claim compensation from the original insurer, who will claim compensation from reinsurers.
3. Contribution The contribution will be made by each insurer in proportion to the sum insured. Reinsurers are not directly required to contribute to losses.

Question 6.
What is the role or advantages of insurance in business?
Answer:
Role/Advantages Of Insurance In The Business: The future is always uncertain. Uncertainties in the business make plans futile and investments valueless. In order to minimize risk different types of insurance policies are taken. Insurance policies are advantageous in the following respects:

Role/Advantages of Insurance in the Business:

1. Protection
2. Indemnity
3. Diffusion of Risk
4. Social utility
5. Industrial Development

1. Protection: Insurance provides protection against the risk of loss. Incaseoflife insurance the loss of an individual cannot be compensated but the receipt of the insured amount from Life Insurance Corporation helps him in standing. Insurance enables the businessman to carry on business with confidence and peace of mind.

2. Indemnity: The loss caused by fire and other mishappenings is compensated by the insurance company. Insurance, as such is the protection against losses and businessman feels secured and free from anxiety.

3. Diffusion of Risk: The burden of loss is distributed among a large number of persons through insurance. The impact of loss on one industry is not unduly heavy, that can be transferred by the insurance company to others.

4. Social utility: Insurance provides safety to the common man. It is a means of social security. It also generates employment opportunities.

5. Industrial Development: The insurance companies collect a huge amount as a premium. These funds are invested in industrial ventures and cause industrial development.

Question 7.
What do you understand by the term Merchant Bankers? Mention in brief the services provided by Merchant Bankers.
Answer:
Merchant Bankers: Merchant bankers or lead managers undertake the management of new capital issues of companies. A merchant banker helps the company intending to raise fresh capital in drafting the prospectus (or statement in lieu of prospectus), the arrangement of underwriters, selection of brokers, publicity, the appointment of the registrar to the issue, etc. In India, several banks have specialized divisions or subsidiaries for offering merchant banking services.

Merchant bankers provide a wide variety of financial services to the corporate sector. They look into various legal and procedural aspects involved in the issue of securities and the raising of loans. They also provide consultancy in matters of investment, capital restructuring, valuation, merger, acquisition, etc.

Question 8.
Define Reserve Bank of India and its important functions:
Answer:
Reserve Bank of India: It was incorporated on April l, 1935 as a shareholders’ bank. The majority of shares were held by the central government. After independence, the Reserve Bank of India was nationalized on April l, 1949. Reserve bank of India performs the following important functions:

Functions Of Reserve Bank Of India:
A. Primary functions:

  1. Issuingcurrencynotes(except one rupee note, which is issued by the ministry of finance)
  2. Bank of the government
  3. Working as the banker of banks
  4. Controlling bank rate or rate of interest
  5. Controlling exchange rate.

B. Subsidiary functions:

  1. Dealing in foreign exchange
  2. Discounting bills of different banks
  3. Dealing in government securities
  4. Accepting deposits without interest
  5. Managing clearing houses
  6. Managing agricultural credit
  7. Extending short-term loans to banks and financial institutions
  8. Regulating developmental, industrial, and commercial activities in India.

Question 9.
Explain the term E-banking and the services of E-banking in today’s economic world.
Answer:
E-Banking (Electronic Banking): Many banks have introduced electronic banking services for their customers. Like IDBI Bank, UTI Bank, Global Trust Bank, Citibank, State Bank of India, etc. They make use of computers and satellites for the transfer of funds and communication. E-banking includes the following services:

1. EFT (Electronic Fund Transfer System): It is a cost-saving scheme for the convenience of customers. Under the schemes accompany may transfer wages and salaries from its bank account to the personal accounts of its employees. Similarly, a company can distribute dividends to its shareholders electronically. This is a very safe method of transfer of money as compared to banker’s draft, traveler’s cheque, etc.

2. ATMs (Automated Teller Machines): Many banks have installed ATMs in big cities. An ATM renders a teller’s job 24 hours a day. It is a self-service terminal that renders the facility of withdrawal and deposit of money to the bank customers. Each customer is given a separate plastic card to avail of the services at the ATM.

The customer has to insert the plastic card into the terminal and enter his identification code. The machine would then respond to the customer’s instruction of giving cash, taking a deposit, and handling other banking transactions.

3. Credit Card: The card issued to selected customers to enable them to make payment of credit bills of the credit limit. It is also called plastic money as it allows the credit cardholder to withdraw money without making any deposit into the bank. It allows an overdraft facility to the customer up to a specified limit. The cardholder can use the card for making payments for goods and services to the suppliers having Internet service is provided in India by many companies like VSNL, Bharti Telecom, and MTNL known as Internet Providers.

Any individual or organization can open an account with any Internet Service Provider who will give an account number for a monthly or yearly charge. Then the user may have access to the internet and the e-mail through it.

4. Debit Card: This is the card issued to the holder of a bank account against the balance amount in the account to facilitate and simplify the payment, withdrawal, and transfer of money any time, anywhere through the computer is known as a debit card.

Question 10.
Banking is the lifeblood of the economy. Mention the role or importance of Banking in the economy.
Answer:
Role/Importance Of Banking In National Economy:
As capital is the lifeblood of trade, commerce, and industry, so banking, in the same way, is the lifeblood of the economy. The importance of banking can be justified on the following grounds:

The modem economy is helpless without banking services. Banking as the lifeblood of the economy has assumed the following significance.
1. Credit creation: Banks accept deposits, retain a nominal percentage of the deposit as a cash reserve, and the best major part is lent to trade, industry and commerce at a higher rate of interest. It is known as credit creation.

2. Mobilisation of savings: Banking accept surplus savings and return together with interest, whenever required. It inculcates the habit of savings among people. It is responsible for capital formation.

3. Safe custody of valuables: The banks provide locker services and keep our valuables like ornaments, notes, documents, etc. safe, we can take them from banks whenever required.

4. Promotion of foreign trade: Finance is the lifeblood of all trading activities, even foreign trade. Banks are the source of funds, help in the payment and transfer of money, provide foreign exchange, issue letter of credit, and provide assistance to foreign trade in many ways.

5. Social and national welfare: Surplus funds of the people are accepted as deposits by banks and lent to trade, commerce, and industry for productive purposes. It promotes the welfare of the people.

Question 11.
Define the term internet and its various benefits and uses in the modern world.
Answer:
Internet is a worldwide or global network of computers, connected through telephone lines and other high-speed links. It is a facility of communicating and sharing information with mi 11 ions of people all over the World Internet Service is provided in India by many companies like VSNL, Bharti Telecom, and MTNL, known as internet service providers.

Any individual or organization can open an account with any internet service provider who will give an account number for a monthly or yearly charge. Then the user may have access to the internet and the e-mail through its information in a variety of forms.

Benefits/Uses of the Internet: Nowadays the internet is being used in every place and every sphere of life, whether it is a share market, shopping complex, companies, organization, departments, or Government understandings. There are a lot of buyers and sellers who use the internet as their buying and selling market. A newspaper or magazine can be read out through it. Seminars, conference, workshop, meeting, the conversation is also possible on the internet.

Thus, the benefits/uses of the internet can be classified as follows:
1. Sharing information: The first and foremost use of the internet is to share information. Company employees and many other people can share research and business data among colleagues and like-minded individuals

2. Communication- Internet facility is also used for communication Through the internet individuals can make communicate directly in various “chat-sessions” and E-mail facilities. With the help of modern technology people can talk face-to-face with the help of a Digital Camera which is possible only with the internet.

3. Marketing of products: Various Multinational Companies (MNC’s), departmental stores, shopping complexes, manufacturers, organizations, etc. used the internet for selling their products. Buyers also used interest to buy different kinds of goods from different corners of the world.

4. Entertainment: The Internet is a good source of entertainment. It offers a lot of entertainment programs as you can play any game by downloading it on your computers. You can come to know the latest Hollywood and Bollywood information about your favorite stars and films. You can hear songs and see films on the internet.

5. Making of queries: The Internet provides the facility to make queries regarding various places, products, films, books, personalities, institutions, etc. That’s why. a high and suitable institute, college, or school with its features and co-curricular activities can be approached.

6. Feedback and suggestions: Through the internet, many institutions and governments can take feedback and suggestions about their decisions and orders (e.g. Daily voting regarding decisions in Hindustan Times and Aaj Ki Baat).

7. Other uses: The Internet facility is also used to find out vacancies and provide opportunities. It can also be used for matrimonial and many more things.

Question 12.
Discuss the various characteristics of an Ideal warehouse.
Answer:
Characteristics Of An Ideal Warehouse: Efficient storage facilities are essential for the smooth flow of trade and commerce.

The essential features of good storage facilities are as follows:
1. Economy: An ideal warehouse must not be very expensive. The benefits provided by it should be much greater than the cost involved in its establishment and maintenance. Its hire charges should be reasonably low.

2. Safety: A good warehouse must provide adequate protection to the goods stored in it. It should be properly constructed to safeguard the goods from heat, moisture, rats, fire, theft, etc.

3. Proximity: The warehouse should be easy Iy accessible. It should be centrally located so that goods can be loaded, unloaded, and transported quickly at the minimum possible cost. It should provide easy or convenient entry and exit of goods.

4. Security: A good warehouse should facilitate pledging or hypothecation of the goods kept therein. It should provide for the supervision and control of the pledged goods. It should be capable of being insured at low rates of premium.

5. Control-There must be proper arrangements for the inspection, measurement, identification, etc. of goods. Proper care should be taken of the stored goods. There should be trained and experienced staff to manage the warehouse.

Question 13.
Differentiate between Private Warehouses and Public Warehouses.
Answer:
Difference between Private Warehouses and Public Warehouses:

Points of Difference Private Warehouse Public Warehouse
1. Capital Large investment in equipment. trained personnel and other facilities. There is no need for capital investment in public warehouses.
2. Risk More risk of obsolescence due to change in demand and technology. Minimum risk is involved investment in public warehouses.
3. Tax benefit Advantages of depreciation allowance white calculating taxable income. No property advantage due to free-port State real estate is taxless.
4. Operating cost Operating costs are lower. if sufficient volume is stored. Higher due to inclusion of profit factor. selling and advertising costs.
5. Economies of scale. Depends on company volume. Possible due to serving a large number of customers.
6. Storage and handling Cost Only estimated costs of storage and handling are made. Storage and handling costs depend on decision making.

Question 14.
What are Bonded Warehouse and its significance in business?
Answer:
Bonded warehouses are those licensed by the government to accept imported goods for storage before the payment of customs duty by the importers of such goods. Such warehouses work under the supervision and control of customs authorities. Bonded warehouses are located near ports. The goods stored in a bonded warehouse cannot be withdrawn without payment of necessary duty charges. These warehouses are very useful in foreign trade.

Significance Or Objectives Of Bonded Warehouse:
1. Facility in the payment of customs duty: The importer after depositing goods with the bonded warehouse gets sufficient time to arrange funds for the customs duties. There is also an opportunity to pay customs in parts and get the required quantity of goods cleared from the bonded warehouse.

2. Possibility of sales even without clearance of goods: Warehouse receipt is a negotiable instrument, so goods can be sold by endorsing warehouse receipt to the purchaser, who will have to pay customs duty for clearing goods.

3. Exemption from customs duty in case of the report: If goods are imported for exporting abroad, the importer can get it cleared from the bonded warehouse without the payment of customs duty’.

4. Encouragement to foreign trade: Bonded warehouse facilitates foreign trade by its location at the ports and deferred payment of custom duty.

5. Borrowing facility: Warehouse receipt issued by bonded warehouses can be deposited with the bank as security for obtaining loans.

Question 15.
Differentiate between service and goods.
Answer:
Difference between service and goods:

Points of Difference Services Goods
1.Nature An activity or Process A Physical Product
2. Type Heterogeneous Homogenous
3. Tangibility Intangible Tangible
4. Ownership Not transferable Transferable
5. Inventory Cannot be kept as stock Can be kept as stock
6. Involvement Participation of customers at the time of service delivery Transferable at the time of delivery not possible
7. Inseparability Simultaneous production and consumption Separation of production and consumption
8. Production Services are performed not produced Goods are produced

Question 16.
Write a short note on types of services?
Answer:
Services can be classified into:

  1. Business Services
  2. Personal Services
  3. Social Services

1. Business Services: Business services are those services which are used by business to conduct its day to day work. Like Warehousing, Transport, Banking, Insurance, Communication, etc. There are gaps of distance, time, knowledge, and risk between the production and consumption of goods. Business services bridges these gaps. The need for services arises because production is concentrated whereas customers are scattered widely.

2. Personal Services: Those services which are experienced differently by different customers are called personal services. These services are not consistent in nature. They will differ depending upon the service provider. They will also depend upon the customer’s preferences and demands, for example, tourism, restaurants, etc.

3. Social Services: Social services are those services that are performed for the benefit of society. They provide voluntarily in pursuit of certain goals. These goals may improve the standard of living of the weaker sections of the society or may educate them or provide health care and hygienic conditions in slum areas. For example health care and education services provided by certain NGOs and government agencies.

Question 17.
Explain the various types of Banks?
Answer:
Banks can be classified into the following:

  1. Commercial Banks
  2. Cooperative Banks
  3. Specialized Banks
  4. Central Banks

1. Commercial Banks: These banks perform all kinds of banking business. They accept deposits from the public and provide short term loans and advances to customers. They act as financial intermediaries or dealers in debt. Commercial banks are regular banks.

In India, there are three types of commercial banks:

  1. Public Sector Commercial Banks
  2. Private sector commercial banks
  3. Foreign Banks

2. Cooperative Banks: These banks are formed and set up the principles of cooperations. They are registered under Cooperative Societies Act. They provide credit and other facilities to their members. The members may be farmers, small scale traders, etc. They accept deposits from the members and grant loans to them at low rates of interest.

3. Specialised Banks: Specialised banks are foreign exchange banks, industrial banks, development banks, export-import banks catering to specific needs of these unique activities. They provide financial aid to industries, heavy turnkey projects, and foreign trade.

4. Central Bank: Central bank is the apex institution that supervises and controls the entire banking system of the country. Every country has a central bank. The Reserve Bank of India is the central bank of our country. A central bank does not deal directly with the public and its aim is not to earn profits. It also acts as a government banker. It controls and coordinates the currency and credit policies of any country. It maintains the exchange rate. It is the custodian of foreign currency reserves of the country.

Question 18.
What do you understand by insurance? How is it facilitates business?
Answer:
Insurance is a means of spreading risks. It involves the pooling of risks. A group of people who are subject to an insurable risk contributes regularly and the fund so created is utilized to compensate those unfortunate few members of the group who actually suffer a loss due to some unexpected density:

In other words, Insurance is a social device for pooling arid dividing risks among a large number of persons.

Importance of Insurance: Insurance plays a very important role in business. It provides the following advantages of businessman:
1. Protection against risks: It provides protection against risks of loss. By providing security against heavy risks and losses, insurance stimulates the expansion of trade and industry.

2. Division of labor- It facilitates the division of labor. A businessman can concentrate fully on his own business because his risks are taken over by an insurance company that is a specialist in risk-bearing.

3. Ability to face competition- It helps in improving the efficiency of business because an insured businessman feels more safe and active. Insurance contributes to the survival and continuity of business.

4. Better utilization of capital: In absence of insurance facilities businessmen will have to maintain large reserves to face risks due to loss. Insurance avoids the need for such reserves and the fund so released can be invested to generate additional wealth.

5. Facility for loan: Insurance companies provide loans and underwriting facilities to businessmen and also invest in industrial securities. After insuring his goods, the creditworthiness of the businessman increases.

Business Services Important Extra Questions Long Answer Type

Question 1.
What is Business Finance? Explain the various types of business finance and their uses ¡n business.
Answer:
Business Finance:
Ours is a developing economy. Agriculture is the dominant source of income. The 11m citations of agriculture here and the exploding population results in mass unemployment and non-Litilisation of resources. The remedy lies in the rapid growth of the business. consisting of trade, industry, and commerce. The establish is out. nursing and growth of the trade, industry, and commerce owe to the finance. It is rightly said that finance is the lifeblood of business.

The ancient business was very small in size, so own funds and loans from friends and relatives were sufficient to carry on business activities. Modem business has assumed a large size. It has become complicated and complex. Mechanization, specialization, and tough competition are their common features. Own funds are incapable of meeting present financial requirements. This is why, we have developed new sources of finance such as shares, debentures, public deposits, and institutional finance.

Types of Business Finance: Finance is required at every stage of the business. We need different types of finance for different purposes. On the basis of purpose the finance can be classified as under:

Manufacturers require long term finance more than the traders, who purchase and sell goods for-profit motive. More long-term finance will be required if the size of the business is larger.

Special features of long term finance:

  1. Finance required for a period of more than 5 years is known n as long-term finance. According to certain authorities, finance for a period exceeding ten years is known as long term finance.
  2. Long-term finance is required for making investments in fixed assets, such as land, building, plant, machinery, vehicles, equipment, furniture, etc.
  3. Long term finance meets the long term financial needs of the business. These needs are the permanent needs of the business.
  4. Fixed assets purchased out of long term finance are revenue-generating.
  5. Long term finance once invested in the business cannot be taken without dissolving the business or scaling down the business.
  6. Long-term finance is acquired through the issue of shares, debentures, or loans from specialized financial institutions.

(2) Medium Term Finance:
Medium-term finance, also known as medium-term capital is required for a period ranging between 2 to 5 or 2-10 years (The period of finance depends upon the nature and purpose of the business).

Medium-term finance is required for the following business activities:

  1. Introduction of a new product.
  2. Expenses on the modernization of business.
  3. Advertising campaign.
  4. Investment in permanent working capital.
  5. Adopting new methods and techniques of production.

Medium-term finance is raised through debentures, banks, public deposits, and financial institutions. Medium-term finance is required by manufacturing industries more than trading industries. Medium-term expenditures are also treated as capital expenditure because it is supposed to be revenue-generating.

3. Short Term Finance: Short term finance, also known as, short term capital or working capital is required for a period not exceeding one year. It is required to meet the day-to-day needs of the business. Short term finance is used again and again in the business, so it is also known as circulating capital.

Uses of Short Term Finance:
The short term finance is required for the following business activities:

  1. Meeting short-term financial needs of the business.
  2. Purchase of raw material.
  3. Payment of direct expenses i.e. expenses on acquiring and manufacturing goods i.e. carriage, freight, wages, power and manufacturing expenses, etc.
  4. Payment of selling, distribution, and administrative expenses. Trading concerns require more short term finances than manufacturing concerns. Lesser short term finance will be required if the gap between production and sale is lesser. Small factories need lesser short term finance than a large manufacturing enterprise.

Question 2.
Explain the various types of banks and their important functions:
Answer:
1. Commercial Banks: These banks perforin all types of the banking business. They accept deposits from the public and provide short-term loans and advances to customers. They act as financial intermediaries. State bank of India is the largest commercial bank in India. These banks have been established to assist trade commerce and industry.

2. Agricultural Banks: Agriculture needs both short-term and long-term loans. Short-term loans are required for purchasing seeds, manure, tractor, cattle, and tube wells, etc. These short-term needs are met by cooperative banks. Long-term financial needs are regarding the purchase of a land, tractor, other equipment and for installation of tube wells., Long-term financial needs are met by land mortgage banks also known as an agricultural bank. These banks are very helpful for the fanning community.

3. Industrial Banks: These banks provide long-term funds, so their requirement cannot be effectively met by commercial banks. They assist in the promotion of new industrial units. Unfortunately in India, we do not have a sufficient number of industrial banks. In place of industrial banks, we have the industrial development bank of India and various financial corporations to meet the financial needs of industrial enterprises. In western countries like the UK and the USA, there is a large number of industrial banks.

4. Exchange Banks: These banks provide finance for foreign trade. Many Indian commercial banks provide exchange services also. Most foreign banks work as exchange banks in India. These banks are City banks, Bank of Tokyo, Grindlays Bank, Chartered Bank, etc. These banks deal in foreign exchange and assist importers and exporters.

5. Indigenous Banks: Before independence financial needs of fanners and small business units were met by indigenous banks in rural areas. These are money lenders who accept deposits and grant loans. The working of these banks is so simple that farmers and borrowers may approach them at any time. Borrowers have to pledge their ornaments, land, and valuables for borrowing funds.

These banks are virtually exploiter of poor rural people. In spite of our development in financial fields by establishing big banks and financial corporations, indigenous banks are still serving the needs of the poor masses.

Special features of Indigenous banks:

  1. Loans can be granted for any purpose.
  2. Loans are granted at a higher rate of interest.
  3. Loans may be granted without security and a pledge.
  4. (They are easily accessible.
  5. Their method of work is simple.
  6. They may not observe business rules and regulations.

6. Central Bank: Reserve Bank of India: Every country has a central bank responsible for the overall control of cash and credit money in the economy. In India, the Reserve Bank of India works as the Central Bank of the country.

Question 3.
What are the various functions of a bank to be performed for the benefit of customers?
Answer:
Business Studies Class 11 Important Questions Chapter 4 Business Services 2

1. Accepting Deposits: The basic function of a bank is to accept deposits from the general public and business community. People deposit their money in banks for sake of safety. The deposits can be collected in the following forms:
(a) Current Accounts/Demand Deposits
(b) Saving Deposits,)
(c) Fixed Deposits,
(d) MiscellaneousdepositslikeRecunringDeposits, Cumulative Deposits, Cash Certificates, Old Age Pension Plan, etc.

2. Granting of Loans: The second important function of a bank is lending. Commercial banks lend finance to businessmen, farmers, artisans, and others. It generally makes advances to those who can pay back the principal with interest.

A bank makes advances to businessmen in various ways:
(a) cash credits;
(b) overdrafts;
(c) loans.

3. Discounting of Bills of Exchange: To meet the credit needs of businessmen, a bank can extend financial assistance through discounting bills of exchange. When a businessman needs funds, he may get his bills of exchange discounted with the bank. The bank will deduct the discounting charges (i.e. interest on credit) from the amount of the bill and credit the account of the customer with the balance amount.

4. Agency functions: A commercial bank acts as an agent of its customers in the following ways:
(a) Collecting bills of exchange, promissory notes, and cheques.
(b) It collects dividend and interest warrants of its customers.
(c) It collects the salaries and pensions of its customers.
(d) Itpurchasesand sells securities the instructions its customers.
(e) It executes standing instructions like payment of rent, electricity bill, telephone bill, insurance premium, etc.

5. Miscellaneous Services: A commercial bank performs the following general utility services:
(a) Safecustodyofvaluableslikegoldjewelleryandimportantdocuments.
(b) Providing lockers for the safe custody of Jewellery, valuable documents, and other valuables.
(c) Remittance of money through drafts, mail transfer, and telegraphic transfer.
(d) Purchase and sale in foreign currencies.
(e) Giving references about the financial position of customers.
(f) Underwriting issue of shares and debentures.
(g) Advising customers on financial and investment matters.

Question 4.
What are the major contributions of the Life Insurance business for the development of safety and security of the public?
Answer:
Importance of Life Insurance: Life insurance renders a variety of services to individuals, to groups, to business houses, and to society in general.

The important contributions made by life insurance are as follows:
1. Protection against Premature Death-Life Insurance provides protection to the dependents of the assured in case of his untimely death. The dependents get a large sum in case of death of the person insured. If he has taken a whole life policy, there will be a considerable relief to the dependants.

2. Provision for Old Age: Life Insurance also marked good provision for old age. After retirement. The earning capacity of a person is substantially reduced. He cannot maintain his standard of living. Endowment insurance affords comfortable support in old age and the money is available just when it is most urgently required.

3. Promote on of Thrift or Savings: Life insurance is one of the most important agencies for the promotion of savings. It fosters compulsory savings as the premium assumes the character of a debt or an obligation to be met. If any premium is not paid on the due date, the policy may lapse. Only surrender value can be obtained.

4. Sense of Security: The policyholder and his dependants have a sense of security as their future is financially secured. If the policyholder survives, he will get the stipulated money from the insurance company during his old age and if he dies, his dependants will get money from the insurance company.

5. Commercial Value-Life insurance policy can be used as collateral security to raise loans. It serves as a basis for credit. When a life policy after remaining in force for a good time acquires a cash value, it can be furnished as collateral security to acquire a ready loan in times of stringency.

6. Social Utility: If the earning member of a family undertakes a suitable life insurance policy, it will make the family self-reliant in case the earning member dies. This will relieve the society of a great problem of supporting such families. Life insurance serves as a measure of social security.

7. Funds for Investment-The life insurance companies accumulate vast sums in the form of a premium. This fund has rightly been called “a vast economic reservoir” which furnishes a good means of investment for the economic development of the country.

Question 5.
Explain in detail the comparative study of life, fire, and marine insurance businesses.
Answer:
Comparative Study Of Life, Fire And Marine Insurance:

Basis Life Insurance Fire Insurance Marine Insurance
1. Indemnity or Compensation It is not a contract of iri1emnity. The assured. if he survives. or his dependants have entitled to the sum assured. It is a contract of guarantee. It is a contract of indemnity. The insured can claim the actual market value of the property destroyed by fire. It is a contract of indemnity. The insured can claim the market value of the ship and the cost of goods destroyed by perils of the sea plus a reasonable margin for anticipated profits.
2. Insurable Interest Insurable ¡ntcrest must be present at the time of taking the policy. Insurable interest must be present at the time of taking the policy and at the time of actual loss. Insurable interest must be present at the time of loss only.
3. Assignment A life assurance policy can be assigned without the permission of the insurance company. A fire insurance policy can’t be assigned without the permission of the insurance company. Marine insurance policy can be assigned without the permission of the insurance company.
4. Period Life assurance is for a long period ranging from 5 to 30 years. whole life policy may also be taken. A fire insurance policy is generally for one year. Marine insurance policy is either for a voyage or for one year. h may also be a mixed policy.
5. investment It contains both the element of investment & safety. The element of protection only. it contains elements of protection only.
6. Double Insurance The benefit of double or multi-insurance is allowed. The benefit of double insurance is not allowed. The benefit of doubt insurance is not allowed
7. Surrender or Paid-up ‘value Provides both surrender value and paid-up value. Does not provide surrender or paid-up-value. Does not provide surrender or paid-up-value.
8. Contingency of Risk Risk is inevitable. Every policy becomes a claim sooner or later. Risk is contingent and uncertain. Every policy does not become a claim. Risk is contingent and uncertain. Every policy does not become a claim.

Question 6.
Discuss in detail the various clauses in a Life Insurance Policy.
Answer:
Clauses In A Life Policy (Terms and conditions)

A life policy is a legal document in writing containing the contract between the insurer and insured. It determines the terms and conditions under which a life has been insured. It is issued after the proposal form, duly filled in and signed by the proposer, is accepted by the insurer and the first installment of the premium is paid.

The main clauses or conditions in a contract of life insurance are as follows:
1. Premium-Premium is the payment or price of insurance payable by the insured. The amount of premium depends upon the sum assured age of the insured, and period of policy taken.

2. Days of Grace: Days of grace are the days after the due date of the premium. The premium can be paid within the days of grace. In the case of the monthly payment, 15 days of grace are allowed to pay the premium from the due date. In other cases, 3 0 days of grace are allowed. In case the assured dies within the days of grace without paying the installment of premium, the premium due shall be deducted from the full amount of the policy, insured.

3. Proof of Age: Proof of age is necessary because the rate of premium charged is decided on the basis of the age at taken the policy. If the date of birth is not admitted at the time of the policy, it will have to be proved to the satisfaction of the insurance company at the time of the claim. Proof of age may be given in the form of a school certificate, horoscope, birth certificate from the municipal authority, certificate of baptism, the service book, or a declaration by a court of law.

4. Nomination and Assignment: The insured can nominate anyone at any time during the period of the policy. The nominee is the person who will get the amount of the policy in the event of the death of the assured. The nomination is made by an endorsement of the policy.

Assignment of a policy implies the transfer of the policy to a third person. A life insurance policy can be assigned freely to any person with or without consideration. The assignment can be made at any time but it must be in writing, witnessed, and registered with the insurer.

The assignment can be made by an endorsement on the policy or on a separate signed instrument. The person making the assignment is known as the assignment, and the person to whom the policy is assigned is called the assignee, upon a valid assignment, all rights in the policy vest in the assignee who can sue on the policy in his own name. By registering an assignment or nomination the insurer does not accept responsibility as to its validity or legal effect.

5. Surrender Value: If a person is unable or unwilling to pay the premium, he may surrender the policy and ask for the surrender value which is the cash value of the policy. Surrender value depends upon the types of policy and the amount of premium paid. Usually, it is one-third of the total premium paid. A policy acquires surrender value only when the premium has been paid for at least three years.

6. Paid-up Value: When a policyholder wants to terminate the policy, he may convert his policy into a paid-up policy. The Paid-up value is equal to the premium paid and, therefore, it is higher than the surrender value. But the amount of paid-up value is payable to the insured only at the time of maturity of the policy. A policy can be made paid-up only after it has remained in force for at least two years.

7. Incontestable Clause: Under the Insurance Act 1950, no life policy can be contested after it has run for two years except in case of fraud or proof of age.

8. Forfeiture: This clause states that the occurrence of any fault or fraudulent event will result in the forfeiture of the right in the policy. Non-payment of premium, concealment of material facts, furnishing wrong information, etc. are some examples of these faults and fraudulent actions.

9. Revival of Lapsed Policies: When the premium is not paid within the days of grace, the policy lapses. But the policy may be revived within a period of five years from the due date of the first unpaid premium and before the date of maturity.

10. Claims: This clause states the office of the insurance company at which the amount of the policy is payable.

Question 7.
Explain the various clauses mentioned in Marine Insurance Policy.
Answer:
Clauses In Marine Policy: A policy of marine insurance may contain several clauses. Some of the clauses are common to all marine policies while others are included to meet the special requirements of the insured.

Some of the important clauses in a marine policy are described below:
1. Valuation Clause: This clause states the value of the subject matter insured as agreed upon between both parties.

2. Sue and Labour Clause: This clause authorizes the insured to take all possible steps to avoid or minimize the loss or to protect the subject-matter insured in case of danger. The insurer is liable to pay the expenses, if any, incurred by the insured for this purpose. It is also called S.L.T. (Sue, Labour, and Travel clause).

3. Waiver Clause: This clause is an extension of the sue and labor clause and it covers both the insurer and the insured. The clause states that any act of the insured or the insurer to protect, recover or preserve the subject matter of insurance shall not be taken to mean that the insured wants to forget the compensation, nor. will it mean that the insurer accepts the act as an abandonment of the policy?

4. Reach and Stay Clause: This clause requires the ship to reach and stay at such ports and in such order as specified in the policy. If nothing is mentioned, the ship must reach and stay at such ports which are usually called at in that particular trade route. Any departure from the route mentioned in the policy or the ordinary trade route followed will be considered as deviation unless such departure is essential to save the ship or the lives on board in an emergency.

5. Warehouse-to-Warehouse Clause: This clause is inserted to cover the risks to goods from the time they are despatched from the consignor’s warehouse until their delivery at the consignee’s warehouse at the port of destination.

6. Inchmaree Clause: This clause covers the loss or damage caused to the ship or machinery by the negligence of the master of the ship as well as by explosives or latent defect in the machinery or the hull.

7. F.P.A. and F.A.A. Clause: The F.P.A. (Free of Particular Average) clause relieves the insurer from particular average liability. The F.A.A. (Free of all Average) clause relieves the insurer from liability arising from both particular average and general average.

8. Lost or Not Lost Clause: Under this clause, the insurer is liable even if the ship insured is found to be lost prior to the contract of insurance, provided the insured had no knowledge of such loss and he did not commit any fraud. This clause covers the risks between the issue of the policy and the shipment of the goods.

9. Running down Clause (RDC): This clause covers the risk arising out of a collision between two ships. The insurer is liable to pay compensation to the owner of the damaged ship. This clause is used in hull insurance.

10. Free of Capture and Seizure Clause: This clause relieves the insurer from the liability of making compensation for the capture and seizure of the vessel by enemy countries. The insured can insure such abnormal risks by taking an extra ‘war risks’ policy.

11. Continuation Clause: This clause authorizes the vessel to continue and complete her voyage even if the time of the policy has expired. This clause is used in time policy. The insured has to give prior notice for this and deposit a monthly prorate premium.

12. Barratry Clause: This clause covers losses sustained by the shipowner or the cargo-owner due to the willful conduct of the master the crew of the ship.

13. Jettison Clause: Jettison means throwing overboard a pan of the ship’s cargo so as to reduce her weight or to save other goods. The cargo is thrown deliberately to save the ship from sinking. The jettison clause covers the loss arising out of such throwing of goods. The owner of jettisoned goods is compensated by all interested parties.

14. At and From Clause: This clause covers the subject matter while it is lying at the port of departure and until it reaches the port of destination. It is used in voyage policies. If the policy contains only the words ‘from’ instead of and from’, the risk is covered only from the time of departure of the ship.

Question 8.
Discuss in detail the various types of fire insurance policies
Answer:
Types of Fire Policies: Different types of fire insurance policies are issued to meet the varying needs of the public.

The important fire insurance policies are discussed below:
1. Valued Policy: Valued policies are the exception in fire insurance Undervalued policy, the value declared in the policy is the amount the insurer will have to pay to the insured in the event of a total loss irrespective of the actual value of the loss. This policy is usually issued for artistic wort antiques, work of art, and similar rare articles whose value cannot lx determined easily.

2. Specific Policy: Under this policy, the insurer undertakes to make good the loss to the insured up to the amount specified in the policy. Suppose a building worth Rs.40,000 is insured against fire for Rs.20,000. If tin- damage to the property is Rs. 15,000, the insured will get the full compensation Even if the loss is Rs.20,000, the insured will get the full amount. But if the tin loss is more than Rs. 20,000 (/.e. the amount insured), the insured will Rs. 20,000 on !y. Hence, the value of the property is not relevant in determining the amount of indemnity in the case of a specific policy.

3. Average Policy: The policy contains an ‘average clause’, the insured is penalized for under-insurance of the property. The insured liable for such proportion of the loss as the value of the uncovered property bears to the value of the whole property. The amount of elm is arrived at by dividing the insured sum by the actual value of the property and then multiplying it by the amount of loss.

Thus, if a person gets his house insured for Rs.8,00,000 though its actual value is Rs. 12,00,000, if a part of the house is damaged in a fire and the insured suffers a loss of Rs.6,00,000, the amount of compensation to be paid by the insurer comes out to Rs.4,00,000 arrived as follows:
Business Studies Class 11 Important Questions Chapter 4 Business Services 3

4. Floating Policy: A floating policy is taken for covering fluctuating stocks of goods held in different lots and several places under one sum and for one premium. It is taken by big businessmen whose goods 1 ie in different stations, ports, and warehouses. With every transaction of sale or purchase, the quantities of goods kept at different places fluctuate. It is difficult for the owner to take a policy for a specific amount. The best way open to him is to take out a floating policy for all the goods lying at different places.

5. Reinstatement Policy: In such a policy, the insurer has the right to reinstate or replenish the property destroyed instead of paying compensation to the insured in cash.

6. Consequential Loss Policy: The insured has to suffer a greater financial loss on account of the dislocation of the business caused by fire. When a fire breaks out in a factory, it disrupts production because the factory has to be closed down for repairs. The production suffers and so also profits of the insured. He has also to meet the fixed expenses such as rent, rates, taxes, salaries, and other expenses as usual. Such considerable loss to the insured is not covered by the ordinary fire policy. In order to cover such loss by fire, the “ConsequentiaJ Loss Policy” has been introduced. The loss so suffered is separately calculated from the loss actually suffered.

Question 9.
Explain the various Marine Policies undertaken by the Insurer under Marine Insurance Contract.
Answer:
Types of Marine Policies: The important types of marine insurance policies are discussed below:
1. Voyage Policy: It is a policy under which the subject matter is insured for a particular voyage irrespective of the time involved to cover transit from one part to another e.g., Mumbai to New York or New York to London. The risk starts from the departure of a ship from the port and it ends on its arrival at the port of destination.

2. Time Policy: It is one under which the insurance is affected for a specified period of time, usually not exceeding twelve months. Tin*e poles are generally used in connection with the insurance of the ship.

3. Mixed Policy: It is a mixture of voyage and time policies. Under it, the subject matter is insured for a certain voyage and time, e.g, Kolkata to New York, for a period of one year. Mixed policies are generally issued to ships operating on particular routes.

4. Valued Policy-It is one under which the value of the subject-matter insured is specified on the face of the policy itself. The value which is agreed upon is called the insured value. It forms the measure of indemnity in the event of a loss. Insured value is not necessarily the actual value.

It includes:
(a) the invoice price of goods;
(b) freight, insurance, and other charges;
(c) ten to fifteen percent margin to cover expected profits.

5. Open or Unvalued Policy: When the value of the subject-matter insured is not specified and agreed upon. It is known as an open policy. In such a policy the value is ascertained as the time of damage.

6. Floating Policy: A merchant who is a regular shipper of goods can take out a ‘floating policy’ to avoid the botheration and waste of time involved in taking a new policy for every shipment. This policy stands for the contract of insurance in general terms and leaves the details to be declared subsequently.

Thus, the insured takes a policy for a huge amount and he informs the underwriter as and when he makes shipment of goods. The underwriter goes on recording the entries in the policy. When the sum assured is exhausted, the policy is said to be “fully declared” or “run-off’.

7. BlockPolicy-Thispolicycoversotherrisks also in addition to marine risks. When goods are to be transported by ship to the place of destination, a single policy is known as block policy may be taken to cover all risks.

Question 10.
Explain the term E-Mail used in communication. Also, give its working and significance.
Answer:
E-Mail-the great new way to send messages: E-mail or electronic mail is the recent electronic media to send messages. It is an addictive way to communicate with people all over the world throughout the day and night without waiting for the recipient to wake up and making himself present. The messages may consist of short notes and greetings, text files plus graphics, images, video clips, or sound.

Working of E-mail:
Step I: We compose the message first. After that, the messages are input by a keyboard and sent off to a machine at ISP (known as a mail server) which handles all the mail from its members.
Step II: The mail server locates the mail server which accepts the recipient’s incoming email.
Step III: After this our mail server send our message to the recipient’s mail server at his E-mail address. ,
Step IV: The message waits at the recipient’s mail server.
Step V: The recipient checks for new mail and picks up the message.

Communicating messages through E-mail is a very short process. Most/of the E-mail is delivered in a matter of minutes.

Significance of E-Mail:

  1. Easy to communicate: Sending messages through E-mail is easy. It is simple to operate E-mail equipment.
  2. Fast communication: Messages are sent within minutes to distant places.
  3. Economical Taking into consideration the distance and the contents of the message, sending messages through E-mail is economical. If you have got your own computer with an Internet facility e-mail is virtually free.
  4. Secrecy of the message: Messages sent through E-mail remain a secret between the communicator and communicatee.

Electronic mail or e-mail is joining popularity in modern business organizations. Companies around the world are now using e-mail to enhance their effectiveness.

E-mail is fast replacing the telephone and the fax communication mode. Its cost is lower and has a very high speed of travel. It takes only a second to reach the receiver at any distance of the world. It ensures a higher degree of secrecy of the message.

Question 11.
What factors to be considered while selecting public warehouses and Private warehouses.
Answer:
Choice Between Public Warehouses And Private Warehouse: While making a choice between the public warehouse and private warehouses the following factors should be considered:
1. Capital investment: No investment is required in the case of public warehouses. On the other hand, a considerable initial investment is required for the construction of a private warehouse. In addition to the building, start-up equipment and trained personnel are required.

2. Operating costs: A user of a public warehouse has to pay high rent because the owner of a public warehouse seeks profits. Operating costs of private warehouses are lower provided full space is used.

3. Economies of scale: A public warehouse can enjoy economies of scale because it serves a large number of customers. The owner of a private warehouse can achieve economies of scale only when sufficient volume is stored.

4. Control over cost: A public warehouse has to perform well on short-term contracts and, therefore, it is motivated to exercise effective cost control. The owner of a private warehouse has direct responsibility for staff and procedures which permit control over cost.

5. Risk: The use of public warehouses involves nominal risk. Private warehouses involve the risk of obsolescence due to changes in demand and technology.

6. Consolidation of shipments: A public warehouse can consolidate various shipments to and from the warehouse. But no such consolidation is possible under a private warehouse.

7. Storage and handling costs: In a public warehouse these costs are accurately known to the user. But these costs can only be estimated in the case of a private warehouse.

8. Tax benefit: A user of a public warehouse does not get a tax benefit. But the owner of a private warehouse can claim depreciation allowance while calculating his taxable income.

Private, Public and Global Enterprises Class 11 Important Extra Questions Business Studies Chapter 3

Here we are providing Business Studies Class 11 Important Extra Questions and Answers Chapter 3 Private, Public and Global Enterprises. Business Studies Class 11 Important Questions with Answers are the best resource for students which helps in class 11 board exams.

Class 11 Business Studies Chapter 3 Important Extra Questions Private, Public and Global Enterprises

Private, Public and Global Enterprises Important Extra Questions Short Answer Type

Question 1.
What are the major objectives of public enterprises?
Answer:
Public enterprises are guided by several socio-economic and political objectives.

Importance/Need/Objectives of Public Enterprises: Public enterprises have got the following motives:
1. Economic Motive/Objectives:
(a) Availability of essential commodities of life in sufficient quantity at cheaper rates: These industries aim at supplying essential commodities at cheaper rates to the people. Mother Daily and Delhi Milk Scheme in Delhi are its examples. Water, electricity, and gas supply are also in this category.

(b) Establishment of basic industries: In order to accelerate the pace of economic growth, basic industries are a must. These industries require heavy expenditure and Carry a lot of risks. This is why private entrepreneurs hesitate in establishing basic industries. These industries are iron and steel, locomotives, aircraft, and ship-building, etc.

(c) Balanced regional development: Private industries are not keen to set up industries in remote and backward regions due to a lack of facilities and infrastructure. It leads to the haphazard and unbalanced development of the country. Public enterprises aim at balanced regional development of the country in addition to their economic motive.

2. Social Motive: Involvement of the government in these enterprises is because of the social welfare of the people. Satisfying the basic needs of the people at cheaper rates shows the social motive of these enterprises. They help to check malpractices on the part of private entrepreneurs.

3. Political Motive: Ministers, Members of Parliament and Legislative Assemblies try to establish a public enterprise in their constituencies for political gain This results in the growth of the area, and the states of the politician, and thus political motive is achieved and regional imbalance takes place.

4. Reduced wasteful expenditures: Wasteful expenditures can be reduced with the nationalization of industries, because under private ownership company may not be managed efficiently, so expenditures of 11 that company may be excessive. Therefore, there is a need b nationalization.

5. Self-Reliance: These enterprises save valuable foreign exchange through import substitution In addition, public sector enterprises export goods on a large scale to attain the goal of economic self-reliance.

6. Infrastructure: Transport, communication, irrigation, power, etc. can be developed only on large scale, with huge resources but a private owner cannot have huge resources. So there should be a nationalization of infrastructure.

Question 2.
Differentiate between Private and Public Sector Enterprises.
Answer:
Difference between Private and Public Sector Enterprises:

Basis Private Sector Enterprise Public Sector Enterprise
1. Ownership It is owned and controlled by private individuals. It is owned and controlled by the Central or State Govt
2. Profit motive The profit motive is very important and essential for its survival. The profit motive is not the main guiding factor. It serves social objectives as well.
3. Purpose The motivating factor is to earn profits. Its main objective is to promote public welfare.
4. Social Objectives Social objectives are not seriously undertaken. It is guided by social objectives like the development of backward regions, the creation of employment, and the equitable distribution of wealth.
5. Efficiency Quick decision-making promotes efficiency. Decision-making is not fast. There is red-tapism. bureaucratic control has reduced its efficiency.
6, Government Control It is not subject to strict financial control by the Government It is subject to strict financial control by the Government.
7, Management It is managed by professional managers. It is managed mostly by Government officials.
8. Public Accountability It is accountable to the owners. It is accountable to the public through Parliament.

Question 3.
Mention, in brief, the significant features/characteristics of Global Corporations.
Answer:
Features of Global Corporation: The important features of most multinational corporations are as under:
1. Giant-Size: The assets and sales of MNCs run into billions of dollars and they make huge profits through their operations. For example, the physical assets of IBM are worth around 8 billion dollars. The sales turnover of some global corporations exceeds the gross national product of several developing countries.

2. International Operations: An MNC operates in many countries through a parent corporation in the home country. It runs its operations through a network of branches, subsidiaries, and affiliates in host countries. Production, marketing, and other operations are scattered in different countries to get the economies of local operations.

3. Centralized Control: An MNC has its headquarters in the home country in order to control the branches and subsidiaries. The local management of branches and subsidiaries operate within the policy framework of the parent corporation. This is possible due to the fact the parent-company holds40% to 100% of the equity of the subsidiary company.

4. Oligopolistic Power: Many multinational corporations enjoy oligopolistic power. They occupy a dominant position in the market. Through the process of merger and takeover of other firms, an MNC may acquire a huge economic power. This makes it oligopolistic in character because of which it has a dominant position in the market. For example, Hindustan Lever Limited acquired Tata Oil Mills to improve its market shares.

5. Collective Transfer of Resources: A multinational corporation facilitates a multilateral transfer of resources. Such transfer takes place in the form of a” package” which includes technical know-how, machinery, and equipment, raw materials, management expertise, etc.

6. Professional Management: These corporations employ professional skills, specialized knowledge, and training. These managers have specialized training and skills in different functional areas like finance, marketing, and human resources.

7. International Market: MNCs have access to international markets as a result of vast resources and superior marketing skills. Because of this, MNCs are in a position to sell whatever products they manufacture in different countries throughout the world.

Question 4.
Explain the term Foreign Collaboration in brief.
Answer:
Foreign Collaboration: Enterprises having equity participation V of foreign units is known as foreign collaboration. It is an enterprise jointly owned, managed, and controlled by Indians and foreigners. These enterprises are enjoying the vast resources of the country and abroad. It combines the financial resources and managerial and technological expertise of two or more countries.

The amount of profit is shared ( between the Indian owner and the foreign partner. The government these days has liberalized its policies regarding foreign participation in Indian i: enterprises. According to New Industrial Policy, 1999 approval will be given for direct foreign investment up to 51% equity in high priority industries and automatic permission will be given to foreign technology agreements in identified high priority industries.

Question 5.
What do you mean by the term “Disinvestment of Shares”?
Answer:
Disinvestment of Shares-The major plank of the privatization program in India has been the disinvestment of government shareholdings in a select number of public enterprises. The rationale behind this program is to raise a non-inflationary form of finance for the budget.

The program commenced in 1991-92 and till 1995-96 the „ government had disinvested a part of its equity in 40 public sector enterprises and had raised an amount of Rs. 10,915 crore through the various rounds of disinvestment undertaken between this period. The shares were offered to financial institutions, mutual funds, private sector enterprises, and the general public as well.

The Union Government constituted Disinvestment Commission in August 1996 to advise it on the disinvestment program of public sector enterprises. The government referred 40 public sector undertakings to the Commission for advice about disinvestment. The government has now been taking steps to implement the report of the Disinvestment Commission.

To ensure transparency in the Disinvestment program, bids are invited for the sale of shares of selected public enterprises and the shares are sold to the highest bidder. During 2000-01, the Government raised Rs. 1,869 crores through disinvestment in public undertakings as compared to Rs. 1,829 crores during 1999-2000 and Rs. 5,371 crores during 1998-99. The target of disinvestment for the year 2002-03 was Rs. 0 crores.

Question 6.
What is Departmental Undertaking? Mention its advantages and disadvantages.
Answer:
Departmental Undertaking – It works as the ministry or a department of the government. The budget of these departmental organizations is presented to the parliament just like other ministries. Indian Railways and Post and Telegraph departments are its examples.

Departmental organizations are entirely owned and controlled by either the Central Government or by a State Government.

Advantages of Departmental Undertaking: Departmental organization enjoy the following merits:
1. Service Motive: These enterprises are formed with a service motive. Public interest and social welfare hold priority for them. These undertakings also help to reduce the burden of tax on the public.

2. National Importance: Activities that have got national importance are performed by these departmental organizations. The risk of misuse of public money minimized due to strict budget, accounting, and audit.

3. Secrecy: These organizations are capable of maintaining secrecy. because these are under the control of the government. The government can avoid disclosure of facts on the plea of public interest.

4. Proper Management: These enterprises are managed by qualified government staff. Their work is systematic. They are properly v managed and supervised. Such control and management keep the government official alert.

Disadvantages of Departmental Undertaking – Departmental organization suffer from the following disadvantages:
1. Least profit earning venture: Departmental organization is owned and controlled by the government. It is formed with a service motive, so it does not remain an excellent profit earning venture.

2. Red tape: Employees follow the beaten track. They do not take much interest in the work. They are careless and bother much for their salaries. Officers worry much for their status and respect. Decisions are generally delayed due to bureaucratic procedures and political considerations.

3. Lack of competent workers: Government employees are not much efficient in business affairs. They have sufficient administrative experience but not experienced enough to manage the activities of, the enterprise. Promotion to the higher rank is based upon seniority, so competent employees are not recruited.

4. Political evils: Every important decision in the departmental organization has a political motive. It is managed and controlled by the minister, who is the representative of a political party. The minister has to look after the interest of his party.

5. Lack of competition: Generally departmental organizations have the status of monopoly. Lack of competition makes them incompetent. In the absence of competition and profit motive, there is little incentive for hard work and efficiency. There is hardly any link between reward and performance.,

Question 7.
What are statutory/public corporations? Present their advantages and disadvantages.
Answer:
Statutory/Public Corporations: Public corporations are; formedbythespecialactofParliamentorLegislativeAssemblies. Their existence is separate from the government. This is why these corporations are called autonomous bodies. Though these corporations are independent in financial matters, even then they remain under the control of the government. It is an autonomous body fully financed by the government.

AccordingtoMorrison, “Public corporation is a combination of public ownership, public accountability, and business management for the public end.” Examples of such corporations are Air India, Life Insurance Corporation of India, etc.

Merits/Advantages of Statutory Corporations – Public corporations enjoy the following merits:
1. Free from government control: These are autonomous bodies, which are not under the direct control of the government.

2. Service motive: These corporations are also formed in the public interest for social welfare like other public enterprises. Service motive dominates their activities.

3. Independent decision: Public corporations are independent in making their own decisions, policies, and plans.

4. Efficient management: These corporations are benefited from efficient management because they are managed by a competent board of directors, who are professional in their attitude and work. As changes in government do not affect its stability, it can take long term policy decisions.

5. Economic self-independence: Public corporations are financially independent. They have to arrange their own finances. It is free from political interference by ministers and bureaucrats.

Demerits/Disadvantages of Statutory Corporations – Public corporations have got the following disadvantages:

  1. Red Tape: Like other public enterprises and government departments public corporations are also victims of red-tapism.
  2. Rigid structure: The constitution of such a corporation is rigid. The objects and powers of such corporations cannot be changed without amending the statute, which is a time-consuming and cumbersome process.
  3. Theoretical autonomy: The autonomy of these corporations exist only on paper. In actual practice, interference by political bosses and ministers gives the wrong signal to their autonomy. Red-tapism and bureaucratic control reduce the flexibility of operations.

Question 8.
What is a Government Company? Define its merits and demerits.
Answer:
Government Company – A Government company is a company in Which not less than 51 percent of paid-up capital is held by the Central Government or State Government jointly by both Governments. It is formed and registered under the Companies Act 1956.

Merits of a Government Company: A government company enjoys the following benefits:
1. Internal autonomy: A government company is a separate entity and so can manage its affairs on its own. There is the absence of direction. parliamentary and ministerial control. Thus, it is an autonomous body. It can be operated on commercial principles. It can manage its affairs independently.

2. Flexibility in operations: It can be operated on commercial principles because of independence in respect of internal management, finance, and appointment of person.

3. Collaboration: It is the only form of organization by which the Government can make use of managerial skill, technical know-how, and expertise of the private sector.

4. Easy formation: It is relatively easy to form a government company as no statute is required to be enacted. It can be created by the executive decision of the Government.

5. Expert management: Since the annual report on the working of the enterprise is placed before the Parliament or the State Legislature, the management of a government company tends to be cautious, and efficient to avoid its criticism.

6. Sound management policies: It cannot afford to follow unsound policies because its working can be compared with similar companies in the private sector.

Demerits of a Government Company-The government companies usually suffer from the following weaknesses:
1. Board composed with yes men: The Board of Directors of a government company usually consists of politicians and civil servants which may not be able to follow sound business principles.

2. Theoretical autonomy: The independent character of a government company exists in paper only. Government officials, ministers, and politicians often interfere with its work.

3. Lacking responsibilities: A government company evades the constitutional responsibilities which other state enterprises owe to the Parliament. The Parliament is not, taken into confidence for its creation as government decision is sufficient in this regard. The accounts of a government company need not be audited by the Comptroller and Auditor General of India. Thus, it concentrates more power in the hands of the Government than a statutory corporation.

4. The practice of forming government companies has been regarded as a fraud on the Companies Act because the Government is empowered to specify the provisions of the Company Act to be followed or applied and modifications if any.

Question 9.
Present a comparative view of different forms of public enterprises.
Answer:
A comparative view of different forms of Public Enterprises:

Basis of comparison Departmental Organisation Statutory Corporation Government Company
1. Formation It is formed by a Ministry Concerned. Formed by Special Act of Parliament or State Legislature It is established Under the Companies Act by a Ministry with or without private sector participation.
2. Ownership The departmental organization is wholly under Government Department. It is wholly owned by the Government Central or State. At least 51 percent of paid-up share capital owned by the Government.
3. Legal Status No separate entity is distinct from the Government Department. It enjoys a separate legal entity. Separate legal entity as per Companies Act 1956.
4. Control /Management Control vested with Government officials from the Ministry concerned. Control is exercised by the Nominated Board of Directors. Board of Directors as required under Companies Act 1956.
5. Staffing System Staff mainly consists of Civil Servants- Govt. Service It is managed by professionals under a contract of service (not Govt. Service) Managed by professionals under a contract of service (not Govt. Service)
6. Financing Budgetary allocation, can’t borrow from the general public. Separate arrangements can borrow also. Separate arrangements can borrow also.
7. Autonomy It enjoys no autonomy. Sufficient autonomy. Govt does not interfere in day-to-day affairs. Sufficient autonomy Government does not interfere so much in operations.
8. Public Accountability (Parliamentary control) Highest Higher High accountability for investors and creditors.
9. Suitability The public utility, defense, etc. Industrial and Commercial Undertakings, Industrial and Commercial undertakings, providing for the participation of domestic and foreign private capital.

Question 10.
The role of the public sector is not satisfactory. What are the suggestions for improvement in public sector operations?
Answer:
Suggestions for the improvement of Public enterprises- In order to make public sector enterprises more efficient and prone to public welfare the government set up various committees from time to time. Some important committees set up by the government are as under:

Name of Committee Year of Inception of Committee/Commission
(1) Chagla Commission 1958
(2) Krishna Mohan Gupta Committee 1959
(3) Administrative Reforms Committee
(4) F. Mohammad Committee 1980
(5) Arjunsen Gupta Committee 1998

The annual evaluation is made by the Standing Conference of Public Enterprises (SCOPE) of al 1 committees made by the government. Some of the main and important suggestions offered by the various committees are as follows:

1. Labour policy: Public enterprises should design their labor policy in such a fashion that there should be a cordial relationship between the workers and the management. Actions should be initiated in such a fashion to minimize industrial disputes.

Appointments should be based not on pulls and pressures but on ability and expertise. Efficient workers must be suitably rewarded. The compensation of employees should be linked with their productivity. Workers must have a sense of belonging to their organization. There should be only one trade union in an industry whose role is to maintain industrial peace.

2. Price policy: The government has realized that the price policy of public enterprises should be based on the existing price structure because there is no clear-cut perception about the price policy of public sector enterprises. Prices of commodities must commensurate to the public. The seventh plan emphasized that public enterprises must generate at least a 10% profit margin. So, the price should be determined with a view to raising the level of profitability in the government monopolies and promote public welfare.

3. Commercial outlook: In any case; public sector enterprises must generate the surplus which may be plowed both in the form of investment for economic development. This is possible only when the cost is reduced and productivity is raised. It will benefit the consumer and the entire society. So, the public enterprises must adopt a commercial outlook by which there would Beacon-ordination between national and profitability.

4.Autonomy to public enterprises: Public sector enterprises should have complete autonomy and minimum possible government interference in operation and routine matters. Politicians should not be appointed to control public sector enterprises. However, incompetent and able politicians are available, they should better be appointed as honorary consultants. In no case, should they be directly involved in the control and management of the enterprises?

Only professionals should be appointed as managers. They have been given greater autonomy so as to make them more efficient and competitive.

5. Inspection of public enterprises: Public Sector Enterprises Bureau has been established for the annual inspection of public enterprises for preparing their annual progress reports offering guidelines to public enterprises. Public enterprises are also inspected by Public Accounts Committee and Estimates Committee. Every public enterprise should publish a comprehensive report of its functioning, so that people may come to know of their performance. The government lays down performance targets for the management of public sector enterprises.

Dr. Lanka Sundram suggested that “Parliamentary committees be appointed for inspection of public enterprises.” Accepting this suggestion the government set up the Parliamentary Committee of Public Undertaking in 1963 which controls the public sector enterprises.

6. Proper auditing: According to the Administrative Reforms Committee, there should be three or four audit boards established under the direct control of the Comptroller and Auditor General so that accounts of public enterprises should be properly audited. Any lapse in this regard must be brought to the notice of the government by an independent auditor.

7. Full utilization of productive capacity: The public sector enterprises should fully utilize their productive capacity; the Productive capacity of the existing enterprises should be raised only if at least 85% of the existing capacity is being utilized. Thus new industries and enterprises should be established only after complete utilization of the existing productive capacity.

Question 11.
DefineMultinational Corporations and their role in the economic development of a nation.
Answer:
Definition of Multinational Corporation (MNC): A Multinational Corporation refers to an organization that has its headquarters in one country and business operations in other countries. This means that this type of organization will have business across many countries. An MNC has its registered office in one country (called home country) and it carries its business operations in a number of foreign countries (called host countries).

A multinational corporation controls production and marketing facilities in more than one country. For instance, Coca-Cola is a company registered in the U.S.A., has production and marketing operations in many countries of the world.

Role of MNCs – Multinational companies have been playing an important role in several developing economies including India.

Multinationals can offer the following gains to the host country:
1.Investment of Foreign Capital: MNCs can help the developing economies to secure capital from the developed countries as they suffer from a shortage of capital required for rapid industrialization. They facilitate the transfer of capital from countries where it is abundant to countries where it is scarce. Thus, MNCs can help increase the investment level and thereby the pace of development of the host country. Since liberalization, India has attracted foreign investment worth several billion dollars.

2. Advanced Technology: The developing countries have old and obsolete technology. MNCs can be used as vehicles for the transfer of superior technology to developing countries. Advanced technological know-how, improved skills, and consultancy help the developing countries to improve the quality of products and reduce costs.

3. Creation of Job Opportunities: The MNCs set up facilities for the production and distribution of goods and thereby create employment opportunities it is In the host country. M u Itinational offer excess lent pay scales and career advancement opportunities to managers, technical and other staff.

4. Utilization ofldleResources: TheMNCs help in the utilization of idle resources of the host country and thus generate income for the country.

5. Creation of Healthy Competition-MNCs increase competition and break domestic monopolies. The inefficient firms are forced to either improve or withdraw from the market. Many Indian companies now compete with multinationals after liberalization through improved technology.

6. Professional Management: The MNCs kindle a managerial revolution in the host countries by professional management and the employment of the latest management techniques. The host countries are thus able to develop a culture of professional management. Multinationals build up a knowledge base through management techniques like MBO and corporate planning.

7. Growth of Domestic Firms: The MNCs can help the growth of domestic firms to supply them materials, components, etc. Over the years, several ancillary units have grown to provide support to the MNCs.

8. Higher Standard of Living: Because of their superior technology, MNCs provide a large variety of quality products to the people in the host country. This helps to increase their standard of living.

9. Integration with the World Economy: The MNCs facilitate the integration of the economy of the host country with the world market. They encourage international brotherhood and cultural exchanges in the host country.

Question 12.
Write down the full form of the following: BIFR, MOU, MNC, MOFA, VRS, NRF, PSES, IMF, IBRD.
Answer:

BIFR Board of Industrial and Financial Reconstructions
MOU Memorandum of Understanding
MNC Multi-National Corporation
MOFA Majority Owned Foreign Affiliates
VRS Voluntary Retirement Scheme
NRF National Renewal Fund
PSES Public Sector Enterprises
IMF International Monetary Fund
IBRD International Bank for Reconstruction and Development

Question 13.
What are the objectives of privatizing public sector enterprises?
Answer:
The primary objectives of privatizing public sector enterprises are following:

  1. Releasing the large amount, to utilize on other social priority areas.
  2. Reducing the huge amount of public debt and interest burden.
  3. Transferring the commercial risk to the private sector.
  4. Freeing these enterprises from government control.

Question 14.
State some danger of MNC towards the domestic economy?
Answer:
Dangers from MNC:
1. Creation of Monopoly: MNC joins hands with big business houses and gives rise to monopoly and concentration of economic power in host countries.

2. Threat to National Sovereignty: These corporations tend to interfere in the political affairs of host nations. Some MNCs like IT1 are accused of overthrowing governments in countries such as Chile.

3. Alien Culture: MNCs tend to vitiate the cultural heritage of local people and propagate their own culture to sell their products. For example, MNCs have encouraged the consumption of synthetic food, soft drinks, etc. in India.

4. Depletion of Natural Resources: MNCs cause rapid depletion of some of the non-renewable natural resources in host countries.

Small Business and Entrepreneurship Class 11 Important Extra Questions Business Studies Chapter 9

Here we are providing Business Studies Class 11 Important Extra Questions and Answers Chapter 9 Small Business and Entrepreneurship. Business Studies Class 11 Important Questions with Answers are the best resource for students which helps in class 11 board exams.

Class 11 Business Studies Chapter 9 Important Extra Questions Small Business and Entrepreneurship

Small Business and Entrepreneurship Important Extra Questions Short Answer Type

Question 1.
Write the full form of the following:
SIDO, KVIC, DIC, LPG, NABARD, RSBDC, NSIC, StDBI, NCEUS, RWED, WASME, SFURTI, DWCRA, TRYSEM, WTO.
Answer:

(i) SIDO Small Industries Development Organisation
(ii) KVIC Khadi and Village Industries Commission
(iii) DIC District Industries Centers Liberalisation, Privatisation and Globalisation
(iv) LPG Liberal isation. Privatisation and Globalisation
(v) NABARD National Bank for Agriculture and Rural Development
(vi) RSBOC Rural Small Business Development Centre
(vii) NSIC National Small Industries Corporation
(viii) STDBI Small Industries Development Bank of hid la
(ix) NCEUS The National Commission for Enterprises in the IJnorganised Sector.
(x) RWED Rural and Women Entrepreneurship Development
(xi) WASME World Association for Small and Medium Enterprises
(xii) SFURTI Scheme of Fund for Regeneration of Traditional Industries
(xiii) DWCRA Development of Women and Children in Rural Areas
(xiv) TRYSEM Training of Rural Youth for Self-Employment
(xv) WHO World Health Organization

Question 2.
Compare different small scale business on the basis of the investment.
Answer:

Types of Industries Investment Unit Remarks
Small Scale Industry One crore Up to 5 crores in export
Ancillary Industry One crore 50% of output should be supplied to a parent unit
Tiny Enterprise 25 lakh
Service and Business enterprise 10 lakh
Women Enterprise Any of the above 51% equity holding by women and managed by women.
Export Oriented Unit One crore 100% EOU can sell 25% in the domestic market.

Question 3.
Name the major industry groups that come in the small scale sectors.
Answer:
The following major industry group comes in the small scale sectors:

  1. Food Products
  2. Paper Products and Printing
  3. Chemical and Chemical Products
  4. Basic Metal Industries
  5. Electrical Machinery and Parts
  6. Rubber and Plastic Products
  7. Machinery and Parts expect Electrical Goods
  8. Hosiery and Garments – wool product
  9. Non-metal lie Mineral Products
  10. Transport Equipment and Parts
  11. Leather and leather products
  12. Miscellaneous Manufacturing Industries
  13. Beverages, Tabacco and Tobacco products
  14. Repair Services
  15. Cotton Textiles
  16. Wool. Silk, Synthetic Fibre, and Textile
  17. Jute, Hemp, and Mesta Textiles
  18. Other Services.

Question 4.
What are the different forms of support offers to small industries by the Government?
Answer:
The Government provides support to anal industries in the following forms:

  1. Institutional support in respect of credit facilities.
  2. Provision of developed sites for the construction of sheds.
  3. Provision of trading facilities.
  4. Supply of machinery on hire purchase terms.
  5. Assistance for domestic and export marketing.
  6. Technical and financial assistance for technological Upgradation.
  7. Special incentives for setting up enterprises in backward areas.

Question 5.
What are the main characteristics of a small business?
Answer:
Any small business is characterized by at least two of the following key features:

  1. Management is independent. Usually, the managers are also owners.
  2. Capital is supplied and ownership is held by an individual or a small group.
  3. The area of operations is mainly local. Workers and owners are of one home community. Markets need not be local.
  4. Relative size within the industry the business is small when compared to the biggest units in its field. The size of the top bracket varies greatly so that what might seem large in one field would be definitely small in another.

Question 6.
Distinguish between cottage and small scale industries.
Answer:
The basis of classification between the cottage and small-scale business is that the cottage industry embraces a predominantly manual process of work.

The difference between the small-scale and cottage industries are basically two:

  1. Small-scale industries are mainly located in urban centers as separates establishments, the cottage industries are generally associated with agriculture and provide subsidiary employment in rural areas.
  2. Small-scale industries produce goods with partially or wholly mechanized equipment employing outside labor, while cottage industries involve operations mostly by hand which is carried on primarily with the help of members of the family.

Question 7.
What are the main objectives/features of small scale business?
Answer:
Objectives of small-scale business:

  1. To provide an opportunity for large-scale employment at a minimum cost.
  2. To provide a steady source of income to the low-income groups living in rural and urban areas of the country.
  3. To meet the growing demands of the consumer’s goods and simple producers goods.
  4. To mobilize resources of capital and skill and their optimum utilization.
  5. To eliminate the economic backwardness of rural and underdeveloped regions in the country.
  6. To attain self-reliance.
  7. To reduce regional imbalances.
  8. To effect an integration of the activities of a small business with the rural economy on the one hand and with the large scale business on the other.
  9. To reduce disparities in income, wealth, and consumption.
  10. To provide substitutes for various industrial products now being imported into the country.
  11. To improve the quality of industrial products manufactured in the cottage industry sector and to enhance both production and exports.
  12. To remove the problems created by urbanization and the consequent growth of big towns and cities.

Question 8.
Discuss the role and importance of small-scale enterprises in the economic development of India?
Answer:
The small-scale sector promotes entrepreneurship and helps to earn foreign exchange and is very important to the Indian economy.

The following points will highlight the importance of small scale enterprises.

  1. Innovation and productivity: It is the small-scale enterprises that lead to innovation and productivity although they do not maintain their own research and development wings.
  2. Individual tastes, fashion, and personalized service: Small- scale firms are receptive to change in taste and fashions of consumers and in adjusting the production process accordingly.
  3. Symbols of national identity: They are locally owned and controlled. They can strengthen the social system and cultural traditions of India. They are perceived as valuable symbols of national identity.
  4. The tendency of dispersal over wide-area: Small-scale enterprises have a tendency to disperse over wide areas. More than 62.19% of the units are located in the backward areas.

Question 9.
What type of problems faced by small scale sector in the field of marketing?
Answer:
Problems faced by the small scale sector in marketing their products are briefly enumerated below:

  • Lack of standardization.
  • Financial weakness.
  • Unfamiliarity with expert activities-procedures and market know-how.
  • Poor designing.
  • Poor quality.
  • Ignorance of potential markets.
  • Competition.
  • Lack of quality control.
  • Lack of precision.
  • Lack of knowledge of marketing.
  • Distribution contacts.
  • Poor finish.
  • Poor bargaining power.
  • Brand preferences.
  • The scale of production.
  • Lack of service after-sales.

Forms of Business Organisation Class 11 Important Extra Questions Business Studies Chapter 2

Here we are providing Business Studies Class 11 Important Extra Questions and Answers Chapter 2 Forms of Business Organisation. Business Studies Class 11 Important Questions with Answers are the best resource for students which helps in class 11 board exams.

Class 11 Business Studies Chapter 2 Important Extra Questions Forms of Business Organisation

Forms of Business Organisation Important Extra Questions Short Answer Type

Question 1.
Differentiate sole proprietorship and partnership form of business.
Answer:
Difference between Partnership and Sole Trader:

Points of Difference Partnership Sole Trader
l. Specific Act It is governed by Partnership Act 1932. There is no specific Act.
2. Number of Member The minimum number of partners is two and the maximum number in the case of banking business is ten and in other business is twenty. It is owned and carried on by only one person. He may employ other persons or take help from the members of his family.
3. Agreement It arises only by agreement among partners. No agreement is required in a sole proprietorship.
4. Distribution of profit Profit is shared among partners. The entire profit is enjoyed by the proprietor alone.
5. Capital It has got more capital because there are more members. It has limited capital because the capital is contributed by one person only.
6. Secrecy In a partnership business, secrets are open to each partner. Business secrecy is maintained.
7. Personal touch It does not have a personal touch as much as the sole trader has with his customers. It is located amidst consumers, so it has personal contact and touch with them.

Question 2.
What is partnership deed and mention in brief the provisions contained in partnership deep?
Answer:
Partnership Deed: A partnership agreement contains the terms and conditions relating to partnership and the rules and regulations governing its management. It may be oral or in writing. A written agreement of partnership is called ‘Deed of Partnership’. A partnership deed contains all the details on which partnership has been formed. These terms and conditions are also known as articles of partnership.

A partnership deed usually contains the following details:

  1. The names and addresses of the partnership firm and its partners.
  2. The nature of the business proposed to be carried on by the firm.
  3. The duration of the partnership.
  4. The amount of capital contributed by each partner.
  5. The rate of interest payable to partners on their capital or to be paid by partners on the amount drawn by them.
  6. The mode of maintaining accounts and operation of the bank account.
  7. Rights and duties of the partners for the management of the business of the firm.
  8. The ratio in which profits will be shared by the partners.
  9. The amount of salary and/or commission payable to the partners
  10. Arbitration clause for settlement of disputes between the partners,
  11. Mode of dissolution and settlement of accounts.

Question 3.
What are the advantages and disadvantages of employment of Paid Assistant insole proprietorship business?
Answer:
Employment of Paid Assistant When the sole proprietor employs a paid assistant, he has the following advantages and disadvantages –

Advantages:
(a) Division of work: A specialist assistant can be appointed whose expertise can be used for the benefit of the business. By delegating some of the work, the proprietor can concentrate on more important matters.

(b) No share in profits: The assistant is not entitled to any share in the profits of the firm. He gets a fixed salary which is an expense of the business. The assistant is not given a share in the profits.

(c) Complete control: The paid assistant has no right to interfere in the decision making. Therefore, the proprietor has full control over the affairs of the business.

(d) Independent decision: The proprietor can take decisions independently without consulting the assistant. There is no interference from the assistant.

(e) Easy to dismiss: The proprietor can terminate the services of the assistant as and widen he likes.

Disadvantages:
(a) Lack of motivation: The assistant does not have sufficient incentive to work hard unless he is given a share in the profits. Therefore, he may not be as sincere and careful as the proprietor himself,

(b) Lack of sharing right: The employee is not responsible for the losses incurred in the business. The risk of failure has to be borne by the proprietor himself. The monthly salary of a paid assistant is assured as long as he remains in business.

(c) Problem of capital: Appointing a paid assistant does not solve the problem of finance. The employee does not bring any capital with him.

(d) Disclosure of secrets: The business is, in a way, at the mercy of the paid assistant. He may leak out trade secrets to competitors or join them. He may quit his job and set up his own business in competition.

Question 4.
What are the advantages and disadvantages of admitting a partner in a sole proprietorship form of business?
Answer:
Admission of a Partner:
By taking one or more partners, the proprietor obtains the following benefits and drawbacks –

Advantages:
(a) Availability of additional capital: The new partner brings some capital into the business. This strengthens the financial position of the business.

(b) Division of work: Work can be divided between the original proprietor and the partner on the basis of knowledge and skills. There is the pooling of judgment and experience. This will improve the efficiency of the business.

(c) Motivation: A partner gets a share in profits and, therefore, has an incentive to work hard for the success of the business. Admission of partners also increases the goodwill and borrowing capacity of the firm.

(d) Reduced risk: Each partner shares the loss and liability of a business. As a result, the risk of the sole proprietor is reduced.

(e) Economy of costs: No wage or salary is to be paid to the partner. Therefore, the cost of management is comparatively low.

Disadvantages:
(a) Profit-sharing: The proprietor has to share the profits with the partner.

(b) Dilution of freedom: Every partner has a right to be consulted. The proprietor cannot take decisions independently without consulting his partner. Freedom of action and complete control of one individual in the decision: making are lost. As a result, there may be delays in taking decisions.

(c) Lack of stability: By taking a partner the continuity of business is endangered. Lunacy, insolvency or death of one partner may terminate the partnership.

(d) Difficulty in removing partner: A partner cannot be pushed out from the business without the consent of all the other partners. The capital is blocked as a partner cannot withdraw his capital or transfer his interest to outsiders without the approval of the other partners.

(e) Source of disputes: When the partners are unable to take decisions unanimously, conflicts may develop between the partners.

(f) Risk of dishonesty: If a partner is not fair and honest in dealings, the risk of the business may increase manifold.

Question 5.
Explain the difference between a private limited company and a public limited company form of business.
Answer:
Difference Between Private Company And Public Company:

Basis Private Limited Company Public Limited Company
1. Number of Members Minimum – 2, Maximum – 50 Minimum – 7, Maximum As large as paid-up shares divided by the share lot.
2. Articles of association It must prepare its own articles of association. It may adopt Table A given, in the Companies Act
3. Minimum no. of Directors Minimum -2 Minimum – 3
4. Use of the word ‘Limited’ Use the word ‘Private Limited’ after its name. Use only the word ‘Limited after its name.
5. Commencement of Business Can commence business immediately after Incorporation. Can commence business only after complying with certain statutory’ formalities and obtaining the ‘Certificate to Commence Business.
6. Issue of Shares and Debentures Prohibited from inviting the public to subscribe to its shares and debentures. Can issue its shares and debentures to the general public.
7. Issue of Prospectus Not required to issue a prospectus. Can proceed to allot after incorporation. It must issue a prospectus or statement in lieu of a prospectus. Can proceed to allot shares only after compliance with certain statutory formalities.
8. Transferability of Shares Restricted by the Articles of associations. Shares are freely transferable.
9. Share Certificates Cannot issue share warrants or share certificates. Can do so.
10. Statutory Meeting Not required to bold such a meeting. Required to hold such a meeting and submit a statutory report to the Registrar of Companies.
11. Qualification Shares Not prescribed for the directors. Prescribed as a stipulation to become a director.
12. Filling of Documents Need not send the list of directors and their consent to act as directors to the Registrar. Must send the list of directors and their consent to act as directors to the Registrar.

Question 6.
What is the difference between a Joint Stock Company and a Cooperative Society? Explain.
Answer:
Difference between Joint Stock Company and Cooperative:

Points of Difference Joint Stock Company Cooperative
1. Formation Companies are formed under the Companies Act. 1956. It is formed under the Cooperative Societies Act, 1912 in general.
2. Number of Members There must be at least 2 members in Private and 7 in Public company. The maximum number in the case of a Private company is fifty and unlimited in the case of a Public company. There should be at least ten members to form a cooperative. The maximum number is unlimited, as many as the number of shares.
3. Objective The profit motive is the main objective. Service motive is the main objective.
4. Liability The maximum liability of its shareholders is limited to the face value of shares held by them The liability of its members may’ be both limited and unlimited.
5. Transfer of shares The shares of the public company are transferable. Shares are not transferable but can be returned to society.
6. Voting rights One share one vote is the principle regarding voting rights of the company. One member one vote
7. Distribution of profits A dividend is distributed on the basis of shares held by the shareholders. The dividend is distributed on an equitable basis i.e. equal to all members irrespective of the number of shares held by them.
8. Return of capital No member can demand back his capital except at the time of winding up. A member can demand his capital during the Lifetime of the society.
9. Privileges No special exemption except in the case of a Private Company. Special exemptions by the government.

Question 7.
Explain in brief the merits or advantages of a joint-stock company.
Answer:
Merits/Advantages of Joint Stock Company:
A joint-stock company form of business organization is based on the following advantages –
1. Permanent existence: The life of the company is permanent, ft is not affected by the death, incapability, lunacy, and insolvency of the shareholders. It has a separate legal entity. The ownership and the management of the company change smoothly without the dissolution of the company.

2. Limited liability: The liability of a shareholder is limited to the face value of shares held by him. The personal assets of the shareholders cannot be attached, even if the company is unable to meet the claims of outsiders.

3. Availability of large capital: The capital of the company is contributed by its shareholders, whose number is unlimited as much as the company requires. Different types of securities can be issued to mobilize funds from different kinds of investors.

4. Transferability of shares: The shares of the company are listed on the stock exchange so that member can easily sell their shares. These special features also ensure that the company will not be required ‘to refund the capital. The shares of the company are purchased and sold in the stock exchange in the open market.

5. Economies of large scale: The company form of a business organization provides tremendous scope for growth and expansion. urge capital facilitates. This is why the company enjoys internal and external economies of large scale enterprise.

6. Tax relief: Tax law s offer certain developmental rebates and concessions on certain commodities of export promotion and for the establishment of industries in backward regions. The company is charged income tax at the Hat rate. As such the tax liability on higher-income is comparatively lower.

7. Diffused risk: The risk of business is shared among innumerable shareholders, so every shareholder has to bear the nominal risk. This is not the case in proprietorship and partnership, where the loss has to be borne by the individual proprietor and a limited number of partners of a firm individually or collectively.

Question 8.
Mention in brief the main features of sole: proprietorship.
Answer:
Features of Sole Proprietorship: The salient features or characteristics of sole proprietorship form of organization are discussed below:

1. Single Ownership: A sole proprietorship is wholly owned by an individual. It is run entirely at his risk of loss. The sole trader provides both capital and management to the business from his own resources or borrowed funds.

2. Common Identity: A sole trader ship concern has no separate 1 legal entity independent of the owner. The owner and business exist together. Thus, there is no difference between the sole trader and his business.

3. Capital: Insole tradership, the capital is employed by the owner himself from his personal resources. He may also borrow money from his friends and relatives for investment in the business.

4. Unlimited Liability: The proprietor is personally liable for all the debts of the business. The creditors have the right to recover their dues even from the personal property of the proprietor in case the business assets are not sufficient to pay the debts.

5. Management and Control: Sole leadership is a one-man show. The sole trader provides management to the business. He takes all the decisions, procures materials and other resources, employs workers, and directs and controls the affairs of the enterprise. He is not required to consult anyone else in taking any decision. The sole trader may delegate some of his authority to his employees, but the ultimate authority to manage and control rests with him.

6. No Profit Sharing: The sole proprietor alone is entitled to all the profits and losses of a business. He bears the complete risk and there is nobody to share the profits or losses.

(vii) No Legal Formalities: No legal formalities are required to start, manage and dissolve this type of business. Only a license is necessary for certain business-like chemist shops etc.

Question 9.
Explain the meaning and important features of the Joint Hindu Family business.
Answer:
Meaning of Joint Hindu Family (JHF) The Joint Hindu Family firm is a form of business organization in which the family possesses some inherited property and the ‘Karta’, the head of the family, manages its affairs. It comes into existence by the operation of Hindu Law and not out of a contract between the members or coparcener. If the persons who have coparcenary interest in the ancestral property canyon business, k is a case of Joint Hindu Family firm. Thus, the Joint Hindu Family Business is a business by a coparcener of a Hindu undivided estate.

The Joint Hindu Family Business may be defined as a form of business organization in which all the male members of a Hindu undivided family carries on business under the management and control by the head of the family called ‘Karta’. The property is managed and held by the senior male member of the father as the Head of the Family, technically known as Karta’.

In Hindu law, a family business is taken as a part and parcel of the inheritable property, and therefore’, the family business becomes the subject matter of coparcenary interest. The rights and liabilities, of coparcener, are determined by the general rules of the Hindu Law. It should be noted that a joint family firm is created by the operation of law and does not arise out of a contract between the coparceners.

Features of Joint Hindu Family Firm:
The Joint Hindu Family Firm possesses the following features –
1. Status: The membership of the family business is the result of / status arising from birth in the family. There is no question of the members being discriminated against in terms of minority and majority on the basis of age.

2. Male Members: Only male persons of the family can claim coparcenary’s interest in the Joint Hindu Family business firm. The male child becomes copartners immediately on his birth.:

3. Karta: The right to manage the business vests in Karta alone. He has the legal right to obtain loans through a mortgage, etc. for the purpose of the business. Other members have neither any right to manage the affairs of the business nor any right to take loans on the mortgage of business property.

4. Liability: The liability of Karta is unlimited and that of other members of the family is limited to the extent of their share in the property.

5. No need for Registration: The activities of a Joint Hindu Family business are governed by Hindu Law. But the law does not require any registration of the business.

Forms of Business Organisation Important Extra Questions Long Answer Type

Question 1.
Explain the important characteristics and differentiate between the various types of business enterprises.
Answer:
Characteristics of Business Enterprises:
The main characteristics of various types of business enterprises are given below –
1. Public Sector Enterprises: Public enterprises or public sector enterprises are those enterprises that are owned and operated by the government. The capital of such enterprise is contributed by the central government, state government, or the local government.

Their characteristics are as follows:
(a) State ownership: Public enterprises are owned by the government. Even where private entrepreneurs are permitted to invest capital, more than 50 percent of capital is in government hands.

(b) Government control: The management and control of public enterprise exclusively risk with the government. Parliamentary control is exercised over public enterprises.

(c) Service motive: The public welfare or service is the main objective of public enterprise though it may also earn profits. There is usually benevolent management in public enterprises.

(d) Public accountability: The capital of public enterprise is supplied from the public exchequer or government department in charge of public money. Therefore, public enterprises are accountable to the general public.

2. Private Sector Enterprises: The characteristics of private sector enterprises are as follows:
(a) Private ownership: It is owned and managed by a private enterprise or group of individuals. The entire share capital is provided by these businessmen.

(b) No state participation: There is no participation by the Central or state governments in the establishment and ownership of a private-sector enterprise.

(c) Independent management: The management and control of a private-sector enterprise are vested in the hands of one or more private businessmen.

Management is accountable to the owners (their elected representatives). There is no interference by the government in internal management.

(d) Profit motive: The main object of a private-sector enterprise is to earn profits rather than to render service to society.

3. Joint Sector Enterprises: The characteristics of joint sector enterprises are as follows:
(a) Mixed ownership: The government, private entrepreneurs, and the investing public jointly own a joint sector enterprise.

(b) Combined management: The management and control of a joint sector enterprise lie with the nominees or representatives of the government, private businessmen, and the public.

(c) Share capital: The shares of the government, private businessmen and the public in the capital are 26 percent, 25 percent, and 49 percent, respectively. The aim is to pool the financial resources and technical knowledge how of the state and the private individuals.

Comparison Between Private, Public, And Joint Sector Enterprises:

Point of Distinction Public enterprise Private enterprise Joint sector
1. Ownership Government-owned Private persons Government and private both
2. Management By government officials By private owners or professional managers Both government and private individuals
3. Capital 51 percent or more by the government By private investors Government and private both
4. Purpose Service to the society Barning profits Profit and social objectives
5. Government control Control by Parliament No strict control by Parliament Mayor may not be
6. Audit By Comptroller and Auditor General. Compulsory in all cases By practicing chartered accountants. Not compulsory in all cases By qualified auditors
7. Accountability To the public To the owner authority To both government and private

Question 2.
What is the scope of setting small business and also give reasons for considerable scope of setting small scale businesses in our country?
Answer:
Scope of setting up small business enterprises:
There is considerable scope for setting up small scale units due to the following reasons –

1. Limited Demand:
The demand for certain products is local and seasonal. In such cases, it is not economical to attempt a scale of operation which exceeds local demands. Brick kilns, hair: cutting saloons, restaurants, etc. are examples of such cases. In the case of perishable goods also, the size of firms tends to be small. In certain cases, the nature of the production process favors small units.

2. Specialised Service:
When an enterprise supplies specialized services, small scale firms are more suitable. Beauty parlors, interior decorators, and tailoring shops are examples of this type. A small firm can understand its customers and can provide personal attention which may not be possible in a large-scale enterprise. Similarly, firms providing professional services like eye clinic, tax consultancy, chartered accountancy, etc. are also organized as a small scale because they must maintain, personal touch with their clients. Thus, small firms are required to cater to individual tastes and fashions and to render personalized services to consumers.

3. Flexibility:
Certain businesses are subject to wide variations in demand, e.g. manufacture of jewelry, ready: made garments, etc. In such cases, greater flexibility of operations is required. Small firms can be more flexible due to simple technology and low overheads. They are capable of being adapted to changing tastes and fashions. They can easily make changes in products and can shift to new lines of business whenever the need arises. Therefore, small firms are more suitable for manufacturing and selling specialty items that may be popular for only a short period of time.

4. Employee relations:
When close rapport with employees is essential to provide high-quality products to the customers, small scale unit is in a better position. The owners, also the managers of such business have the most valuable advantage of being close to the employees. They know better their problems and can take necessary remedial measures quickly and efficiently.

5. Introduction of New Products:
Before starting the production of a new product on a commercial scale, it is always desirable to test it in the market. In the initial stages, the requirements of customers and management are uncertain and unknown. Therefore, operations are usually carried on a small scale when new products or ideas are being introduced in the market. This also helps to reduce the risk.

6. Direct Motivation:
Small scale enterprises foster individual initiative and skill. The identity of ownership and management serves to curb misconduct as mistakes bear directly on one’s property and income. There is maximum incentive to put the resources to best use because the resulting gains accrue directly to the owner. Red: tapis is absent and prompt decisions are possible.

7. Human Inertia:
Many businessmen do not want to expand their business due to fear of loss of freedom. Growth may involve more work and worry. People who want to lead a comfortable and simple life may be satisfied with the small scale of business.

8. Shield to Big Business Many small firms serve as ancillary units or feeders to large firms. Such units also provide a training ground for entrepreneurs. Small firms also provide some guarantee against the emergence of new competition. A threat to the big firms. They provide superficial evidence that monopoly does not exist in the industry.

9. Social Utility:
Small scale industries are helpful in generating self: employment for a large number of persons. These industries are also useful in preventing the concentration of income and wealth. They facilitate the economic development of rural and backward areas. Small firms use local resources and their social cost is comparatively low.

10. State Assistance and Patronage:
Small scale industries get several concessions from the government on account of their social benefits. The government provides then loans on concessional rates of interest. Technical, managerial, and marketing assistance is also provided. The government has reserved several products for exclusive production in the small scale sector. Several institutions have been set up to protect and promote the growth of small scale industries in the country.

Question 3.
Discuss the main types of partners.
Answer:
A partnership firm can have different types of partners with different roles and liabilities. There can be the following types of partners:

  1. Active Partner
  2. Sleeping or Dormant Partner
  3. Secret Partner
  4. Nominal Partner
  5. Partner by estoppel
  6. Partner by holding out
  7. Minor Partner

1. Active Partner: Those partners who contribute capital and also takes an active part in the management of the firm are called active partners. These partners act as agents of the firm and have unlimited liability. All other partners are responsible for their deals.

2. Sleeping or dormant partner: Those partners who contribute capital only but do not take an active part in the affairs of the business are called sleeping partners. They have shared in the profit loss of the firm and also have unlimited liability. But they do not come face to face with the third party.

3. Secret Partner: This type of partner contributes capital and takes an active part in the management of the firm’s business. He shares in the profit and losses of the firm and has unlimited liability. However, his connection with the business of a partnership firm is not known to the outside world.

4. Nominal Partner: Those partners who neither invest money nor have shared in the profit and loss and also have no role in the administration of the firm. The firm makes them partners to gain from their personal goodwill. They have unlimited liability also.

5. Partner by estoppel: A person who by his words or conduct, represents himself as a partner becomes liable to those who advance money to the firm on the basis of such representation. He cannot avoid the consequences of his previous act.

6. Partner by holding out: When a person is declared as a partner and he does not deny this even after becoming aware of it, he becomes liable to third parties who lend money or credit to the firm on the basis of such a declaration.

7. Minor Partner: A minor is a person who has not completed 18 years of age. Minor may be admitted as a partner only for the benefits of the partnership with the mutual consent of all the partners. On being so admitted, a minor can impact and copy the books of accounts but could not take an active part in the management. His liability is limited to the intent of his share in the capital and profit of the firm.

Question 4.
Explain the various types of partnerships.
Answer:
A partnership can be classified on the basis of two factors:

  1. Duration,
  2. Liability.

On the basis of duration, there can be two types of partnership:

  1. Partnership at will,
  2. Particular partnership

On the basis of liability, the two types of partnership are:

  1. Partnership with limited liability
  2. Partnership with unlimited liability.

On the basis of Duration:
1. Partnership at will: It is a partnership formed for an indefinite period. It can continue for any length at any time depending upon the will of the partners. It can be dissolved by any partner by giving notice to the other partners of his desire to quit the firm.

2. Particular Partnership: It is a partnership formed for a particular objective. It is formed fora specific time period or to achieve specified objectives. It is automatically dissolved on the expiry of the specified period or on the completion of the specific purpose for which it was formed.

On the basis of liability:
1. Partnership with limited liability:
In this type of partnership the liabilities of partners are limited to the amount of capital introduced by them except one partner who has unlimited liability. Registration of such a partnership is compulsory. The limited partner could not take an active part in the firm’s management and their acts also do not bind the firm or other partners.

2. Partnership with unlimited liability:
This is also called a general partnership. In this liability of the partner is unlimited and joint. They enjoy the right to participate in the management of the firm and their acts are binding on each other as well as on the firm. Registration of this type of firm is optional. Because of unlimited liability, the firm’s creditors can realize these dues in full from any of the partners by attaching their personal property if the firm’s assets are found to be inadequate to pay off its debts.

Local Governments Class 11 Important Extra Questions Political Science Chapter 8

Here we are providing Class 11 Political Science Important Extra Questions and Answers Chapter 8 Local Governments. Political Science Class 11 Important Questions with Answers are the best resource for students which helps in class 11 board exams.

Class 11 Political Science Chapter 8 Important Extra Questions Local Governments

Local Governments Important Extra Questions Very Short Answer Type

Question 1.
What is Local Government?
Answer:
Local Government refers to the Government institutions at the local level which is very close to the people. Local Governments is about Government that involves the day to day life and problems of the people. The local government discusses and decides the local issues and problems with more and more involvement of the people of the area. Local governments include people friendly administration. It also involves the decentralization of powers and resources.

Question 2.
What is Grass-root democracy?
Answer:
Grass root democracy means strengthening the democratic process at the local level.

Grass root democracy ensures the meaningful participation of the people and also promote the accountability of the administration. For this strong and vibrant local governmental institutions are made.

Question 3.
What is the utility of the Local Governments?
Answer:
As Local governments involve the participation of the local people, therefore it ensures the solving the local problems with the involvement of the local people themselves. It is the mechanism of decentralization and efficient administration. -It gives the opportunity to the people to discuss and decide their own affairs. In this way it strong than democracy.

Question 4.
Who introduced the Gram Panchayat system in India?
Answer:
In 1882 Lord Rippon, who was the Viceroy of India at that time took the initiative in creating local governments in India. At that time they were called the Local boards. Lord Rippon is called the pioneer of Local governments in India. He wanted more and more involvement of the local people in solving the local problems. He was also in favor of the decentralization of the administration.

Question 5.
How Local governments strengthen democracy?
Answer:
In a democracy, the issues and problems should be given to the hands of the local people and their representatives because they are more familiar with their affairs and problems. They can have better control over the decision-makers and local administrators. We can say that strengthening local government means strengthening bf local democracy.

Question 6.
Discuss Mahatma Gandhi’s concept of Local government.
Answer:
Mahatma Gandhi was a very strong supporter of grass-root democracy for which he advocated the organization of local governments in rural and urban areas. He wanted to achieve economic and political decentralization through there; local governments. It is because of the wish of Mahatma Gandhiji that the concept of local government was introduced in the chapter of Directive Principles of State Policy.

Question 7.
Write the Government structure in rural areas per Balwant Rai Mehta Committee Recommendation.
Answer:
Governments of India appointed a committee under the Presidentship of Balwant Rai Mehta to go into the issue of organization and powers of the local governments in rural areas. This gave its recommendation in 1957 and suggested following a three-tier structure in rural areas.

  1. Village Panchayats at the village level.
  2. Block Samiti at Block level.
  3. Zila Parishad at the district level

Question 8.
What attention was given to local governments in the constitution?
Answer:

  1. Local governments did not get due attention in the Constitution
  2. The issue of local governments was left for the State governments
  3. The issue of Local governments was added in the chapter of Directive Principles of State Policy which is non-justiciable and primarily advisory in nature.
  4. Location was considered a threat to national integration.

Question 9.
Give four lessons for their poor functioning.
Answer:
In post-independent India, in many states, local governments were organized in rural and urban areas but they did not get the desired results became of the following reasons:

  1. Factional politics in the village
  2. Caste ridden society of the village
  3. Financial problems
  4. Bureaucratic apathy
  5. Violence

Question 10.
What was the position of women and scheduled castes in local governments?
Answer:

  1. Women were inadequately represented
  2. Women were discouraged in raising any local issue particularly related to the women.
  3. There were social taboos which became a hindrance in the participation of woman.
  4. Scheduled castes were also poorly represented as there was upper caste domination in rural society.

Question 11.
Write four main provisions of the 73rd amendment of the constitution.
Answer:

  1. Local governments were given a constitutional base.
  2. Elections to the local governments were made direct.
  3. The tenure of these elected bodies was fixed.
  4. 33% seats were reserved for women and 33% seats were reserved for the scheduled castes.

Question 12.
What is Block Samiti and what is its function?
Answer:
There is a three-tier structure in rural areas. At the village level, there is Gram Panchayats. At block, the level is the Block Samiti and at the district level, Zila Parishad works. Therefore Block Samiti stands between Village Panchayat and Zila Parishad.

The main concern and responsibility of the Block Samiti is the development of rural area block level and fulfill the agriculture needs of the people. It acts as a link between Zila Parishads and village Panchayats. It controls and supervises the village Panchayats in Block.

Question 13.
Write the function of Panchayats.
Answer:

  1. To discuss the issues and problems of the village
  2. To maintain law and order and co-operation in the village.
  3. To fulfill the agriculture-based needs of the village
  4. To co-ordinate with the government through the Block Samitis and Zila Parishad for the development of the village.
  5. To get and utilize the aids from the government
  6. To decide the disputes of the village.

Question 14.
How the reservations of women in local government have improved the status of women.
Answer:
The rural society is male-dominated which is why so far there is no adequate representation of women in local government. To ensure adequate representation of the women 33% reservation is made which has certainly helped in increasing the participation of women in politics and democratic institutions. Now there is a change in socio, economic status of women.

Question 15.
What are the powers of Local government?
Answer:
The Constitutional amendments (73rd and 74th) assigned as many as 29 subjects to the local governments. All these subjects are related to functions linked to local welfare and developmental needs of the people. But still lot of work is to be done to make their local governments truly representative functional and powerful.

Local Governments Important Extra Questions Short Answer Type

Question 1.
Discuss the meaning of Local Government.
Answer:
The local government refers to the institutionalized structures for governance at the local level both in rural as well as urban areas. Local governments are the mechanism of decentralized administration with the joint co-operation and co-ordination between the government’s machinery (Bu-reaucracy) and the people themselves. The concept of local government is based on the thesis that the local issues and problems of the people should be solved by the local people themselves because they know their problems better. The role of the Government should be only of facilitator and advisor.

As a part of democratic decentralization, the government should allow the local governments to raise and utilize their resources. The concept of local government aims at strengthening the people and democracy.

Question 2.
What is the significance of the Local government?
Answer:
Local governments can play a significant role in strengthening Grass-root democracy and so the state and national democracy. Local Governments promote responsibility and accountability. These institutions provide opportunities for active participation and involvement of the local people in the decision-making process. Local government is significant in the promotion of efficiency and decentralization of administration. This institution helps in solving the local people by their cooperation. Local people get the oppor¬tunity to control the administration. ‘ ‘

Question 3.
Trace the development of local governments in India.
Answer:
The concept and existence of Local governments are not new to India. There has been the existence of Local governments in different names in ancient India. They enjoyed good position and powers. In the course of time, ) these village bodies took the shape of Panchayats (an assembly of five persons) and there five persons solved the issues at the local level. Their role and position kept on changing at different points in time. However, in medieval periods the status and significance of the Local governments eroded due to changes in socio-economic and political scenarios.

During the British period, the significance of the Local government got the attention of Lord Rippon (Then Viceroy of India) who tried to strengthen them. During the independence movement, many leaders including Mahatma Gandhi demanded the strengthing of Local government. After Independence, we have the existence of Local governments but it is the subject of the state.

Question 4.
Discuss the constitutional position of the Local government.
Answer:
The issue of local governments was also discussed in the Constituent Assembly but the idea of decentralization of powers did not get adequate favor because of turmoil due to the partition of India. Most of the leaders wanted a strong center. Too much localism was considered a threat to national integration. At the same time, many leaders wanted decentralized administration and the participation and involvement of the people involving the local issues. Therefore the subject of local government is added in the chapter of Directive Principles of State Policy in part in the Constitution.

Question 5.
Discuss the working of local government in the post-independence period.
Answer:
As said earlier that local government becomes the state subject but this did not get the proper attention of the local government, so could not produce results due to the following reasons :

  1. Bureaucratic apathy
  2. Poor financial position
  3. Dy-functional for long times
  4. Poor representation of women and scheduled caste
  5. The dominance of upper castes
  6. Inadequate powers
  7. The dominance of anti-social elements
  8. Political interference
  9. No recognition of local initiatives
  10. Dependence of Local government on State and Centre.

Question 6.
What was Balwant Rai Mehta Committee? Report?
Answer:
After the implementation of the Indian Constitution, the Indian government appointed a committee under the Chairmanship of Balwant Rai Mehta to study the issue of Local government in the light of the issue of Local government given in the IVth Part of the Constitution in the chapter of Directive Principles of State Policy and recommended the mode of structure and function of these local governments in a rural area. Balwant Rai Mehta committee studied and discussed the issue in detail and gave its recommen¬dation which included three-tier structure in a rural area as under.

  1. Village Panchayat at the village level
  2. Block Samiti at the block level
  3. Zila Parishad at the district level

Question 7.
Discuss the background of the 73rd and 64th constitutional amendments.
Answer:
As the Local governments were not doing well in most of the states^ except Gujarat and Rajasthan, the issue of strengthening the focal Government by restructuring them had always invited the attention of the political leadership states as well as the center. Many times this issue was raised at different levels.

When Narshima’s Congress Government came to power in 1991, he took the issue of strengthening and restructuring the Local government seriously.

In fact, he meant business. He brought two constitutional amendments which were passed in 1992 and came into force in 1993. All the states were directed to change their laws of Local government as per the provisions of the 73rd and 74th amendments.

Question 8.
Write main features of 73rd and 74th constitutional amendments.
Answer:
It was an exhaustive amendment and brought about far-reaching changes in the existing structure and status of the local governments. The main provisions are as under:

  1. The Local Governments were given constitutional status.
  2. Elections for these local governments were made direct.
  3. The tenure of these local government was fixed for five years.
  4. 33% reservation was made in total seats for. woman and 33% seats reserved for .the Scheduled castes.
  5. Provision of state Election Commissioner was made.
  6. Provision of the setting of state finance commission was made.

Question 9.
Discuss the utility of reservation of seats for women and scheduled castes and OBC in Local Government.
Answer:
Before the 73rd amendment and 74th amendment, women and weaker sections did not get adequate representation in proportion to thick numerical strength in the Local government because of the socio-economic structure’ at the grass-root level.

It is because of the provision of 33% reservation for women and 33% reservation for Scheduled castes in the 73rd and 74th amendment, these people have started to get the due representation in the local government. As a result of this representation, the^ have got empowered and are actively participating in the decision-making process. It has promoted their political recruitment and also has enhanced their social and economic status.

Question 10.
Discuss the composition of the state finance commission.
Answer:
As per the 73rd constitutional amendment, the state government is also required to appoint a state finance commission once in five years. This commission would examine the financial position of the local government in the state. It would also review the distribution of revenue between the state and local government on one hand and between rural and urban local government on the other hand. State Finance will monitor the financial health of the local government and will ensure the proper allocation and utilization of funds. It will also use the misuse of funds at the political and bureaucratic levels.

Question 11.
Discuss the main provisions of the 74th amendment.
Answer:
Local bodies at the urban level are organized to meet the needs of the urban areas. Since the needs of urban areas are different, their composition and functions are a bit different. 28% people in India like in urban area as per census 200%. The urban local Government is organized on the basis of population. The census of India defines an urban area as having.

  1. a minimum population of 5000
  2. 75% of the male working population engaged in non-agricultural occupations
  3. a density of population of a learned 400 persons per sq km.

74th amendment is a repetition of the 73rd amendment in matters of the election, tenure, reservation, transfer of subject, state Election Commissioner, and Finance Commissioner. Functions of the Local Governments have been listed in the Eleventh Schedule of the Constitution.

Question 12.
Discuss the implementation of the 73rd and 74th Amendments of the constitution.
Answer:
Local Government has been given constitutional status therefore as per the 73rd and 74th Amendments it has become mandatory for all states to changes their laws about local Government accordingly. Today there are nearly 500 Zila Parishad, about 6000- Block Samitis, and about 2.50,000 gram panchayats in rural areas in India. There are about 100 city corporations, 1400 town municipalities, and over 2000 Nagar Panchayats in urban India. More than 32 lakh members are elected to these local bodies for five years out of this 32 lakh, 10 lakh members are women 73rd and 74th amendments have brought uniformity in urban and rural local Governments.

Question 13.
State the improvement in the status of women as a result of reservation in seats in Local government.
Answer:
The provision of reservation for women at the Panchayat and Nagar Palika has ensured the presence of a significant number Of women in local bodies. As this provision of the reservation is also applicable for the position of Sarpanch and Presidents a large number of women elected representatives have been able to occupy these positions. This could become possible only because of the provision of reservation. There are at least 200 women Presidents in Zila Parishad another 2000 women are the Presidents of Block Samitis and more than8000 women are occupying the position of Sarpanch in Gram Panchayat.

Similarly, we have 30 women Mayors in corporations over 500 women our President of Town municipalities, and nearly 650 Nagar Panchayats are headed by women. This shows the sharp rise in the status of women as a result of reservation by the 73rd and 74th Amendments.

Question 14.
State the improvement in the status of scheduled castes and scheduled tribes as a result of reservation by the 73rd and 74th Amendments.
Answer:
Before the 73rd and 74th amendment, certain seats were reserved for Scheduled castes and Scheduled tribes in Lok Sabha, Rajya Sabha, and State Assemblies. There was no reservation for Scs. and Sts. in Local Government. In spite of adult Franchise, the Scheduled castes and Scheduled tribes remained underrepresented due to socio-economic structure. Liberal economic and social policies also did not ensure adequate representation of scheduled castes and tribes in social government. But now due to the provision of reservation of 33% seats reserved for Scheduled caste, the situation is changed appreciably and the people of Scheduled castes and

Scheduled tribes have been able to occupy the positions in local bodies. As the Indian population has 16.2 percent Scheduled castes and 8.2 percent Scheduled tribes, about 6.6 lakh elected members in the urban and local bodies • to belong to their two communities. This shows the rise of the status of the people of their two communities as a result of the 73rd and 74th Amendments.

Question 15.
Assess the performance of local government after the 73rd and 74th amendments.
Answer:
Local Governments are subjected not only to structure changes but wide powers are also given to them through 73rd and 74th subjects over the local resources. Now many important subjects have been brought up in the presence of local governments. Now there is real decentralization of powers and responsibility which is the result of democracy.

Now the structure of the local Government includes all the elected persons. Scheduled castes, back¬ward castes, women, bureaucratic officers, and politicians with combined opinions and co-operation of all the decisions are taken. With the addition of new subjects, the area of activities of local bodies is increased. Therefore we can say that the laws about the local governments are an important step in the direction of democratization and decentralization. We should make these provisions more and more real and practical.

Local Governments Important Extra Questions Long Answer Type

Question 1.
Discuss the main provisions of the 73rd and 74th amendments and also assess their significance.
Answer:
73rd and 74th amendments regarding the local government were passed in 1992 by the Narsimha Government and they came into force in 1993. The main provisions of these amendments are as under

  1. Direct election to the local government at different levels.
  2. It also provided a three-tier structure at the rural level
  3. It also provided fixed and uniform tenure for local governments
  4. It provided 33% reservation of seats for women and 33% reservation of seats for Scheduled castes.
  5. Constitutional status for local bodies
  6. Constitution for state Election commissioner
  7. Composition for state Finance commissioner
  8. Transfer of subjects to local bodies means giving more powers

Impact of these amendments

  1. Due to constitutional status, all the states were made to make their laws of local governments as per the provision of the 73rd and 74th Amendments.
  2. Now elections are held at regular intervals in all the states and their tenure is fixed uniformly.
  3. Women have got adequate representation
  4. Scheduled caste also have got adequate representation
  5. The status of women and scheduled castes is improved
  6. The local bodies have become more function
  7. Grass root democracy is strengthened.
  8. They have become the instrument of decentralization.